You are on page 1of 136

Cathode Ray Oscilloscope (CRO)

I
21.1 INTRODUCTION into voltages with the help of transducers and thus to
present visual representations of a wide variety of
The cathode ray oscilloscope (CRO) is a very
useful and versatile laboratory instrument used for dynamic phenomena on CROs.
display, measurement and analysis of waveforms and Cathode Ray Oscilloscopes are also used to
other phenomena in electrical and electronic circuits. investigate waveforms, transient phenomena, and
CROs are in fact very fast X-Y plotters, displaying an other time varying quantities from a very low
input signal versus another signal or versus time. The frequency range to the radio frequencies.
"stylus" of this "plotter" is a luminous spot which Oscilloscopes have been evolved continuously,
moves over the display area in response to an input and they are now available which can measure
voltage. The luminous spot is produced by a beam of frequencies upto 1 GHz, and observer events as small
electrons striking a fluorescent screen. The extremely as 20 Hz in duration.
low inertia effects associated with a beam of electrons
Many additional features are available with some
enables such a beam to be used for following the
oscilloscopes and these include built in digital multi­
changes in instantaneous values of rapidly varying
meters and counters. The oscilloscopes are progres­
voltages. The extremely low inertia of electrons as
sively getting smarter, and many are microprocessor
compared to metallic conductors used in an
controlled. They have the ability to calculate several
electro-mechanical Duddell's oscilloscope enables a
CRO to be used at frequencies much above the highest features, such as rise time or pulse width of the mea­
on which the Duddell's oscilloscope can be used. sured waveform, and to display these values along
with the display of waveforms. They are easier to use
The normal form of a CRO uses a horizontal input
and internal routines often act as a guide for the user,
voltage which is an internally generated ramp voltage
and display a warning if there is any error in setting.
called "Time Base". This horizontal voltage moves the
luminous spot periodically in a horizontal direction Many oscilloscopes are now available with IEEE
from left to right over the display area or screen. The 488 bus capabilities, so that they can be used as a part
vertical input to the CRO is the voltage under of measurement test bed, with the instrument's
investigation. The vertical input voltage moves the controls set at a remote location, and the readings
luminous spot up and down in accordance with the digitised and retrieved for the purposes of recording
instantaneous value of the voltage. The luminous spot and analysis.
thus traces the waveform of the input voltage with Although, most oscilloscopes are monochromatic,
respect to time. When the input voltage repeats itself colour oscilloscopes are finding increasing applications
at a fast rate, the trace (display) on the screen appears in computers and in television.
stationary on the screen. The CRO thus provides a The applications of oscilloscopes have been
means of visualizing time varying voltages. As such, enhanced on account of many recent developments.
the CRO has become a universal tool in all kinds of Most present day oscilloscopes are capable of
electrical and electronic investigations. accepting two or more inputs displaying them
Cathode Ray Oscilloscopes operate on voltages. simultaneously. This may be achieved through using a
However, it is possible to convert current, strain, split beam or by using a multiple beam tube. Sampling
acceleration, pressure and other physical quantities oscilloscopes are used for high speed applications.
(641)
642 Electrical and Electronic Measurements and Instrumentation

These oscilloscopes employ time sampling and After leaving the electron gun, the electron beam
through their use it is possible to measure signals of passes through two pairs of "Electrostatic deflection
about 20 GHz. They can only detect a repetitive plates". Voltages applied to these plates deflect the
waveform, and work on the principle of taking a beam. Voltages applied to one pair of plates move the
sample once every cycle, over several cycles, each beam vertically up and down and the voltages applied
sample point being shifted from the previous point. to the other pair of plates move the beam horizontally
The complete picture of the waveform is stored, and form one side to another. These two movements i.e.,
can be displayed as a stationary signal. horizontal and vertical are independent of each other
Storage oscilloscopes can be used for capturing and thus the beam may be positioned anywhere on the
transient signals, and then display them for periods screen.
which may vary from a few minutes to several years. The working parts of a CRT are enclosed in an
An analog oscilloscope uses a modified form of a evacuated glass envelope so that the emitted electrons
conventional cathode ray tube to store the trace. The are able to move about freely from one end of the tube
digital storage oscilloscope first converts the analog to the other.
signal to a digital form and stores it in digital memory.
The signal can then be recalled for display as and 21.3 ELECTRON GUN
when required. The source of focused and accelerated electron
beam is the electron gun. The electron gun, which
21.2 CATHODE RAY TUBE (CRT)
emits electrons and forms them into a beam consists of
A cathode ray oscilloscope consists of a cathode a heater, a cathode, a grid, a pre-accelerating anode, a
ray tube (CRT), which is the heart of the tube, and focusing anode and an accelerating anode.
some additional-circuitry to operate the "CRT. The
In smaller CRTs, connections to the various
main parts of a CRT are :
electrodes are brought out through pins in the base or
(z) Electron gun assembly, the tube as shown in Fig. 21.1. Large and medium
(zz) Deflection plate assembly, sized high performance tubes operate at very high
(z'n) Fluorescent screen, voltages, and these leads are usually brought out
through the sides of the glass envelope.
(iv) Glass envelope,
Electrons are emitted from the indirectly heated
(v) Base, through which connections are made
cathode. A layer of barium and strontium oxide is
to various parts.
deposited on the end of the cathode-which is a
The main parts of a CRT are shown in Fig. 21.1. cylinder - to obtain high emission of electrons, at
Before going into details of working of various parts of moderate temperatures. The typical values of current
a CRT, a summary of functions of the different parts is and voltage required by an indirectedly heated
given below : cathode are 600 mA at 6.3 V. High efficiency systems
The “Electron gun assembly" produces a sharply use 300 mA at 6.3 V. The special low power designs
focused beam of electrons which are accelerated to use 140 mA at 1.5 V. These electrons pass through a
high velocity. This focused beam of electrons strikes small hole in the "control grid". This control grid is
the fluorescent screen with sufficient energy to cause a usually a nickel cylinder, with a centrally located hole,
luminous spot on the screen. co-axial with the CRT axis. This is usually a metal cup
of low permeability steel, about 15 mm in diameter
Electron gun Aquadag and 15 mm long. An aperture of about 0.25 mm is
-Vertical coating
Pre-accelerating deflection drilled in the cap of the grid for the electrons to flow
anode plates through. The intensity of electron beam depends upon
E ectron the number of electrons emitted from the cathode. The
beam grid with its negative bias controls the number of
electrons emitted from the cathode and hence the
Grid A >
intensity is controlled by the grid.
Cathode Accelerating Horizontal
The electrons, emitted from the cathode and
anode deflection
plates phosphor passing through the hole in the control grid are
screen accelerated by the high positive potential which is
applied to the "pre-accelerating" and "accelerating
Fig. 21.1 Internal structure of a CRT. anodes".
Cathode Ray Oscilloscope (CRO) 643

The electron beam is focused by the "focusing Figure 21.3 shows two concentric cylinders with a
anode". The accelerating and focusing anodes are potential applied between them. Lateral repulsion
cylindrical in form, with small openings located in the again causes the spreading of the flux lines producing
centre of each electrode, coaxial with the tube axis. a field as shown. The equipotential surfaces are shown
After leaving the focusing anodes, the electron beam as solid lines. It is clear from the diagram that the
passes through the vertical and horizontal deflection
plates and then goes on to the fluorescent screen.
The pre-accelerating anode and the accelerating
anode are connected to a common positive high
voltage of about 1500 V. The focusing anode is
connected to a lower adjustable voltage of 500 V.
There are two methods of focusing an electron
beam :
(i) Electrostatic focusing and ■smmbbhim
Fig. 21.3 Field between two co-axial cylinders.
(ii) Electromagnetic focusing.
The CRO uses electrostatic method of focusing as equipotential surfaces are curved. Let us consider the
compared to a TV picture tube which employees region on the two sides of an equipotential surface S as
electromagnetic focusing. shown in Fig. 21.4. The potential on the left side of the
surface is -V and on the right \ side is + V. Let an
21.4 ELECTROSTATIC FOCUSING electron moving in a direction AB enter the area to the
Figure 21.2 shows an electron at rest placed in an left of S. This electron experiences a force which is
electric field produced two parallel plates. Force on normal to the surface S and is thus accelerated.
the electron is,
F = -ee newton ...(21.1)
where e = electric field intensity ; V/m,
and e = charge of electron =1.602 x 10-19 C
The minus sign indicates that the force acts in the
opposite direction to that of the field. The above
discussion is valid only if the electron is situated in a
field of uniform intensity. In practice, however, the
field is not uniform. The lateral repulsion of the
electric field lines causes a spreading of space between
the lines, resulting in curved field lines at the ends. Since the force acts in a direction normal to the
Thus the field intensity will be less at the ends. surface, it is the normal component of velocity that is
Figure 21.2 also shows equipotential surfaces, increased after refraction while the tangential
indicated by solid lines. Since the force is in a direction component remains the same.
opposite the field and the equipotential surfaces are The tangential components are :
perpendicular to the field, the force on an electron is in
yfl = sin0; and vf2 = i?2sin0^
a direction normal to the equipotential surfaces.
Now vtl = vf2 or v1 sin0. = v2sin0r
sin0. _ u2
...(21.2)
sin0r V1
where
= initial velocity of electrons,
v2 = velocity of electrons after leaving surface S,
0;. = angle of incidence, and
0r = angle of refraction.
Equation 21.2 is identical to the expression relating
Fig. 21.2 Electric field between parallel plates. the refraction of a light beam in geometrical optics.
644 Electrical and Electronic Measurements and Instrumentation

The refraction of an electron beam follows as the direction. There is no force either in X direction or Z
bending of a light beam at a refracting surface such as direction and hence there will be no acceleration of
an optical lens. For this reason the focusing system in a electrons in these directions.
CRT is known as electron lens.
Figure 21.5 shows the functional diagram of an
electrostatic focusing arrangement. The pre-accelera­
ting anode, which is a metal cylinder containing many
baffles, collimates the electron beam which enters it
through a small opening on the left hand side. The
pre-accelerating anode is connected to a high positive
potential.

Preaccelerating Focusing Accelerating

Let
Eo = voltage of pre-accelerating anode ; V,
e = charge of an electron ; C,
//i = mass of electron ; kg,
vQX = velocity of electron when entering the field
of deflecting plates ; m/s,
Ed = potential between deflecting plates ; V,
Fig. 21.5 Electrostatic focusing arrangement. d = distance between deflecting plates ; m,
ld = length of deflecting plates ; m,
The focusing anode and the accelerating anodes
L = distance between screen and the centre of the
are co-axial with the pre-accelerating anode. The pre­
deflecting plates ; m, and
accelerating and accelerating anodes are connected to
the same potential while the focusing anode is D = deflection of electron beam on the screen in
connected to a lower potential. Y direction ; m.
On account of difference of potential between The loss of potential energy (P.E.) when the
focusing anode and the two accelerating anodes a electron moves from cathode to accelerating anode ;
non-uniform field exists on each of the two ends of the P.E. =eEn ...(21.3)
focusing anode. The equipotential surfaces, thus, form The gain in kinetic energy (K.E.) by an electron
a "double concave lens". K.E. =(l/2)nw2x ...(21.4)
The electron beams entering the field at angles
(The mass of an electron is m = 9.109 x ICT31 kg)
other than the normal to the equipotential surfaces,
Equating the two energies, we have :
will be deflected towards the normal and the beam is
thus focused towards the centre of the tube axis. By =(2<-E„/m)1/2 -(215)
changing the voltage of the focusing anode, the This is the velocity of the electron in the X
refractive index of the electron lens is changed and direction when it enters the deflecting plates. The
therefore the focal point of the beam can be changed. velocity in the X direction remains the same through­
The change in voltage is brought about by changing out the passage of electrons through the deflecting
the setting of a potentiometer. This control is brought
plates as there is no force acting in this direction.
to the front panel of CRO and is marked Focus. The electric field intensity in the Y direction,
^ = Ed/d ...(21.6)
21.5 ELECTROSTATIC DEFLECTION Force acting on an electron in Y direction
Figure 21.6 shows a general arrangement for Fy=e£y =eEd/d ...(21.7)
electrostatic deflection. There are two parallel plates
Suppose fly is the acceleration of the electron in Y
with a potential applied between. These plates
direction therefore,
produce a uniform electrostatic field in the Y direction.
Thus any electron entering the field will experience a F, =
force in the Y direction and will be accelerated in that or ay=eey!m ...(21.8)
Cathode Ray Oscilloscope (CRO) 645

As there is no initial velocity in the Y direction the usually a time varying quantity and the
displacement 1/ at any instant t in the Y direction is : image on the screen thus follows the varia­
tions of the deflection voltage in a linear
...(21.9) manner.
' 2 -,/ 2 m
As the velocity in X direction is constant, the (iii) The deflection is independent of the e/tn
displacement in X direction is given by : ratio. In a cathode ray tube, in addition to the
electrons many types of negative ions such
x= ...(21.10) as oxygen, carbon, chlorine etc. are present.
f = x/v,„ ...(21.11) With electrostatic deflection system, because
Substituting the above value of t in Eqn. 21.9, we deflection is independent of e/m, the ions
have : travel with the electrons and are not
11 ee 1/ concentrated at one point. Hence cathode
2
y = tx ...(21.12) ray tube with electrostatic deflection system
2 "Wox
does not produce an ion burn.
This is the equation of a parabola. Deflection sensitivity. The deflection sensitivity
The slope at any point (x,y) is of a CRT is defined as the deflection of the screen per
dy _ gev unit deflection voltage.
...(21.13) Deflection sensitivity
dx "rfc

Putting x = ld in Eqn. 21.13, we get the value of S = — = —m/V ...(21.17)


£,a 2dEna
tan0.
ee„ eF I The Deflection Factor of a CRT is defined as the
tan0 =—...(21.14) reciprocal of sensitivity.
mvox mdvox
Deflection factor
After leaving the deflection plates, the electrons 1 2dE
travel in a straight line. The straight line of travel of G = — =---- - V/m ...(21.18)
S Lld
electrons is tangent to the parabola at x = ld and this
tangent intersects the X-axis at point O'. The location It is clear from Eqn. 21.17, that the sensitivity can
of this point is given by : be increased by decreasing the value of accelerating
x = _y_ = ^L /JY, voltage Ea. But this has a disadvantage as the
tan 9 2zn^r/ mv2
gx d 2 luminosity of the spot is decreased with decrease in
E . On the other hand a high value of £ , produces a
The apparent origin is thus at the centre of highly accelerated beam and thus produces a bright
deflection plates. The deflection D on the screen is spot.
given by : However, a high accelerating voltage (£;)
Le E, I.
D=£tan0=------4/ ...(21.15) requires a high deflection potential (£rf) for a given
mdvox deflection. Also highly accelerated beam is more
2 2eE difficult to deflect and is sometimes called hard beam.
Substituting the value vox =----- - in Eqn. 21.15, we
m Typical values of sensitivities are 0.1 mm/V to
get 1.0 mm/V corresponding to deflection factors of
LeE.l, in LI, E, 10 V/mm and 1 V/mm.
D =----- = ...(21.16)
md 2eEan 2dEna
21.6 POST DEFLECTION ACCELERATION
From Eqn. 21.16, we conclude : OF ELECTRON BEAM
(i) For a given accelerating voltage Ea, and for After electrons pass beyond the deflection plates,
particular dimensions of CRT, the deflection they may or may not experience additional
of the electron beam is directly proportional acceleration. This depends primarily upon the
to the deflecting voltage. This means that the maximum frequencies to be applied to CRT. In an
CRT may be used as a linear indicating electrostatic system of deflection, Eqn. 21.17 indicates
device. that a low value of accelerating voltage, £fl, should be
(n) The discussion above assumes that Ed is a used. This voltage is usually kept below 4 kV.
fixed d.c. voltage. The deflection voltage is Although this voltage gives good sensitivity but
646 Electrical and Electronic Measurements and Instrumentation

reduces brightness, which can be seriously impaired Both the systems illustrated in Fig. 21.7 are
at high frequencies. In general, mono-accelerator limited in scan angle to between 35° and 40°. An
tubes such as shown in Fig. 21.1 may be used, if the alternative PDA system, called a high expansion mesh
maximum frequency to be displayed is below 10 MHz. tube, uses a spherical shaped mesh without a helix
However, if signals of frequencies higher than 10 MHz tube. The mesh is shaped so as to expand the
are to displayed, post deflection acceleration is necessary deflection of the beam, and with this system beam
to increase the brightness of the trace which otherwise deflections up to 90° be obtained. However the spot
would be dim. size is also increased in proportion to the increase in
When measurements are to be made above a sensitivity.
frequency of 10 MHz, post-accelerators or post deflection
21.7 EFFECT OF BEAM TRANSIT TIME
acceleration tubes (PDA) are used. These have a large
AND FREQUENCY LIMITATIONS
beam accelerating bias after the deflection plates so as
to increase the beam energy and give a bright display In the derivation of Eqn. 21.16, the plate voltage
on the screen. Ed is assumed constant during the motion of the
electrons through the deflecting field. If the voltage
Several types of PDA structures can be used.
applied to the vertical deflecting plates changes during
Figure 21.7(a) shows a spiral accelerator in which a
the transit time of the electrons through the hori­
high resistance narrow spiral of graphite is painted
zontal plates, the deflection sensitivity gets decreased.
over a considerable length of the inside of the envelope
funnel. The spiral is connected to the aluminium film Transit time f, =—— ...(21.19)
on the phosphor, if present. A voltage of about 10 kV 1 vOX
is applied to this spiral, and the effect is to accelerate The transit time imposes a limitation of the upper
the beam, but also to bend it slightly towards the frequency limit. An upper limiting frequency is
X-axis. This decreases the deflection sensitivity, which defined as that frequency at which the transit time is
can be compensated for by increasing the length of the equal to one quarter of the period of the voltage
tube. Alternatively spherical mesh may be inserted applied to vertical plates.
into the helix tube, as in Fig. 21.7(b). This shapes the Upper limiting frequency,
accelera- ting field and prevents it from affecting the 1 v
original beam deflection, so giving the same f ...(21.20)
Jc 4fT 4Z
sensitivity as a mono-accelerator tube of equal length.
The frequency range of the oscilloscopes can be
increased by subdividing the deflecting plates in a
number of sections in the path of the electron beam.
The voltage being measured is applied to the vertical
plates through an iterative network, whose propagation
time corresponds to the velocity of electrons, thereby
the voltage applied to the vertical plates is made to
synchronize with the velocity of the beam. The use of
this technique allows the cathode ray oscilloscope to
be used upto frequencies of 500 MHz and above.

21.8 DEFLECTION PLATES


Referring to Fig. 20.1, we observe that the electron
beam, after leaving the electron gun, passes through
two pairs of deflection plates. One pair of plates is
mounted horizontally and produces an electric field in
the vertical plane. This pair produces a vertical
deflection and is thus called Vertical Deflection
Plates or Y Plates. The other pair of plates is mounted
vertically and produces a horizontal deflection. This
pair of plates is called Horizontal Deflection Plates or
X Plates. The plates are flared so as to allow the beam
Fig. 21.7 CRT post accelerators : (a) spiral (b) mesh. to pass through them without striking the plates.
Cathode Ray Oscilloscope (CRO) 647

21.9 SCREEN FOR CRTs The electron beam on striking the phosphor gives
The front of the CRT is called the face plate. It is off light and heat. Excessive beam current on one spot
flat for screen sizes up to about 100 mm x 100 mm, and of the screen, over a long time, can degrade the light
is slightly curved for larger displays. The face plate is output from the phosphor, and can bum a hole in it.
formed by pressing molten glass in a mould and then Phosphors can be classified as having low burn
annealing it. Some CRTs have a face plate which is made resistance (P19, P26, P33), medium bum resistance (Pl,
entirely from fibre optics, which has special charac­ P2, P4, P7, Pll) and high burn resistance (P15, P31).
teristics. It is also feasible to have a small fibre optic The luminance of the phosphor is a measure of its
region which gives direct contact exposure of photo­ brightness. It is determined by the luminance effi­
graphic film, if required, without needing a lens system. ciency of the phosphor, and by the beam energy,
The inside surface of the face plate is coated with which is a product of the beam current density, the
phosphor. This consists of very pure inorganic accelerating potential and the writing time. Therefore
crystalline phosphor crystals, about 2-3 microns in one can have high light output, at low beam accele­
diameter, to which traces of other elements, called rating potential provided the sweep speed is low, but
activators, have been added. Activators in current use for fast single-shot transient recording, a high beam
are metals such as silver, manganese, copper and current and accelerating voltage, and phosphor with
chromium, and they affect the characteristics of the
high luminance, are required.
phosphor, such as its luminous efficiency, spectral
Each phosphor has a unique colour, the human
emission and persistence.
eye response peaks at 555 nm, in the yellow-green
Many types of phosphors are used for CRTs and
region, so phosphors which gives light in these regions
the characteristics of a few of them are given in
(P2, P31, P39) would appear to be brighter than others
Table 20.1. A phosphor converts electrical energy to
such as Pll, P16, P19. A camera film, however, is
light energy. When an electron beam strikes phosphor
sensitive to ultraviolet or blue, so for high speed
crystals it raises, their energy level. This is known as
cathodoluminescence. Light is emitted during phosphor photography phosphors Pll or P16 may be better. The
excitation and this is called fluorescence. When the spectral emission from a phosphor is broadband, i.e., a
electron beam is switched off the phosphor crystals range of wavelengths go to determine its colour. The
return to their initial state, and release a quantum of gradation of colour (hue) and the chromatic purity
light energy. This is called phosphorescence or persistence. with respect to white (saturation) can be precisely
The writing speed of a phosphor is measured by specified by X and Y co-ordinates on a chromaticity
its fluorescence rise time, which is the time from the diagram.
beginning of excitation to reach 90% of the maximum A thin film of metal, such as aluminium (0.1 - 0.15
emission state, and by the decay time, which is the pm) is usually deposited on the non-viewing side the
time to fall from the maximum state to the 10% level. phosphor.
Writing speed is determined by the phosphor type, This has three effects :
crystal size, impurity content, and the manufacturing
(z) The metal layer acts as a heatsink and
process. A phosphor must be refreshed by electrons,
reduces the danger of phosphor burn.
before the end of its decay time, in order to give a
flicker free display. Short persistence phosphors (zz) The light scatter from the phosphor is
require more frequent refreshes, whereas long reduced, and the aluminium reflects light
persistence phosphors may result in characters fading going back into the tube towards the viewer,
slowly, giving a trace after glow or trails. Long so increasing the brightness.
persistence phosphors are also more prone to (zzz) Electrons which strike the phosphor release
permanent discolourations, and loss of luminous secondary electrons. These secondary
efficiency, at high beam current or fixed data use. electrons are usually collected by the
Most modem oscilloscopes use short persistence aquadag. Under certain conditions the
phosphors (e.g., Pl, P2, Pll, P31) because of their fast electrons may remain on the screen, and
refresh capability, whilst the slower speed signals seen cause a negative voltage build up. This
in medical applications need longer persistence would decrease the accelerating voltage of
phosphors (P7 and P39). For use in radars, very long the beam. This charge build up is prevented
persistence is needed (Pl 9, P26, P33). Long persistance by the aluminium layer, which is connected
is even needed in storage type oscilloscopes. electrically to the aquadag layer.
6^-8 Electrical and Electronic Measurements and Instrumentation

Table 21.1 Phosphor Characteristics


Colour Persistence Lumi­ Writing
Type Intended use
I Fluorescent Phosphorescence (to 10% level) *nance speed

1 P1 yellow-green yellow-green 1 - 100 ms 4 7 oscilloscope, radar

P2 yeliow-green yellow-green 1 - 100 ms 2 4 oscilloscopes

P4 white white 10 gs -1 ms 3 3 monochrome television


radar, medical

P7 blue yellow 10 gs -1 ms(b) 4 2 radar, medical

100 ms - 1 s(y)

Pll blue blue 10 gs -1 ms 5 1 photographic

j P15 ultra-violet green < 1 gs (uv) 9 8 TV (flying spot scanner),

1 - 10 gs (g)
P16 ultra-violet ultra-violet < 1 gs 10 8 TV (flying spot scanner),
photographic

; P18 white white 10 gs -100 ms - - projection TV

: Pi9 orange orange > 1 s 5 10 radar


I P22 white white 10 gs -1 ms - - tricolour TV
(red, blue, green) (red, blue, green)

P26 orange orange > 1 s 8 10 radar

P28 yellow-green yellow-green 100 ms - 1 s 3 5 radar, medical

P31 green green 10 gs -1 ms 1 3 oscilloscopes, bright TV

P33 orange orange > 1 s 6 9 radar

P39 yellow-green yellow-green 100 ms -Is 3 6 radar, computer graphics


| P40 blue yellow-green 10 gs 1 ms(b) — — low repetition rate P16

100 ms - ls(yg)

P44 yellow-green yellow-green 1 - 100 ms 3 - bistable storage

P45 white white 1 - 100 ms 4 - monochrome TV display

♦ Highest luminance or fastest writing speed.

The electron beam needs more energy to 21.10 GRATICULE


penetrate the aluminium layer in front of the The graticule is a grid of lines that serves as a scale
phosphor, fherefore at low acceleration potentials the when making time and amplitude measurements.
aluminised phosphor screen has lower efficiency than
There are three types of graticules.
a standard screen, but the efficiency increases rapidly
at higher potentials. (/) External graticule. This is scribed on a
The idea! phosphor for a laboratory oscilloscope Plexiglas acrylic plastic and fitted to screen.
CRT would be one which gives off a colour to which This graticule can be easily changed, to make
the eye has maximum response ; has a high bum different types of measurements and its
resistance, to prevent accidental damage ; has a high position can be adjusted to align it with the
luminance and writing speed; and has short trace on CRT. External graticules suffer from
persistence to avoid multiple images when recording parallax errors as they are not in the plane of
fast displays. A typical choice of phosphor is P31. the phosphor.
Cathode Ray Oscilloscope (CRO) 649

(n) Internal graticule. This is deposited on the horizontal axis. The tick marks on the centre graticule
internal surface of the CRT face plate, and is represent minor divisions or subdivisions.
therefore on the same surface as the
phosphor. Although there is now no
parallax, an internal graticule cannot be
changed, and also needs some method of
electrical trace alignment. The graticule is
also difficult to illuminate for photography,
unless special illumination is provided in the
oscilloscope.
(in) Porjected graticule. It is available with some
cameras and provides flexibility which can
include additional features such as legends
on the graticule.
The face plate is sometimes tinted neutral-grey to
reduce ambient light interference, or external filters
can be used to reduce glare. Black wire mesh filters
increase the contrast ratio and restrict the viewing
angle, so reflections from oblique light sources are
minimised. Polarised filters can also be used in high 21.11 AQUADAG
glare applications. Light filters are sometimes utilised The bombarding electrons, striking the screen,
to affect selective colour components of the phosphor. release secondary emission electrons. These secondary
For example, an amber filter on a P7 phosphor would electrons are collected by an aqueous solution of
eliminate the short persistence blue component, and graphite called 'Aquadag' which is connected to the
increase the longer persistence yellow component ; second anode, collection of secondary electrons is
and a blue filter would do the reverse. necessary to keep the CRT screen in a state of electrical
The graticule may be etched or silk-screened on a equilibrium.
plastic CRT face plate or it may be chemically
21.12 COLOUR CRT DISPLAYS
deposited on the face of CRT along with phosphor. If
the graticule is put on the plastic face plate, it should Although colour CRTs, such as shadow mask
be positioned on the inside surface, that is, the surface tubes, are common in television and computer
that will be in contact with the face of CRT. This places displays, they are not used for oscilloscopes. One of
the display produced by the electron beam and the the reasons is that they are relatively expensive, and
graticule on the same plane, thereby eliminating the colour does not give many advantages when viewing
errors on account of parallax. Parallax errors occur single or multiple traces which are separated from
when the trace and the graticule are in different planes each other. However, limited colour can have
and the observer's eye is shifted from the direct line of advantages, such as in medical oscilloscopes, and an
sight as shown in Fig. 21.8. instrument is available for this application. It uses a
standard CRT, and a liquid crystal shutter to produce
a three colour display, as shown in Fig. 21.10.

Faceplate with
graticule
Viewer
Fig. 21.8 Illustration of parallax error.

The graticules are generally laid out in an 8 x 10


pattern as shown in Fig. 21.9 with 8 major divisions Fig. 21.10 Exploded view of a liquid crystal switch
along the vertical axis and 10 major divisions along the used with a CRT.
650 Electrical and Electronic Measurements and Instrumentation

The CRT has a multi-component phosphor, which


gives peak outputs of oranges and cyan light. Two
colour polarisers are used alter the CRT and these
polarise the orange light vertically, the cyan light
horizontally, and absorb white light. The liquid crystal
switch will not affect the light passing through it when
on, but it will rotate the light through it by 90° when
off. The linear polariser will pass horizontally
Fig. 21.11 Typical sawtooth waveform applied to
polarised light only. the horizontal deflection plates.
The liquid crystal switch is synchronised to the
CRT trace. In operation the CRT first sweeps out the During the sweep time, T, the beam moves from
cyan colour display field with the liquid crystal left to right across the CRT screen. The beam is
switched on. Only cyan light will be displayed since deflected to the right by the increasing amplitude of
this will be passed by the linear polariser. The CRT the ramp voltage and the fact that the positive voltage
then sweeps out the orange coloured portion of the attracts the negative electrons. During the retrace
display with the liquid crystal switched off. Now both time, or flyback time. T, the beam returns quickly to
the cyan and orange lights from the colour polarisers the left side of the screen. The control gird is generally
are rotated, so that the orange is horizontally polarised "gated off", which blanks out the beam during retrace
and is passed by the linear polariser. If any part of the time and prevents an undesirable retrace pattern from
display is to be yellow then it is traced by the CRT, appearing on the screen.
when the liquid crystal switch is both on and off, so Since signals of many different frequencies will
the cyan and orange colours combine to give yellow. be observed with the oscilloscope, the sweep rate must
The liquid crystal shutter oscilloscope has a good be adjustable. We can change the sweep rate in steps
by switching different capacitors into the circuit. The
contrast ratio since the shutter attenuates the non
front panel control for this adjustment is marked
lighted areas giving a black background. However the
time/div or sec/div. The sweep rate can be adjusted in
brightness is low since the light is emitted 50% of the
minor ways by making the resistor, R, in Fig. 21.12(c) a
time for any one colour, due to the duty cycle, and the
variable resistor.
polarisers also attenuate about half the light going
The circuit shown in Fig. 21.12(c) is a simple
through them. Overall, the brightness is less than 20%
sweep circuit in which the capacitor C charges
of that of a CRT without a liquid crystal shutter. The
most serious problem with the liquid crystal shutter
system is that it is limited in switching speed. By
making the liquid crystal switch narrow, and the
rotational distance of the molecules small, the
switching time can be reduced to about 0.5 ms. Even
then the bandwidth of the oscilloscope is limited to
under 100 kHz.

21.13 TIME BASE GENERATORS


Oscilloscopes are generally used to display a
waveform that varies as a function of time. If the
waveform is to be accurately reproduced, the beam
must have a constant horizontal velocity. Since the
beam velocity is a function of the deflecting voltage,
the deflecting voltage must increase linearly with
time. A voltage with this characteristic is called a ramp
voltage. If the voltage decreases rapidly to zero with
the waveform repeatedly reproduced, as shown in
Fig. 21.11, the pattern is generally called a sawtooth Fig. 21.12 Simple sawtooth generator and
waveform. associated waveforms.
Cathode Ray Oscilloscope (CRO) 65 I

through the resistor R. The capacitor discharges where


periodically through the transistor, Qv which causes V° = voltage across capacitor C at time t during
the waveform shown in Fig. 21.12(b) to appear across retrace ; V,
the capacitor. The signal, V{, which must be applied to
Vo = amplitude of the voltage across capacitor C
the base of the transistor to turn it "on" for short time at the start of discharge ; V,
intervals is also shown in Fig. 21.12(b). When the
C = value of capacitor C; F,
transistor is turned completely "on," it presents a
low-resistance discharge path through which the = time of interest after start of retrace ; s,
capacitor discharges quickly. R1 = resistance from collector to emitter of the
If the transistor is not turned "on" the capacitor saturated transistor; Q.
will charge exponentially to the supply voltage Vcc Example 21.2 An input pulse, Vir of 5 ns duration is
according to the equation, applied to the circuit in Fig 21.13 at the instant V, reaches
V0 = VcJl-exp(-f/RC)] ...(21.21) 4.76 V. What is the voltage across the capacitor after 50 ps if
the saturated transistor presents a resistance of 0.2 k£l to the
where v0 = instantaneous voltage across the circuit ?
capacitor at time t; V,
Solution. Using Eqn. 21.22, we find the voltage
Vcc = supply voltage ; V, as :
t = time of interest; s, V' = V„e-W
R = value of series resistor ; Q, = 4.76exp[-50 x IO-6 / (0.2 x 103 x 0.2 x IO-6)]
C = value of capacitor ; F. = 1.36 V
Example 21.1 A trigger pulse is applied to the sweep The relationship between the current charging a
generator in Fig. 21.13 every 10 ms. Compute the amplitude capacitor and the voltage across the capacitor is given
of the voltage, V, across the capacitor when the trigger as
pulse is applied. V0=^icdt ...(21.23)
c0

If the current is constant, the solution of 21.23 is,


V0 = It/C ...(21.24)
Substituting Vcc / R for I, we have,

Vo=~t
KC
-(21.25)

Equation 21.15 is the equation of a straight line


with a slope of VJRC. The voltage Vo would be a
Fig. 21.13 Circuit for Example 21.1 "linear ramp" if it followed this equation. Figure 21.14
Solution. Using Eq. 21.21 we compute the voltage compares the graph of the exponential function of
as Eqn. 21.21 to the graph of the linear function of
Eqn. 21.25.
Vo = Vcc[l-exp(-f/RC)]
= 50[l-exp(10x 10-3)/(500x103x0.2x10“6)]

= 50[l-exp(-0.1)]
= 50(1 -0.905)= 4.76 V.
Since Vo is less than 10% of Vcc, the charge curve
should still be quite linear at this point.
When a trigger pulse 17. is applied to the transistor
Qj in Fig. 21.12 (a), the capacitor C discharges through
the resistance presented by the "on" transistor
according to the equation Fig. 21.14 Graph comparing a capacitor charged by
a constant-current source to one charged
v; = vo[exp(-t,/KjC)] ...(21.22) by a constant-voltage source.
652 Electrical and Electronic Measurements and Instrumentation

As can be seen by comparing the linear and of the transistor. The collector-to-base voltage VCB can
exponential curves, the exponential function is fairly change considerably without any appreciable effect on
linear during the first 10% or 15% of the graph. Some the collector current.
inexpensive oscilloscopes use this type RC, of circuit In order to have a repetitive sweep voltage, the
as a sweep generator with reasonably satisfactory capacitor must be discharged after it has reached its
results. However, linearity can be improved by using maximum voltage. This can be done by connecting
one of several other types of circuits as a sweep transistor Q2 across the capacitor as shown. At specific
generator. The circuits most commonly used are :
time intervals, a voltage pulse applied to the base of Q2
1. Some form of constant-current generator to turns the transistor "on" providing a discharge path
charge a capacitor at a constant rate. for the capacitor.
2. A bootstrap circuit in which the current is
kept constant by maintaining a constant 21.14 OSCILLOSCOPE AMPLIFIERS
voltage across a charging resistor. The purpose of an oscilloscope is to provide a
3. An operational amplifier in a circuit known faithful representation applied to its input terminals,
by one of the following names: integrating considerable attention has to be paid to the designing
amplifier, Miller integrator, Miller sweep of these amplifiers for faithful representation of input
generator. signals. The oscillographic amplifiers can be classified
The circuit shown in Fig. 21.15(a) utilizes an into two major categories
ordinary bipolar junction transistor, Qr connected in (?) a.c. coupled amplifiers, and
the common-base configuration to charge capacitor C
(zz) d.c. coupled amplifiers.
at a constant rate. If the resistor, R has a large value of
Low cost oscilloscopes use a.c. coupled amplifiers
resistance, the emitter current will be nearly constant
A.C. amplifiers are commonly used in oscilloscopes
as will the collector current. Since the collector current
used for laboratory purposes.
charges the capacitor C, it will charge at a nearly
constant rate. Typical common-base output characteri­ The d.c. coupled amplifiers are expensive, they
stic curves are shown in Fig. 21.15(b). Note that the offer the advantage of responding to d.c. voltages.
curves are very nearly horizontal in the active region Their use makes it possible to measure d.c. voltages,
pure a.c. signals, and to measure a.c. signals super­
imposed upon d.c. signals. D.c. coupled amplifiers
have the additional advantage, that they eliminate the
problem of low frequency phase shift and accom­
panying waveform problem of waveform distortion
when observing low frequency pulse train.
There is another way of classifying oscilloscopic
amplifiers. They can be classified according to band­
width they use. On this basis they can be classified as :
(z) narrow bandwidth amplifiers, and
(zz) broad bandwidth amplifiers.
(a) Circuit to generate a ramp voltage by These categories of amplifiers are not sharply
charging a capacitor at a constant rate.
divided, nor is the classification universally accepted.
If the frequency band does not extend upto the
television colour sub-carrier frequency of 3.58 MHz,
the amplifier can be classified as narrow band.
However, if the frequency response curve of the
amplifier is flat beyond 3.58 MHz, the amplifier is
classified as a broad band amplifier. Most general
purpose, laboratory quality oscilloscopes respond
(b) The characteristic curves fully to frequencies in excess of 5 MHz. Therefore, the
for transistor Q}. responsible amplifier, which is the vertical amplifier,
is broadband.
Fig. 21.15
Cathode Ray Oscilloscope (CRO) 653

21.14.1 Vertical Amplifier A typical curve for a broadband amplifier of gain


The vertical amplifier is the principal factor in versus frequency, called a frequency-response curve,
determining the sensitivity and bandwidth of an is shown in Fig. 21.16.
oscilloscope. In general, greater sensitivity, which is The bandwidth of an oscilloscope is the range of
expressed in terms of V/cm of vertical deflection at the frequencies over which the gain of the vertical
midband frequency, is obtained at the expense of amplifier is within 3 db of the midband frequency
bandwidth. As a rule of thumb, an amplifier at some gain. The upper limit of the bandwidth is the
specific cost may be obtained with a certain gain­ frequency, f2, at which the gain has decreased by 3 db.
bandwidth product, which is the product of the The bandwidth can be increased by using feedback in
voltage gain of the amplifier and its bandwidth. designing the amplifier. However, this reduces the
Voltage gain may be sacrificed in favour of greater gain, which is why the gain-bandwidth product for an
bandwidth, or vice versa, without significantly amplifier is constant.
affecting the cost of the amplifier. However, if the
gain-bandwidth product increases, the cost of the
amplifier will increase.
The gain of the vertical amplifier determines the
smallest signal that the oscilloscope can satisfactorily
reproduce on the CRT screen. The sensitivity of an
oscilloscope is directly proportional to gain of the
vertical amplifier that is, as gain increases sensitivity Fig. 21.16 Typical frequency response curve for a
increases, which allows us to observe smaller- broadband amplifier.
amplitude signals.
One of the specifications generally included on
The vertical sensitivity is a measure of how much the specifications sheet for the vertical amplifier of a
the electron beam will be deflected for a specified broadband oscilloscope is the rise time. The rise time of
input signal. The CRT screen is covered with a plastic a pulse is defined as the time required for the edge to
grid pattern called a graticule. The spacing between rise from 10% to 90% of its maximum amplitude.
the grid lines is generally 10 mm. However, vertical When an oscilloscope is used to observe a pulse or a
sensitivity is generally expressed in volt per division. square wave, the rise time of the instrument must be
On the front panel of the oscilloscope shown in faster than the rise time of the pulse or square wave,
Fig. 21.28, one can see a knob attached to a rotary otherwise the observed signal will not be accurately
switch labeled gain. The gain could be designated reproduced. Although the exact relationship between
V/Div. The rotary switch is electrically connected to bandwidth and the rise time varies slightly with ampli­
the input attenuator network, which will be discussed fier design, an approximate relationship is given as
in subsequent paragraphs. The setting of the rotary tr x BW = 0.35 ...(21.26)
switch indicates what amplitude signal is required to where
deflect the beam vertically by one division. t = rise time measured in second,
The vertical sensitivity of an oscilloscope is the BW = bandwidth in hertz.
smallest deflection factor that can be selected with the
rotary switch. As an example, if the most sensitive Example 21.3 If the bandwidth of an oscilloscope is
position on the volt/division rotary switch is given as direct current to 10 MHz, what is the fastest rise
5 mV/division, then the vertical sensitivity of the time a sine wave can have to be accurately reproduced by the
oscilloscope is 5 mV/division. instrument?
Solution. Using Eq. 21.26, we can compute the
The bandwidth of an oscilloscope determines the
range of frequencies that can be accurately maximum rise time as,
035 QC
reproduced on the CRT screen. The greater the t =------------- 7- = 35 ns.
bandwidth, the wider is the range of frequencies that r 10xl06
can be observed with the instrument. Ideally, the gain The vertical amplifier of a laboratory-quality
of the broadband amplifier should be constant from oscilloscope frequently consists of the two major
direct current to near the upper limit of the range of circuit blocks shown in Fig. 21.17, a pre-amplifier and
frequencies that can be observed with the oscilloscope. the main vertical amplifier or mainframe amplifier.
654 Electrical and Electronic Measurements and Instrumentation

The pre-amplifier is sometimes a separate, inter­ the figure reduces, by voltage division, the amplitude
changeable plug-in unit that can quickly and easily be of the horizontal input signal to a level equal to the
plugged into the mainframe of the oscilloscope. If sensitivity of the horizontal amplifier.
different plug-in units are available for specific measure­
ment applications, the measurement capabilities of an
oscilloscope can be broadened considerably at a
reasonable cost.

Fig. 21.17 Block diagram showing the vertical ampli­


fier of a laboratory-quality oscilloscope.

The active element in the first stage of the


pre-amplifier is generally a field-effect transistor (FET)
which provides a high input impedance for the Fig. 21.18 Block diagram of the horizontal amplifier of
an oscilloscope.
oscilloscope. The final stage of the mainframe
amplifier is generally a push-pull amplifier which
21.15 VERTICAL INPUT AND SWEEP
provides voltages of equal amplitude but out of phase
GENERATOR SIGNAL SYNCHRONIZATION
by 180° to achieve balanced deflection.
Most waveforms that we will have occasion to
21.14.2 Horizontal Amplifiers observe with an oscilloscope will be changing at a rate
The horizontal amplifier basically serves two much faster than the eye can follow, perhaps many
purposes. million times per second. If we are to be able to
observe such rapid changes, the beam must retrace the
1. When the oscilloscope is being used in the
same pattern repeatedly. If the pattern is retraced in
ordinary mode of operation to display a
signal applied to the vertical input, the
such a manner that the pattern always occupies the
horizontal amplifier will amplify the sweep same location on the screen, the eye will see a
generator output. stationary display. The beam will retrace the same
pattern at a rapid rate if the vertical input signal and
2. When the oscilloscope is being used in the X-Y
the sweep generator signal are synchronized ; which
mode, the signal applied to the horizontal
means that the frequency of vertical input signal must
input terminal will be amplified by the
be equal to or an exact multiple of, the sweep
horizontal amplifier.
generator signal frequency, as shown in Fig. 21.19. If
When the oscilloscope is being used in its
ordinary mode of operation, the gain and bandwidth Vertical
requirements for the horizontal amplifier are not as input
signal
stringent as those for the vertical amplifier. Although
the vertical amplifier must be able to reproduce
faithfully low-amplitude, high-frequency signals with Sweep
generator
fast rise times, the horizontal amplifier is required to output
provide only a faithful reproduction of the sweep
signal, which has a relatively high amplitude and a
slow rise time.
As with the vertical amplifier, the final stage of
the horizontal amplifier is a push-pull amplifier, as
shown in Fig. 21.18. The attenuator network given in Fig. 21.19 Synchronized waveforms and CRT display.
Cathode Ray Oscilloscope (CRO) 655

the vertical input frequency is not exactly equal to or may receive an input from one of three sources
an exact multiple of, the sawtooth frequency, the depending on the setting of the trigger selector switch.
waveforms will not be synchronized and the display The input signal may come from an external source
"walks" across the screen. If the pattern moves toward when the trigger selector switch is set to EXT, from a
the right, the frequency of the sawtooth waveform is low-amplitude ac voltage at line frequency when the
too high. Movement of the pattern towards the left switch is set to LINE, or from the vertical amplifier
indicates that the frequency of the sawtooth is too low. when the switch is set to INT. When set for internal
Synchronization of the waveforms can be triggering (INT), the trigger circuit receives its input
accomplished in two different ways. from the vertical amplifier. When the vertical input
(z) Free running sweep. In very basic signal that is being amplified by the vertical amplifier
oscilloscopes the sweep generator is continuously reaches a certain level, the trigger circuit provides a
changing and discharging a capacitor. One ramp pulse to The sweep generator, thereby ensuring that
voltage is followed immediately by another; hence, the sweep generator output is synchronized with the
the sawtooth pattern appears. A sweep generator signal that triggers it.
operating in this manner is said to be “free running". In One type of circuit frequently used in the "trigger
order to present a stationary display on the screen, the circuit" block of Fig. 21.20 is called a Schmitt trigger or
sweep generator signal must be forced to run in a voltage-level detector. Basically, the Schmitt trigger
synchronization with the vertical input signal. In basic compares an input voltage, in this case from the
oscilloscopes this is accomplished by carefully vertical amplifier, with a voltage at a point in the
adjusting the sweep frequency to a value very close to circuit. When the input voltage exceeds the voltage to
the exact frequency of the vertical input signal, or a which it is being compared, the circuit changes states,
sub-multiple of this frequency. With both signals at which means the output voltage goes to a high state.
the same frequency, an internal synchronizing pulse The point at which this occurs is called the upper
will lock the sweep generator into the vertical input
trigger point (UTP). When the input voltage drops
signal. This method of synchronization has some
below a certain level called the lower trigger paint
serious limitations when an attempt is made to
(LTP), the Schmitt trigger output returns to its original
observe low-amplitude signals. However, the most
level, and thus the output of the circuit is a square
serious limitation is probably the inability of the
wave. Unless this square wave is of very short
instrument to maintain synchronization when the
duration, it will not be suitable to trigger the sweep
amplitude or frequency of the vertical signal is not
generator directly. It is common practice to apply the
constant, such as with voice or music signals.
square wave to an RC circuit called a differentiator
(zz) Triggered sweep. The above limitations are whose output is a short-duration spike suitable for
overcome by incorporating a trigger circuit into the triggering the sweep generator.
oscilloscope as shown in Fig. 21.20. The trigger circuit

Vertical
input

Ext.
trigger

Fifl. 21.20 Block diagram of an oscilloscope with triggered sweep.


656 Electrical and Electronic Measurements and Instrumentation

21.16 ATTENUATORS Example 21.4 If the switch in Fig. 21.21 is set of


The voltage at the input terminal of the vertical position D, what is the attenuation factor ?
amplifier, which causes the beam to be deflected off Solution. From Eqn. 21.21, we obtain the voltage
the CRT screen, is quite low in amplitude. So that divider ratio as :
high-amplitude signals may be displayed, an attenuator Yl-Yl
network is placed between the vertical input terminal Y
and the input terminal of the vertical amplifier. The 9x103+900 + 90 + 10 _01
term attenuate means to "reduce in size". The purpose of 100 xlO3
the attenuator is to reduce the amplitude of the
Therefore, the attenuation factor = 1/0.1 = 10.
vertical input signal before applying it to the vertical
amplifier. The most basic atenuator is a simple The resistance values shown in Fig. 21.21 will
resistive voltage divider such as the one shown in provide the desired attenuation only if the input
Fig. 21.21. With this circuit, the input voltage V will be resistance R. of the vertical amplifier is much greater
attenuated by a factor of 10,000 with the switch S set to than the attenuator resistance values. If the amplifier
input resistance is not much larger than the attenuator
resistance with which it is in parallel, appreciable
error in the attenuation factor will be noted since the
attenuator is “loaded down" by the amplifier.
Example 21.5 If the switch in Fig. 21.21 is in position
D, as in Example 21.4, and the input resistance of the
amplifier Ri is 100 k£l, what is the attenuation factor ?
Solution. Since the amplifier input resistance is in
parallel with the attenuator resistance, this parallel
combination must be used in the voltage divider
equation. The parallel resistance is
RR
R =—'—
P Ri + R
100xl03xl0xl03 onQ1„
=---------- ------------- = 9.09 kQ
100xl03 + 10xl03
Therefore,
Fig. 21.21 Resistive attenuator network. V 9 09
-2- = = 0.0909
v. 100
position A as shown. In positions B, C and D, the
attenuation factors will be 1000, 100 and 10, respec­ The attenuation factor is 1/0.0909 =11.
tively. In switch position E there is no attenuation. In As can be seen by comparing the results of
any switch position, the total input resistance, as seen Examples 21.4 and 21.5, the attenuation is increased in
by V-, is 100 kQ. The attenuation in any switch response to the parallel effect of the input resistance of
position can be determined from the ratio of the total the amplifier.
resistance from the position of interest to ground to Switch S in Fig. 21.21 is the rotary switch that is
the total series resistance, written as : mounted on the front panel of an oscilloscope and
V R labeled volt/div. as was mentioned briefly in
-^ = — ...(21.22)
17. Rt Section 21.14.1. Example 21.6, shows how to
determine the value of the attenuating resistors to
where obtain desired attenuation factors.
R = total resistance from the desired attenuator
Example, 21,6 An oscilloscope is to have an input
terminal to ground, and
resistance of 8 MQ, a sensitivity of 50 mV, and attenuation
Rt = total series resistance. factors of 4, 10, 40, 100 and 400. Compute the value of the
Equation 21.22 is generally referred to as the attenuating resistors and the volt/division value
voltage divider equation. The attenuation factor is the corresponding to each attenuation factor. Assume R{ of the
reciprocal of the voltage divider ratio. vertical amplifier is sufficiently high to be ignored.
Cathode Ray Oscilloscope (CRO) 657

RA + RB^Rc = ^V0/yi)
= (8 MQ) (50 mV/2 V) = 200 kQ
Rc=(Ra + Rb + Rc)-(Ra + Rb)
= 200 kQ - 80 kQ = 120 kQ
ra + rb + + rd =
= (8 MQ) (50 mV/0.5 V)
= 800 kQ
RD=(RA + RB + RC + Rd)~(RA +RB + RC>

= 800 kQ-200 kQ = 600 kQ


ra + Rb + + rd + re = Vo SVi
= (8MQ)(50mV/0.2V)
= 2MQ
RE = (RA + RB + + RD + re^
- (Ra + Rb + + Rd)
Fig. 21.22 Circuit for Example 21.6. = 2MQ -800kQ =1.2 MQ

Solution. The attenuating network required is Rp = ~(RA + RB + + RD + RE^

shown in Fig. 21.22. When switch S is set to position A, = 8MQ-2MQ =6 MQ.


as indicated, maximum voltage can be applied across
The complete attenuator network is shown in
the network because of the voltage divider action.
Fig- 21-23,
Therefore, at this position the attenuation factor will
be 400. The attenuation factor equals the reciprocal of
the voltage divider ratio. For position A then
1 V-
attenuation factor = 400 =-------- or 400 = —
Vo/Vi Vo

Since Vo equals the value of the sensitivity


(50 mV/div) because Vg is applied directly to the
amplifier in each switch position, we can solve for V) as:
V; = 4001/
= 400 (50 mV/div) =20 V/div at position A
From this result we see that we can multiply the
attenuation factor by the sensitivity to obtain the value
of the volts/division at each switch setting.
Position B: V. = 100 (50 mV/div) = 5 V/div.
Position C : V* . = 40 (50 mV/div) = 2 V/div.
Position D : V = 10 (50 mV/div) = 0.5 V/div.
Position E : V = 4 (50 mV/div) = 0.2 V/div.
Position F: There is no attenuation or the
attenuation factor is 1 ; therefore,
Fig. 21.23 Complete attenuator network
V. = 1 (50 mV/div) = 50 mV/div.
for Example 21.6
Solving for the attenuating resistors, we obtain
the following : If we were measuring de voltages exclusively, our
discussion of voltage divider attenuators might end
Ra = Rr(V0/ V) = (8MQ) (30 mV/20 V) = 20 kQ
here. However, oscilloscopes are used primarily to
Ra + Rb = 1^(1/ IV;.) = (8 MQ) (50 mV/5 V) = 80 kQ display ac waveforms. This introduces the problem of
RDr=(R
' /ia + RD
r)-R
' ria = 80 kQ-20 kQ =60 kQ the stray capacitance of the attenuator network as
658 Electrical and Electronic Measurements and Instrumentation

shown in Fig. 21.24. For a.c. signals, the attenuation Therefore, the attenuation with high frequency
factor is actually related to the ratio of the impedance a.c. is very much different from what it is with d.c.
Consider the attenuator network of Fig. 21.26. The
resistance ratio of to Kj is the reciprocal of the ratio

Fig. 21.26 Compensated attenuator circuit.


n—i—niiiiwirm r '-
Fig. 21.24 Attenuator network with associated of capacitors C2 to Cr When this is true, the voltage
stray capacitance. divider ratio will remain constant over a wide range of
frequencies. Moreover, for a frequency-compensated
If the stray capacitance values are not correctly
voltage divider, the following relationships apply :
related to the resistance values, the voltage divider
R, = i ...(21.24)
ratio will not have the same value at all frequencies.
Example 21.'/ Compare the output voltage of the voltage
divider attenuator shown in Fig. 21.25 for a d.c. voltage and and C2 = C1(fc-l) ...(21.25)
a 10 MHz a.c. signal. where k is the voltage divider ratio, Vi/V0.
cl
with these values,' X . = 1592Q and XcZ„ =177.
< 177 >
V=(l)---- —----- =0.1 V
0 U592+177J
which is the same as with d.c.

21.17 BASIC CRO CIRCUITS


Figure 21.27 shows a block diagram of a typical
oscilloscope. In the following few pages, we describe
the basic circuitry of cathode my oscilloscope.
1. Vertical (Y) deflection system. The signals to
Fig. 21.25 Circuit for Example 21.7. be examined are usually applied to the vertical or Y
deflection plates through an input attenuator and a
Solution. For the d.c. voltage source
number of amplifier stages. Vertical amplifier is
100 XlO3 required because the signals are not strong enough to
900 x 103 + 100x 103 produce measurable deflection on the CRT screen. The
amplifier response must be wide enough to pass
= 0.1 V or k = 1/0.1 = 10
faithfully the entire band of frequencies to be
For the a.c. source, Xfl is approximately 3183 Q and measured.
Xc2 is approximately 1592 Q. Since these values are When high voltage signals are to be examined,
much less than the value of the resistances in parallel they must be attenuated to bring them within the
with them, we can approximate the output voltage as: range of vertical amplifiers. The vertical amplifier
/ 1592 output is also applied to the synchronizing amplifier
K=(l) through the synchronizer selector switch in the
I <3183 + 1592
internal position. This permits the horizontal sweep
= 0.33 York = 1/0.33-3. circuit to be triggered by the signal being investigated.
Cathode Ray Oscilloscope (CRO) 659
Vertical
Intensity (Z) position
input
Vertical (Y) ■< • Focusing
direct Alternate Z input / anode
Vertical
Vertical (Y)
amplifier CRT
input Cathode
Horizontal (X) Heater
direct

External pxt
horizontal(X)
input Horizontal
Sweep amplifier Alternate Accelerating
3 selector blanking
input Horizontal
Int. r position
Ext. 6 Sync
External
sync(trigger) Line ^Sync J amplifier
input selector
Intensity Focus Astigmatism
Sawtooth
sweep d.c. supply
generator d
To vertical To line
input frequency
Calibration Blanking
oscillator circuit

Fig. 21.27 Simplified block diagram of a general purpose oscilloscope.

2. Horizontal (X) deflection system. The initiated by the waveform under examination. In some
horizontal (X) deflection plates are fed by a sweep cases the waveform may be periodic, but it may be
voltage that provides a time base. The horizontal that the interesting part of the waveform is of a very
plates are supplied through an amplifier, but they can short duration compared to the period of the
be fed directly when voltages are of sufficient waveform. Under such cases a triggered sweep is
magnitude. When external signals are to be applied to used.
the horizontal deflection system, they can also be fed In triggered sweep or single sweep, the spot is
through the horizontal amplifier, via the sweep swept once across the screen in response to a trigger
selector switch in the external position. When the signal. The triggered sweep is used for examination of
sweep selector switch is in the internal position, the transients or one time signals and the waveform is
horizontal amplifier receives an input from the saw photographed for record. The trigger can be obtained
tooth sweep generator which is triggered by the from the signal under investigation or by an external
synchronizing amplifier. source.

Types of sweeps. There are four basic types of (zzz) Driven sweep. In most cases, a driven sweep is
used where the sweep is recurrent but triggered by the
sweeps :
signal under test.
(z) Free running or recurrent sweep. In the free
(zzz) Non saw tooth sweep. For some applications like
running or recurrent sweep, the sawthooth waveform
comparison of two frequencies or for finding phase
is repetitive. A new sweep is started immediately after
shift between two voltages, non sawtooth sweep
the previous sweep is terminated and the circuit is not
voltages are utilized for the sweep circuit. Sweep
initiated by any external signal.
frequencies vary with the type of oscilloscope. A
(zz) Triggered sweep. A waveform to be observed on laboratory oscilloscope may have sweep frequencies
the CRO may not be periodic but may perhaps occur up to several MHz ; a simple oscilloscope for audio
at irregular intervals. In this case it is desirable that the work has an upper limit of 100 kHz. Most TV services
sweep circuit remain inoperative and the sweep be require a sweep voltage frequency up to 1 MHz.
Electrical and Electronic Measurements and Instrumentation

3. Synchronization. Whatever type of sweep is 4. Blanking circuit. The sawtooth sweep voltage
used, it must be synchronization with the signal being applied to the X plates moves the beam across the CRT
measured. Synchronization has to be done to obtain a tube in a straight horizontal line from left to right
pattern. This requires that the time base be operated at during the sweep or trace time T. At comparatively
a submultiple frequency of the signal under slow movement of the spot will appear as a solid line,
measurement (applied to Y plates). If synchronization provided the rate of movement exceeds the threshold
is not done, the pattern is not stationary, but appears of persistence of vision. Below this threshold limit, a
to drift across the screen in a random fashion. moving spot is perceived. On the other hand, the
Sources of synchronization. There are three usual comparatively rapid movement of spot will appear as
sources for synchronization which can be selected by a thin and dim line, or may be invisible. Thus if the
synchronization selector : retrace or flyback time is very small, the spot remains
invisible. In an ideal case the flyback time, Tr is zero
(z) Internal. In this type of synchronization, the
and hence the spot while moving from right to left
trigger is obtained from the signal being
remains invisible. However in actual practice the
measured through the vertical amplifier.
flyback time is not zero and therefore the retrace
(zz) External. In this method, an external trigger
(moving of beam from right to left i.e., its starting
source is also used to trigger or initiate the
point) may cause confusion. Thus the retrace should
signal being measured
be eliminated or blanked out. The retrace is blanked
(zzz) Line. In this case, the trigger is obtained from out by applying a high negative voltage to the grid
the power supply to the CRO (say 230 V, 50 Hz). during the flyback period Tr. The blanking voltage is
usually developed (or triggered) by sweep generator.
5. Intensity (Z-axis) modulation. Intensity
modulation (Z-axis modulation) is done by inserting a
signal between the ground and the cathode (or control
grid). Z-axis modulation is applied during normally
visible portion of the trace.
The Z-axis modulation can be used for
brightening the display. Periodic positive pulses are
applied to the grid (alternatively negative pulses are
applied to cathode) to brighten the beam during its
sweep period. These periodically brightened spots
may be used as markers for time calibration of the
main waveform.
6. Positioning controls. It is necessary to
provide some means of positioning the trace on the
screen. The positioning of the trace is done by
applying small independent, internal d.c. voltages to
the deflecting plates and control can be exercised by
varying the voltage with help of potentiometers.
7. FOCUS control. As mentioned earlier, the
focusing electrode acts like a lens whose focal length
can be changed. This change can be brought about by
changing the potential of the focusing anode.
8. Intensity control. The intensity of the beam is
varied by the Intensity control potentiometer which
changes the grid potential with respect to cathode. The
grid potential determines the amount of electrons
leaving the cathode and thus controls the intensity of
the beam.
9. Calibration circuit. Laboratory oscilloscopes
normally have an internally generated and stabilized
fig.21.28 Front panel of a general purpose CRO.
Cathode Ray Oscilloscope (CRO) 661

voltage of known amplitude which is used for magnitude and polarity of the input voltage.
calibration purposes. Usually the calibrating voltage Figure 21.29 shows the waveform displayed on a CRT
has a square waveform. tube due to an input sinusoidal voltage.
10. Astigmatism. In most modem oscilloscopes At the end of one sweep cycle, the sweep voltage
there is an additional focusing control marked abruptly drops down and the spot is immediately
Astigmatism. This is used to correct an effect which transferred to its original position. The process is then
exactly is analogous to astigmatism in optical lenses. repeated again, with the result, that a stationary image
To focus the spot correctly, it is necessary to stop it is seen on the screen.
near the centre of the screen by switching off the time For the case shown the frequency of the input
base and adjusting the X and Y positioning controls.
voltage is twice that of sawtooth (sweep) voltage. To
The spot is then made as sharp as possible by
observe more than one cycle of the input voltage, the
successive adjustment of focus and astigmatism
sweep voltage frequency has to be a submultiple of the
controls.
input voltage frequency.
21.18 OBSERVATION OF WAVEFORM ON CRO
21.19 MEASUREMENT OF VOLTAGES & CURRENTS
In order to observe waveform on a CRO, the
The expression for electrostatic deflection, given
waveform of voltage under test is applied to Y plates
in Eqn. 21.16, shows that the deflection is proportional
and a voltage obtained from a sawtooth generator is
to the deflection-plate voltage. Thus the cathode-ray
applied to X plates. Let us assume that the sawtooth
tube will measure voltage. It is usual to calibrate the
waveform has an idealized waveshape.
tube under the given operating conditions by
When simultaneously with the horizontal
observing the deflection produced by a known
sawtooth (ramp) voltage, an input voltage is applied
voltage. Direct voltages may be obtained from the
to vertical deflection (Y) plates, the beam is under the
static deflection of the spot, alternating voltages from
influence of two forces : (z) one in the horizontal
the length of the line produced when the voltage is
direction moving the beam at a linear rate from left to
right, and (zz) second in the vertical direction moving applied to Y plates while no voltage is applied to X
the beam up and down. Since the deflection is plates. The length of this line corresponds to the
proportional to the voltage applied to the deflection peak-to-peak voltage. When dealing with sinusoidal
plates, the horizontal movement is proportional to the voltages, the rms value is given by dividing the
voltage applied to X plates at any instant and since peak-to-peak voltage by 2a/2 .
the ramp voltage is linear it traces a straight line on the Laboratory oscillographs frequently incorporate
CRT screen. The vertical deflection is proportional to voltage-measurement facilities by including constant­
the voltage applied to the Y plates at any instant and gain amplifiers and calibrated shift controls. The Y-shift
thus the beam moves up and down according to the control is adjusted so that positive peak of the test
voltage coincides with some datum line on the screen;
the shift control is then operated until the negative
peak coincides with the datum. The movement of the
control is arranged to read directly the peak-to-peak
voltage. The valuejof"a current can be obtained by
measuring the vdltage drop across a known resistance
connected in/me circuit.

21.t2p^MEASUREMENT OF PHASE & FREQUENCY


(LISSAJOUS PATTERNS)
It is interesting to consider the characteristics of
patterns that appear on the screen of a CRT when
sinusoidal voltages are simultaneously applied to
horizontal and vertical plates. These patterns are
called 'Lissajous Patterns'.
When two sinusoidal voltages of equal frequency
Fig* 21 29 Observation of waveform on CRO. which are in phase with each other are applied to the
662 Electrical and Electronic Measurements and Instrumentation

horizontal and vertical deflection plates, the pattern


appearing on the screen is a straight line as is clear
from Fig. 21.30.

Fig 21.32 Lissajous pattern with two equal voltages


of same frequency and phase shift of <|>.
(z) A straight line results when the two voltages
Fig. 21.30 Lissajous pattern with equal frequency are equal and are either in phase with each other or
voltages and zero phase shift.
180° out of phase with each other. The angle formed
Thus when two equal voltages of equal frequency with the horizontal is 45° when the magnitudes of
but with 90° phase displacement are applied to a CRO, voltages are equal. An increase in the vertical
the trace on the screen is a circle. This is shown in deflection voltage causes the line to have an angle
Fig. 21.31. greater than 45° with the horizontal. On the other
hand a greater horizontal voltages makes the angle
less than 45° with the horizontal.
(zz) Two sinusoidal waveforms of the same
frequency produce a Lissajous pattern, which may be
a straight line, a circle or an ellipse depending upon
the phase and magnitude of the voltages.
A circle can be formed only when the magnitude
of the two singals are equal and the phase difference
between them is either 90° or 270°. However, if the
two voltages are not equal and/or out of phase an
ellipse is formed. If the Y voltage is larger, an ellipse
with vertical major axis is formed while if the X plate
voltage has a greater magnitude, the major axis of the
ellipse lies along horizontal axis.
(zzz) It is clear from Fig. 21.33, that for equal
voltages of same frequency progressive variation of
phase voltage causes the pattern to vary from a straight
Fig. 21.31 Lissajous pattern with equal voltages of
diagonal line to ellipses of different eccentricities and
equal frequency and a phase shift of 90°.
then to a circle, after that through another series of
When two equal voltages of equal frequency but ellipses and finally a diagonal straight line again.
with a phase shift (j> (not equal to 0° or 90°) are applied Regardless of the two amplitudes of the applied
to a CRO we obtain an ellipse as shown in Fig. 21.32. voltages the ellipse provides a simple means of
An ellipse is also obtained when unequal voltages of finding phase difference between two voltages.
same frequency are applied to the CRO. Referring to Fig. 21.36, the sine of the phase angle
between the voltages is given by :
A number of conclusions can be drawn from the
above discussions. When two sinusoidal voltages of Y X
sin <}» = —= — ...(21.20)
same frequency are applied : Y, X~
Cathode Ray Oscilloscope (CRO) 663

Resulting Vertical deflection Frequency measurements. Lissajous patterns


may be used for accurate measurement of frequency.
The signal, whose frequency is to be measured, is
applied to the Y plates. An accurately calibrated
standard variable frequency source is used to supply
voltage to the X plates, with the internal sweep
generator switched off. The standard frequency is
adjusted unit the pattern appears as a circle or an
ellipse, indicating that both signals are of the same
frequency. Where it is not possible to adjust the
standard signal frequency to the exact frequency of
the unknown signal, the standard i$ adjusted to
multiple or a submutliple of the frequency of the un­
known source so that the pattern appears stationary.
Let us consider an example. Suppose sine waves
are applied to X and Y plates as shown in Fig. 21.35.
Let the frequency of wave applied to Y plates is twice
that of the voltage applied to X plates. This means that
the CRT spot travels two complete cycles in the
vertical direction against one in the horizontal
direction.

phase shifts.

For convenience, the gains of the vertical and


horizontal amplifiers are adjusted so that the ellipse
fits exactly into a square marked by the lines on the
graticule.
If the major axis of the ellipse lies in the first and
third quadrants (i.e., its slope is positive) as in
Fig. 21.34 (tz) the phase angle is either between 0° to 90°
or between 270° to 360°. When the major axis of ellipse
lies in second and fourth quadrants i.e. when its slope
is negative as in Fig. 21.34 (b), the phase angle is either
between 90° and 180° or between 180° and 270°.

Fig. 21.35 Lissajous pattern with frequency ratio 2:1.

The two waves start at the same instant. Lissajous


pattern may be constructed in the usual way and a 8
shaped pattern with two loops is obtained. If the two
waves do not start at the same instant we get different
patterns for the same frequency ratio. The Lissajous
patterns for other frequency ratios can be similarly
drawn. Some of the these patterns are shown in
Fig.21.34 Determination of angle of phase shift. Fig. 21.36.
664 Electrical and Electronic Measurements and Instrumentation

/y _ number of horizontal tangencies


/ number of vertical tangencies
2+1/2 5
1 ~2

Fig. 21.37 Lissajous patterns with half tangencies.

Fig. 21.36 Lissajous patterns with different There are some restrictions on the frequencies
frequency ratios. which can be applied to the deflection plates. One
It can be shown that for all the above cases, the obviously, is that the CRO must have the bandwidth
ratio of the two frequencies is : required for these frequencies. The other restriction is
fy _ number of times tangent touches top or bottom that the ratio of the two frequencies should not be such
as to make the pattern too complicated otherwise
fx number of times tangent touches either side
determination of frequency would become difficult.
_ number of horizontal tangencies As a rule ratios as high as 10 :1 and as low as 10:9 can
number of vertical tangencies be determined comfortably.
where f = frequency of signal applied to Y plates,
21.21 MULTI-INPUT OSCILLOSCOPES
and fx = frequency of signal applied to X plates
Oscilloscopes can have multiple input and
The above rule, however, does not hold for the display facilities. Two inputs is the most common,
Lissajous pattern with free ends as shown in although four and eight inputs are available for
Fig. 21.37(a). The simple rule mentioned above needs special applications. There are two primary types :
following modifications : single beam which can then be converted into several
Two lines are drawn, one horizontal and the other traces, and dual beam, which may also subsequently be
vertical so that they do not pass through any converted into a further number of traces. Two input
intersections of different parts of the Lissajous curve. oscilloscopes are described in this section, under the
The number of intersections of the horizontal and the headings of dual trace and dual beam, although the
vertical lines with the Lissajous curve are individually principles are applicable to any number of inputs.
counted. The frequency ratio is given by :
21.21.1 Dual Trace Oscilloscopes
number of intersections of the horizontal line
fy ' with the curve Figure 21.38 illustrates the construction of a
typical dual trace oscilloscope. There are two separate
fx number of intersections of the vertical line
with the curve vertical input channels, A and B, and these use
separate attenuator and preamplifier stages. Therefore
The applications of this rule to Fig. 21.37 gives a the amplitude of each input, as viewed on the oscillo­
. /v 5
frequency ratio — = —. scope, can be individually controlled. After preampli­
A fication the two channels meet at an electronic switch.
The modified rule is applicable in all cases This has the ability to pass one channel at a time into
whether the Lissajous pattern is open or closed. the vertical amplifier, via the delay line.
The ratio of frequencies when open ended There are two common operating modes for the
Lissajous patterns are obtained can also be found by electronic switch, called alternate and chop, and these
treating the open ends as half tangencies as shown in are selected from the instrument's front panel. The
Fig. 21.37(b)- The pattern of Fig. 21.37(a) is reproduced alternate mode is illustrated in Fig. 21.39. In this the
Cathode Ray Oscilloscope (CRO) 665

Fig. 21.38 A dual trace oscilloscope block diagram.

electronic switch alternates between channels A and B, The chopped operating mode of the electronic
letting each through for one cycle of the horizontal switch is shown in Fig. 21.40. In this mode the
sweep. The display is blanked during the flyback and electronic switch free runs at a high frequency of the
hold-off periods, as in a conventional oscilloscope. order of 100 kHz to 500 kHz. The result is that small
Provided the sweep speed is much greater than the segments from channels A and B are connected
decay time of the CRT phosphor, the screen will show alternately to the vertical amplifier, and displayed on
a stable display of both the waveform at channels A the screen. Provided the chopping rate is much faster
and B. The alternate mode cannot be used for than the horizontal sweep rate, the display will show a
continuous line for each channel. If the sweep rate
displaying very low frequency signals.
approaches the chopping rate then the individual
segments will be visible, and the alternate mode
should now be used.

Time

Time
Fig. 21.39 Waveforms for a dual channel oscilloscope
operating in alternate mode ; (a) horizontal Fig. 21.40 Waveforms of a dual channel oscilloscope
sweep voltage, (b) voltage to channel A. operating in chopped mode, (a) Horizontal
(c) voltage to channel B, sweep voltage (b) Voltage to channel A,
(d) grid control voltage. (c) Voltage to channel B,(d) grid control voltage.
666 Electrical and Electronic Measurements and Instrumentation

The time base circuit shown in Fig. 21.38 is similar between the two vertical deflection plates, It therefore
to that of a single input oscilloscope (Fig. 21.12). isolates the two channels. The split beam arrangement
Switch S2 allows the circuit to be triggered on either has half the brightness of a single beam, which has
the A or B channel waveforms, or on line frequency, or disadvantages at high frequency operation. An
on an external signal. The horizontal amplifier can be alternative method of splitting the beam, which
fed from the sweep generator, or the B channel via improves its brightness, is to have two apertures in the
switch Sr This is the X-Y mode and the oscilloscope last anode, instead of one, so that two beams emerge
operates from channel A as the vertical signal and from it.
channel B as the horizontal signal, giving very The disadvantage of the split beam construction is
accurate X-Y measurements. Several operating modes that the two display may have noticeably different
can be selected from the front panel for display, such brightness, if operated at widely spaced sweep
as channel A only, channel B only, channels A and B as speeds. The brightness and focus controls also affect
two traces, and signals A + B, A-Br B- A or -(A + B) as the two traces at the same time.
a single trace.
21.22 SAMPLING OSCILLOSCOPES
21.21.2 Dual Beam Oscilloscope
A sampling oscilloscope is used to examine very
The dual trace oscilloscope cannot capture two fast signals. It is similar in principle to the use of
fast transient events, as it cannot switch quickly stroboscopic light to look at fast mechanical motion.
enough between traces. The dual beam oscilloscope Samples are taken at different portions of the
has two separate electron beams, and therefore two waveform, over successive cycles, and then the total
completely separate vertical channels, as in Fig. 21.39. picture is stretched, amplified by relatively low
The two channels may have a common time base bandwidth amplifiers, and displayed as continuous
system, as in Fig. 21.38, or they may have independent wave on the screen. The advantage of a sampling
time base circuits, as in Fig. 21.41. An independent oscilloscope is that it can measure very high speed
time base allows different sweep rates for the two events, which require sweep speeds of the order of
channels but increases the size and weight of the 10 ps per division, and amplifier bandwidths of
oscilloscope. 15 GHz, using instruments having bandwidth several
Two methods are used for generating the two orders lower. The disadvantage of a sampling
electron beams within the CRT. The first method uses oscilloscope is that it can only make measurements on
a double gun tube. This allows the brightness and repetitive waveform signals, continuous display for
focus of each beam to be controlled separately but it is frequencies in the 50-300 MHz range depending upon
bulkier than a split beam tube. the design of the oscilloscope. Above this range of
In the second method, known as split beam, a frequencies Sampling Techniques must be employed
single electron gun is used. A horizontal splitter plate to obtain satisfactory display. The display may be
is placed between the last anode and the Y deflection made up from as many 1000 dots of luminescence. The
plates. This plate is held at the same potential as the vertical deflection for each dot is obtained from
anode, and it goes along the length of the tube, progressively later points in each successive cycle of

plate

Fig. 21.41 Dual beam oscilloscope with independent time bases.


Cathode Ray Oscilloscope (CRO) 667

the input waveform as shown in Fig. 21.42. The amplifier to the sampling diode gate. This ensures that
horizontal deflection of the electron beam is obtained the voltage on the capacitor store is only increased by
by application of a staircase waveform to X deflection the incremental value of the input voltage change,
between each sample.
The staircase is reset after a certain number of
steps, typically 100 to 1000, and it then starts again.
Therefore up to about 1000 points are used to create
the waveform on the screen.
The staircase waveform also feeds the horizontal
plates of the CRT, and it is used to move the spot
across the screen in a series of rapid movements.
Figure 21.42 shows the screen display.
Fig. 21.42 Principle of sampling oscilloscopes.
The sample frequency used in sampling
The sampling oscilloscope is able to respond and oscilloscopes can be as low as one hundredth of the
store rapid bits of information and present them in a signal frequency, so a signal frequency of 1 GHz needs
continuous display. It is this ability that enables the an amplifier bandwidth of only 10 MHz.
sampling oscilloscope to side-step the usual limita­ Delayed sweep. Many laboratory-quality
tions in conventional high frequency oscilloscopes oscilloscopes include a delayed-sweep feature. This
which have limited sensitivity and bandwidth and feature increases the versality of the instrument by
small display size. The sampling techniques imme­ making it possible to magnify a selected portion of the
diately transform the input signal into lower frequency, undelayed sweep, measure waveform jitter or rise
domain, where conventional low frequency circuitry time, and check pulse-time modulation, as well as
is then capable of producing a highly effective display. many other applications.
The sampling oscilloscopes can be used beyond Delayed sweep is a technique that adds a precise
50 MHz into the UHF range around 500 MHz and amount of time between the trigger point and the
beyond up to 10 GHz. However, it should be beginning of the scope sweep. When the scope is being
understood that sampling techniques cannot be used used in the delayed-sweep mode, the start of the
for the display of transient waveforms. delayed sweep can range from a few microseconds to
Figure 21.43 shows a block schematic for a typical perhaps 10 seconds or more. The delayed sweep
sampling oscilloscope. The input signal is delayed, operation allows the instrument user to view a small
and then sampled by a diode gate. The sampled signal segment of a waveform-for example, an oscillation or
is saved on a capacitor store, then amplified and fed to ringing that occurs during a small portion of a
the vertical plates. Unity feedback is used from the lower-frequency waveform.

Input
signal

Fig. 21.43 Sampling oscilloscope block diagram.


668 Electrical and Electronic Measurements and Instrumentation

Sometimes the delayed-sweep feature is used for


convenience to allow triggering at some other point
than at the leading or trailing edge. However, in some
situations a measurement is possible only if the
delayed-sweep feature is used. For example, suppose
the part of waveform that is to be measured is too far
from the only available trigger point to permit a stable
display on CRT screen. The problem can be solved by
using delayed sweep to trigger at the only available
trigger point and then starting the sweep at the point
of interest.
Although there are a few exceptions, delayed
sweep is normally a feature of dual-time base
oscilloscopes which have two completely separate
sweep generators. One sweep functions as a main
sweep, and the other serves as the delayed sweep. The
main sweep is initiated by a trigger pulse at the
leading edge of pulse 1 shown in Fig. 21.44. Suppose leading edge of pulse 1 in Fig. 21.45. The main sweep
we wish to observe in detail a portion of the waveform generator ramp, which is applied to a comparator
near its trailing edge by expanding the waveform and along with a voltage from a delay control circuit,
using a higher sweep speed-for example, a sweep increases linearly until it trips the comparator at
speed of 0.1 ps/cm. As can be seen in Fig. 21.43, the time fj. When the comparator changes states, the
portion of the waveform of interest to us is completely delayed-sweep generator is triggered, which
off the screen to the right when the sweep speed is set intensifies a portion of the original display. Adjusting
to 0.1 ps/cm. To observe the portion of the waveform the delay time should intensify the portion of the
that is of interest at the higher sweep speed, we must waveform that is of interest. The exact front panel
use the delayed-sweep feature and reset the sweep adjustments that are necessary to display the
speed to 5 ps/cm. expanded portion of the waveform on the CRT screen
depend on the oscilloscope being used. Some
oscilloscopes have an alternate sweep separation control
so that two waveforms are displayed: the original
waveform, with the portion of the waveform of
interest intensified, and a second expanded
waveform, as shown in Fig. 21.47. By adjusting the

Fig. 21.44 CRT display with sweep speed


equal to 5 ps/cm.

Basically, the delayed sweep works as follows.


The main sweep is initiated by a trigger pulse at time
f shown in Fig. 21.46. This time corresponds to the Fig. 21.46 Delayed-sweep triggering waveforms.
Cathode Ray Oscilloscope (CRO) 669

horizontal mode to channel B, we can now increase simple electrode arrangement to illustrate this
the sweep time so that we can view the portion of the principle. When a beam of electrons from the electron
waveform that is of interest. gun strikes the target it emits secondary electrons,
which are gathered by the collector. The collector is at
a positive potential of V2, the target is at a potential of
Vr which can be varied, and the electron gun is at
ground potential.
If I is the value of the current in the primary
electron beam, coming from the electron gun, and I is
the electron current emitted from the target and
collected by the collector, then the ratio Is / Ip is called
the secondary emission ratio. The value of this ratio
depends on the primary electron velocity and intensity,
and on the chemical composition of the target.
Figure 21.49 shows the variation of the secondary
emission ratio with target voltage, for the arrange­
ment of Fig. 21.48. When the target voltage is much
greater than the collector voltage, all secondary
Fig. 21.47 Delayed-sweep measurement.
electrons emitted from the target are attracted back to
21.23 ANALOG STORAGE OSCILLOSCOPES it. Therefore the collector current, or secondary
current, is zero, and so also is the secondary emission
Storage oscilloscopes, capable of retaining the
image on the screen for longer than that possible with ratio. The operating point is now well to the right of V2
conventional high persistence phosphors, have many in Fig. 21.49. If the target voltage is slightly negative,
applications. Examples of these are the capture and as at point F, then all the electrons from the gun are
storage of transients, and the steady display of a very deflected on-to the collector, before they reach the
low frequency signal. Two techniques are used to target. Therefore although there is no secondary
store signals in an oscilloscope, and these are called emission the collector current equals the beam current,
analog and digital storage. Analog storage is capable and the secondary emission ratio is unity. This point is
of higher speeds, but is less versatile than digital known as the lower stable point. As the target voltage
storage. This section describes the principles of analog increases from this point electrons are attracted away
storage, and the two methods most frequently used, from the collector, but they do not have enough
called variable persistence and bistable storage. energy to release secondary electrons from the target.
Digital storage oscilloscopes are covered in Art. 21.24. Therefore the secondary emission ratio falls to a
minimum at A. Beyond the minimum point secondary
21.23.1 Principle of Secondary Emission
emission from the target starts to occur, and these
Both the variable persistence and bistable storage electrons are accumulated by the collector, so
oscilloscopes depend, for their operation, on the increasing the secondary emission ratio.
principle of secondary emission. Figure 21.48 shows a

Fig. 21.49 Variation of secondary emission ratio for


Fig. 21.48 Simple electrode arrangement to the electrode arrangement shown in
illustrate secondary emission. Fig. 21.48.
670 Electrical and Electronic Measurements and Instrumentation

The secondary emission ratio increases through


the crossover point E until it reaches a peak at C.
Beyond this point secondary electrons emitted from
the target are attracted back, in numbers greater than
those which reach the collector, so that the secondary
emission ratio decreases sharply. The curve reaches
the upper stable point at G, where the primary and
secondary currents are equals, and then decreases to
zero.
The lower and upper stable points represent the
erased and written conditions of the CRT screen, and
in the absence of a target voltage the target can remain
in one of these two stages only. Suppose for instance
that the target is in a state to the left of the crossover
point E arid switch SW is opened to remove the target
supply voltage. Because the secondary emission ratio
is less than one, fewer electrons leave the target than
arrive at its surface, so the target is driven Fig. 21.50 Multiple target and flood gun arrangement.
progressively negative, until point F is reached. Now
the number of electrons reaching the target equals the momentarily taken to a negative voltage. This repels
number which are emitted from it. This represents a secondary emission back into the target, moving its
stable erased state. If the target was to the right of the potential negative due to the electrons reaching it
crossover point when switch SW is opened, then, since from the flood gun. When the target reaches point A
the secondary emission ratio is greater than one, more no more chanrging occurs. The collector mesh is now
electrons leave the target than arrive at its surface. The returned to its original voltage of +V2. This is done
target is therefore driven positive unitl it reaches the gradually to prevent the target from being
upper stable point G which represents the written accidentally driven beyond the crossover point, due to
condition of the screen. The target will stay at the capacitive coupling, and so reaching a written state
upper or lower stable point so long as the electron gun again.
is on. At the crossover point the target is unstable, and Secondary
it will move up or down on the curve depending on emission ratio
the direction in which it is sent by noise.
Figure 21.50 shows an arrangement using a
segmented multiple target, with a collector mesh in
front through which the electrons can pass. Two guns
are used, a high energy writing gun, which emits a
narrow beam of electrons, and a flood gun which
covers the target area with a continuous stream of low
energy electrons. The flood gun remains on all the
time, and maintains the target in either of the two
Fig. 21.51 Secondary emission ratio curves for the
stable states. The writing gun is used to move the
arrangement of Fig. 21.50.
target from the lower to the upper stable state.
Figure 21.50 shows the secondary emission ratio The principle of secondary emission storage
curves for the electron arrangement of Fig. 21.50. described in this section, is applicable to both variable
Suppose that the target is at point A, the erased state. persistence and bistable storage, as described in the
The writing gun is truned on and its high energy next two sections.
electrons result in a large amount of secondary
21.23.2 Variable Persistence Storage
emission, and drive the target towards the written
state. Up to point C the flood gun opposes the writing The variable persistence storage technique is also
gun, but thereafter it aids it; so the writing gun can be known as halftone storage or mesh storage.
swtiched off and the target will reach the written state Figure 21.52 illustrates the construction of a CRT using
at D. To erase the target the collector mesh is this storage technique. There are two screens, a
Cathode Ray Oscilloscope (CRO) 671

storage mesh which retains the image traced on it by onto the display phsophor. The charge on different
the writing gun, and the phosphor screen, which is parts of the storage screen can be varied either by
very similar to that used in a conventional CRT. control of the time spent by the beam on different
Dielectric material, consisting of a thin layer of areas, or by varying the beam current over different
material such as magnesium fluoride, is deposited on areas. Usually four levels can be readily obtained, for
the storage mesh, and this acts as the stroage target. example a mesh voltage of - 10 V represents an off
The writing gun is at a high negative voltage, the flood state, - 5 V and - 2.5 V represent intermediate states,
gun at a few volts negative, the collector mesh at about and 0 V represents full brightness.
100 V positive, and the storage mesh at ground Variable persistence tubes enable the persistence
potential or a few volts negative. The collimator or storage time of the image to be varied from a few
consists of a conductive coating on the inside surface milliseconds to several hours. It can be done in the
of the CRT. It is biased so as to distribute the flood gun CRT shown in Fig. 21.52 by applying a repetitive
electrons evenly over the surface of the target, and negative pulse, of - 4 V to -11 V, to the storage mesh,
causes the electrons to be perpendicular to the storage so as to discharge the positively charged areas. The
mesh. control over the discharge time is usually obtained by
operating the pulses at a constant frequency of about
When the writing gun is aimed at the storage 1 kHz, but varying the pulse width ; the narrowest
target it causes areas where it strikes to be charged to a pulse gives the largest persistence as it takes longer to
positive potential, due to secondary emission as in discharge the storage layer.
Fig. 21.51. These areas are maintained at their upper
Variable persistence storage finds many
stable point, even after the writing gun is switched off,
applications, such as the storage of an entire
due to the action of the flood gun. Electrons from, the
waveform of a slow moving signal, which then fades
flood gun also pass through those areas which are before the next trace is written. It can also be used to
positively charged, causing the phosphor beyond to store several traces before the first one fades, so as to
glow, and displaying the original signal traced by the see how the signal changes with time.
writing gun.
21.23.3 Bistable Storage Oscilloscope
The bistable storage tube is between two and ten
times slower than a comparable variable persistence
tube. However it is capable of much longer storage
times, measured in hours rather than in minutes as for
variable persistence. The bistable tube is also capable
of operating in a split screen mode, where half the
screen has storage capability and the other half is a
conventional phosphor tube.
Figure 21.53 shows the construction of a bistable
Fig.21.52 Construction of a variable persistence storage tube. Unlike the variable persistence tube the
storage tube. same phosphor screen is used for both storage and
display. The screen consists of Pl phosphor, doped to
The pattern stored on the storage target lasts for
about one hour, but it can display a bright image for have good secondary emission characteristics and
about one minute. The stored pattern fades due to deposited on a conductive backplane made from a
electrons from the flood gun charging other parts of transparent metal film. The phosphor layer consists of
the storage surface, giving an impression that the a thin coating of scattered particles, so as to give a
whole screen has been written. This is known as discontinuous surface. This stops the boundary
fading positive. To erase an image which has been migration of stored charge. The thin phosphor coating
stored the storage mesh is momentarily raised to the also has a short life since it suffers from light output
same positive potential as the collector mesh. reduction with time.
The variable persistence storage tube is capable of The conductive film is held at a low positive
producing halftone or gray scale images. As the potential, so as to attract a cloud of low energy
charge on the storage surface is increased, beyond a electrons from the flood gun. These electrons have
critical threshold value, more electrons pass through it insufficient energy to penetrate the phosphor, and are
672 Electrical and Electronic Measurements and Instrumentation

gathered by the collimator. When the write gun is Expansion storage, shown in Fig. 21.54 uses an
switched on, its high energy electrons result in intermediate mesh which is about one fifth the size of
secondary emission from areas traced on the screen, the viewing screen. This mesh can be written on very
moving these to the upper stable point. The trace is quickly, and it is placed between the collector and
therefore at a high positive potential, and this is accelerator meshes. The image on the storage mesh is
maintained due to the low leakage of the phosphor. magnified by a static electron lens system, to give a
The low energy electrons from the flood gun are now full size display on the viewing, screen. This system is
attracted to the positive areas of the screen, and go capable of writing speeds of 2000 cm/ps, and can
through the phosphor to reach the metal film at the capture signals at frequencies of 300 MHz.
back. In passing through the phosphor they cause it to Flood
glow, displaying the area traced out by the writing
gun. The screen can be erased by setting the metal film
to a negative voltage, repelling the electrons back into
the storage area, and returning the phosphor to the
low stable point.
lens mesh
Fig. 21.54 Expansion storage tube construction.

21.24 DIGITAL STORAGE OSCILLOSCOPES

21.24.1 Principle of Operation


Writing The availability of electronic circuitry at low cost
gun has enabled many digital features to be added to
analog oscilloscopes. Examples of these are generation
of a trigger after an elapsed time or after a count of a
number of pulses ; digital display of the parameters;
Fig. 21.53 Construction of a bistable storage tube. integral digital voltmeter and counter: remote control.
However the basic oscilloscope still remains analog,
21.23.4 Fast Storage Oscilloscopes and uses an analog storage CRT, as described in
Two modifications are made to the conventional Art. 21.2.
storage CRTs, to enhance the speed with which they A digital oscilloscope digitises the input signal, so
can capture transient information. These methods are that all subsequent signals are digital. A conventional
known as transfer storage and expansion storage. CRT is used, and storage occurs in electronic digital
Transfer storage uses an intermediate mesh target, memory. Figure 21.55 shows a block diagram of a
called the fast mesh, which has been optimized for basic digital storage oscilloscope. The input signal is
speed. The waveform is initially written onto this digitised and stored in memory in digital form. In this
mesh and it is then transferred, at lower speed, onto a state it is capable of being analysed to produce a
second mesh which has been optimized for storage variety of different information. To view the display
time. The second mesh can be bistable or variable on the CRT the data from memory is reconstructed in
persistence. analog form.

Fig. 21.55 Block diagram of a basic digital storage oscilloscope.


Cathode Ray Oscilloscope (CRO) 673

Digitising occurs by taking a sample of the input digital converter, and the results stored in a digital
waveform at periodic intervals. In order to ensure that store. This method allows operation at up to 100 mega
no information is kst, sampling theory states that the samples per second, and has the advantage that a low
sampling rate must be at least twice as fast as the cost analog to digital converter can be used, whose
highest frequency in the input signal. If this is not resolution does not decrease as the sampling rate is
done then aliasing will result, as shown in Fig. 21.56. changed. The disadvantage is that the oscilloscope
This requirement for a high sampling rate means that cannot accept data during the digitising period, so it
the digitiser, which is an analog to digital converter, has a blind spot. At low sweep speed operations it is
must have a fast conversion rate. This usually requires usual to switch out the analog memory, feeding the
expensive flash analog to digital converters, whose analog to digital converter in real time.
resolution decreases as the sampling rate is increased.
Many different input channels are used with
It is for this reason that the bandwidth and resolution
digital storage oscilloscopes. However if all these
of a digital oscilloscope is usually limited by its analog
channels share a common store, through a multi­
to digital converter.
plexer, then the memory available to each channel is
a Sample e reduced. Oscilloscopes with up to 40 channels are
commercially obtainable, with a storage capability of
25000 dots. Several oscilloscopes also have floppy disc
storage capability to allow non volatile storage of
waveforms, which can later be recalled into the
(«) oscilloscope and manipulated.

21.24.2 Waveform Reconstruction


Although the input signal may be sampled at
greater than twice the highest signal frequency,
aliasing can still result when the output is present as a
series of dots, corresponding to the sampled values.
This is illustrated in Fig. 21.58(a), where the user's
Fig. 21.56 Effect of a low sampling frequency : mind connects together the dots which are physically
(a) input signal, (b) aliased signal. closest to each other, rather than those which are
One method of overcoming the need for a high closest on the time scale.
performance converter is to use an analog store, as in In the illustration of Fig. 21.58(a) it is difficult to
Fig. 21.57. The input signals are sampled, and these visualise the final waveform, and oscilloscopes
are stored in an analog shift register. They can then be generally have the facility to interpolate between the
read out at a much slower rate to the analogue to dots, if required by the user. Two techniques are used,

Fig. 21.57 A digital oscilloscope which uses analogue storage to eliminate the need for a
very fast analogue to digital converter.
674 Electrical and Electronic Measurements and Instrumentation

linear interpolation and sinusoidal interpolation. In Another practical diagram of a digital oscillo­
linear interpolation, shown in Fig. 21.58(b) a straight line scope is shown in Fig. 21.59.
is used to connect the dots together. This works well Another problem with the sampling technique
on a pulsed or square waveform, but not on a used in digital oscilloscopes is that it can miss short
sinusoidal wave, Fig. 21.58(c) shows that sinusoidal term transient, or 'glitches', which occur in between
interpolation gives a much better fit for sine waves, the sample points. To overcome this problem envelope
although it is not suitable for pulse or square waves. mode oscilloscopes may be used. These have special
logic circuitry which causes the sample and digitising
circuitry to run at a high speed, independent of the
setting of the display time. At each sample the value is
compared with the previous stored sample, and the
higher (or lower) value is stored. This is continued for
the screen interval, so that for that interval the highest
and lowest points are always stored. For example,
suppose that an oscilloscope digitises every 2 ms, at a
given sweep speed. If a 0.1 ms transient were to occur
there is a high probability that a conventional digital
(b)
oscilloscope would miss it. In an envelope mode
oscilloscope the input would be sampled, say, every
200 ns, but only the highest, or lowest, values that
occur within a 2 ms window would be stored in
memory. Therefore the transient would be recorded.
(c) The sample rate of the oscilloscope is controlled by the
time setting of the oscilloscope, but the analog to
Fig. 21.58 Illustration of interpolation methods :
digital converter runs much faster.
(a) without interpolation, (b) Linear
interpolation, (c) sinusoidal interpolation.

f ig. 21.59 Block diagram of a digital storage oscilloscope.


Cathode Ray Oscilloscope (CRO) 675

21.25 COMPARISON BETWEEN ANALOG AND oscilloscope. The digital oscilloscope is also able to
DIGITAL STORAGE OSCILLOSCOPES operate in a baby sitting mode. When the scope is
The advantage of the analog storage oscilloscope triggered it prints out the stored results onto a hard
is that it has a higher bandwidth and writing speed copy recorder (or disc storage), and then re-arms itself
than a digital oscilloscope, being capable of operating ready for another reading.
speeds of about 15 GHz. The digital oscilloscope is
21.26 ACCESSORIES OF CATHODE RAY
primarily limited in speed by the digitising capability
OSCILLOSCOPES
of the analog to digital converter. Aliasing effects also
limit the useful storage bandwidth (usb) of the The cathode-ray oscilloscope is one of the most
oscilloscope to a value given by the ratio. useful instruments in the electronic industry. The
usefulness of the oscilloscope is further extended by
, maximum digitising rate
usb =----------------- 5-------------- ...(21.26) provision of accessories or auxiliary equipment. Some
constant C
of the accessories are described below.
The value of constant C is dependent on the
21.26.1 Calibrators
interpolation method used between the dots. For a dot
display C should be about 25, to give an eligible Many oscilloscopes have a built-in reference
display: for straight line interpolation it should be source of voltage which has usually a square
about 10, and for sinusoidal interpolation C should be waveform with a frequency of 1 kHz. The voltage may
about 2.5. have a single magnitude or may be of several
selectable magnitudes. Provision of this facility
The digital storage oscilloscope has a CRT which
enables the oscilloscope amplifiers and time base to be
is much cheaper than an analog storage oscilloscope,
calibrated and checked for accuracy.
making replacements more economical. The digital
oscilloscope is also capable of an infinite storage time, 21.26.2 Probes
using its digital memory. Furthermore it can operate The probe performs the very important function
with a constant CRT refresh time, so giving a bright of connecting the test circuit to the oscilloscope
image even at very fast signal speeds. The digital without altering, loading, or otherwise disturbing the
storage oscilloscope is not, however, capable of test circuit.
functioning in a variable persistence storage mode.
The probes are of three different types :
The time base in a digital oscilloscope is generated
(/) direct reading probe,
by a crystal clock, so that it is more accurate and stable
(n) circuit isolation probe, and
than an analog oscilloscope, where the time base is
generated by a ramp circuit. The analog to digital (zzz) detector probe.
converter used in a digital oscilloscope also gives it 1. Direct reading probe. This probe is simplest of
higher resolution than an analog oscilloscope. For all the probes and uses a shielded co-axial cable. It
example a twelve bit digitser can resovle one part in avoids stray pick-ups which may create problems
4096. A conventional analog oscilloscope typically when low level signals are being measured. It is
resolves to about 1 mm on the screen, that is to about usually used for low frequency or low impedance
one part in 50, equivalent to 6 bit resolution. circuits. However, in using the shielded probe, the
Digital storage oscilloscopes are also capable of shunt capacitance of the probe and cable is added to
operating in a look back mode, as described for the input impedance and capacity of the scope and
waveform recorders. An analog oscilloscope collects acts to lower the response of the oscilloscope to high
data after it has been triggered. A digital storage impedance and high frequency circuits.
oscilloscope is always collecting data, and the trigger External high impedance probes are used to
tells it when to stop. The oscilloscope can stop increase the input resistance and reduce the effective
immediately on trigger, so that all the stored input capacitance of an oscilloscope. A resistor and capa­
information is pretrigger, or it can stop some time after citor combination can be added to an oscilloscope as
it has received the trigger. If the delay is longer than shown in Fig. 21.60, in effect, moving the input termi­
the storage capability of the oscilloscope, then all the nals from the front panel of the instrument to the end
storage capability of the oscilloscope, then all the of the probe, supposing, it is intended to attenuate the
stored information is post trigger, as for an analog signal by a factor of 10. By using Eqns. 21.24 and 21.25,
676 Electrical and Electronic Measurements and Instrumentation

where k = 10, the values of Rj and Cj can be computed small, the leading edge of the square wave is rounded
as under off but if value of is too large, the leading edge of
Rj = R^k-1) =(lx 106)(10-1) = 9MQ square wave overshoots.
q = c2 /(k -1) =30x 10"12 /(10 -1) =3.33 pF 2. Isolation probe. Isolation probe is used in
order to avoid the undesirable circuit loading effects
New input impedance R( is the total resistance,
of the shielded probe. The isolation of the probe,
where R- = R^ + =10MQ which is used along with a capacitive voltage divider,
C C, decreases the input capacitance and increases the
and C- = —-—— = 3 pF
' q + C2 F input resistance of the oscilloscope. This way the
loading effects are drastically reduced.
The input resistance has been increased by a
3. Detector probe. When analyzing the response
factor of 10 while, at the same time, the input
to modulated signals used in communication equip­
capacitance has been decreased by a factor of 10. The
ment like AM, FM and TV receivers, the detector
combination of RT and CT is called xlO (times 10)
probe functions to separate the low frequency
probe.
modulation component from the high frequency
carrier. The amplitude of the modulator carrier (which
is proportional to the response of the receiver to the
much high frequency carrier signal) is displayed on
the oscilloscope by rectifying and bypassing action.
This permits an oscilloscope capable of
audio-frequency response to perform signal tracing
tests on communication signals in the range of
hundreds of MHz, a range, which is beyond the
capabilities of all oscilloscopes except the highly
Fig. 21.60 External High Impedance Probe.
specialized ones.
Capacitor Cv is usually adjustable to compensate
21.26.3 Cameras
for differences in input capacitance between
oscilloscopes. If the probe capacitance is adjusted to The best method of permanent records of
the wrong value, the oscilloscope will exhibit a factor oscilloscope traces for analysis is the use for
frequency response. The adjustment of probe is photography. Special cameras are used for the
usually checked by displaying a square wave on the purpose and they are of the following two types :
CRT screen. If the probe is not properly compensated, (z) 35 mm film. The 35 mm unit is usually
the display of square waveform will be adversely prefocused and bolted over the tube face. Some
affected as shown in Fig. 21.61. If the value of Q is too cameras have a fixed lens aperture, the exposure is
simply controlled by hand operation of flapper
shutter. More sophisticated arrangements use
exposure and aperture control. Panchromatic and
special blue sensitive films are commonly used
because they are faster and hence are better suited for
(a) Correct compensation (b) Value of Q too small recording of high speed transients. Both these films
give a "negative" record from which positive prints
having white traces on a black background are made.
Another type of 35 mm film gives a photograph which
has a black trace on a white background. These films
are, however, difficult to enlarges.
(zz) Polaroid film. This photographic technique
approaches instant photography. A permanent record
is obtained within 10-20 s. However, this process is
(c) Value of C; too large.
much more expensive than the 35 mm photographic
Fig. 21.61 Effect of probe capacitance compensation technique. But the relative cost may be reduced by
on display of square waveform. using the facility of some of the cameras to be moved
Cathode Ray Oscilloscope (CRO) 677

relative to the screen, making it possible to place From Eqn. 21.17, deflection sensitivity,
several traces on a single print. The camera is again 0.5 x 1.5 x IQ'2
S = —— m/V = 0.375 mm/V.
prefocused, but is normally provided with aperture 2d£fl 2 x 5 x 10-3 x 2000
and exposure controls. The photographic record is
obtained on Polaroid film which is a white trace on a 1 1
Deflection factor, G = - =------- = 2.66 V/mm.
black background. Since in many situations, it is the S 0.375
time which is much more important, the Polaroid Example 21.10 A CRT has an anode voltage of2000 V
photography is normally used under such circum­ and parallel deflecting plates 2 cm long and 5 mm apart. The
stances. screen is 50 cm from the centre of the plates. Find the input
voltage required to deflect the beam through 5 cm. The input
21.26.4 ElectronicSwitch voltage is applied to the deflecting plates through amplifiers
The electronic switch is an extremely useful having an overall gain of 100.
accessory for displaying two signals simultaneously Solution. From Eqn. 21.16, deflection
on single trace oscilloscopes. The electronic switch is
D=d d
essentially a square wave generator. The two signals 2dEan 1
which are to be displayed are each connected to a
separate grid of a dual-section amplifier tube, Voltage applied to deflecting plates
operating with cutoff bias. The square wave is applied _2dEa D _2x5xl0“3x2000x3xl0'2
in such a manner that it alternately unblanks each tube d~ Lld ” 03x2 xlO-2
section, thereby presenting each signal alternately to
= 100 V.
the vertical amplifier of the oscilloscope. When the
switching rate, which is determined by the square .•. Input voltage required for a deflection of 3 cm
wave frequency, is high enough each signal appears = ^ = wo = lv
continuous even though it is actually interrupted gain 100
periodically. The switching rates may be as high as
Example 21.11 Calculate the velocity of the electron*12
100 kHz. beam in an oscilloscope if the voltage applied to its vertical
Example 21.8 Calculate the maximum velocity of the deflection plates is 2000 V. Also calculate the cutoff
beam of electrons in a CRT having a cathode anode voltage frequency if the maximum transit time is 1/4 of a cycle. The
of800 V. Assume that the electrons to leave the cathode with length of horizontal plates is 50 mm.
zero velocity. Charge of electron = 1.6x 10~19 C and mass of Solution. Velocity of electron beam
electron = 9.lx 10~31 kg.
12 x 16x10 x 2000 =26 5 x 106 m/s
Solution. From Eqn. 21.5, velocity of electron is :
V 9.1 xlO'31
Cutoff frequency
__vox _ 26.5xlO6
2xL6xlO~19 dc~ 41 -4x50xl0-3
9.1 xlO-31 = 132.5 x 106 Hz = 132.5 MHz.

= 16BxlO3 m/s. Example 21,12 Voltage E1 is applied to the horizontal


input and voltage E^ is applied to the vertical input of a
Example 21.9 An electrically deflected CRT has a final CRO. Ey and E2 have the same frequency. The trace is an
anode voltage of2000 V and parallel deflecting plates 1.5 cm
ellipse. The slope of the major axis is positive. The maximum
long and 5 mm apart. If the screen is 50 cm from the centre
vertical value is 2.5 divisions and the point where the ellipse
of deflecting plates, find (a) beam speed, (b) the deflection
crosses the vertical axis is 1.25 divisions. The ellipse is
sensitivity of the tube, and (c) the deflection factor of the
symmetrical about the horizontal and vertical axis.
tube. Determine the possible phase angles of E^ with respect to Er
Solution. Velocity of beam Solution. Referring to Fig. 21.34(a),
2xL6xlQ~19 Y1 1.25
OX x2000 sin<|> = = 0.5 .-. (|) = 30o
9.1 xlO-31 Y2 " 2.50

= 26.5 m/s. Thus possible phases are 30° or 330°.


678 Electrical and Electronic Measurements and Instrumentation

Example 21.13 sketches shown in Fig- 21.62 Solution. The spot generating the patterns moves
display Lissajous patterns for cases where voltages of same in clockwise direction.
frequency out of different phase arc connected to Y and X y n
(a) sin<j> = -i = -=0
plates of the oscilloscope. Find the phase difference in each y2
case. The spot generating the patterns moves in a clockwise
direction. Calculate the angles if the spot generating the
.-. 4>=o°
patterns moves in the anticlockwise direction. (as the Lissajous pattern is in 1st and 3rd quadrants).
25
(b) sin<j> = y =0.5-

.-. <b = 30°


(as the major axis of the pattern is in 1st & 3rd quadrants).
35
(c) sin<|) = — =0.7

.-. (j) = 45°


(as the major axis of the pattern is in 1st & 3rd quadrants).
25
(d) sin<|> = —=0.5

.-. (j> = 180°-30°=150°


(as the major axis is in 2nd and 4th quadrants).
If the spot generating the patterns moved in the
counter clockwise direction, the angles would be :
(a) 180°, (&) -30°,
(c)-45°, (d) 180+30 =210°.
Fig. 21.62 Diagrams of Example 21.13.

Review Questions
1. Describe an overview of applications of a CRO. 9. Describe in details the vertical amplifier used in a
2. Draw the block diagram of a general purpose CRO CRO.
and explain the functions of the following controls 10. Describe the phenomenon of synchronization of
(i) intensity vertical input signal to its sweep generator.
(ii) focus 11. Describe the different types of sweeps used in a
CRO. Explain their spheres of application.
(iii) horizontal and vertical positioning
12. Describe the function of attenuators in CROs.
(iv) synchronization.
Explain how are they designed with particular
3. Describe the different parts of a CRT.
reference to frequency compensation.
4. Derive an expression for vertical deflection of an 13. Describe the following :
electron beam in a CRT.
(i) Sources of Synchronization.
5. Describe the different types of phosphor materials
used in a CRO and list their applications. (ii) Blanking circuit.

6. Explain the different types of graticules used in a (iii) Z-axis modulation.


CRO. Describe their advantages and disadvantages. (iv) Astigmatism control.
7. Explain the functioning of a time base generator in 14. Describe the following types of oscilloscopes
a CRO. (i) Dual trace type (ii) Dual beam type.
8. What are the different types of amplifiers used for 15. Describe in details the construction and working of
CROs ? Describe the basis on which they are an analog type storage oscilloscope. Explain the
classified. principle of secondary emission.
Cathode Ray Oscilloscope (CRO) 679

16. Explain the following features of an analog type 18. Discuss the advantages and disadvantages of
storage oscilloscope : analog and digital type of oscilloscopes.
(z) bistable persistence storage 19. Discuss in details the delayed sweep.
(iz) bistable storage 20. Describe how the following measurements can be
(in) fast storage. made with the use of a CRO :
17. Describe the principle of working and circuit (z) frequency
diagram of a digital oscilloscope. (ii) phase angle.

Unsolved Problems
1. The deflection sensitivity of an oscilloscope is 4. Compute the value of resistors in the attenuator
35 V/cm. If the distance from the deflection plates network shown in Fig. 21.65 to provide the
to the CRT screen is 16 cm, the length of the volts/division values indicated for each position.
deflection plates is 2.5 cm, and the distance
between the deflection plates is 1.2 cm. What is the
acceleration anode voltage ? [Ans. 583 V]
2. What is the amplitude of the voltage Vo in
Fig. 21.63 after 20 ms ? [Ans. 22 V]

Vf. = 40 V

R = 500 kQ

^0
C = 0.05 nF

Fig. 21.63 Circuit for Problem 2.

3. Identify each of the items indicated in the drawing


in Fig. 21.64. [Ans. 22 V]

Fig. 21.65 Attenuator network for problem 4.

[Ans. R, = 6 kQ, R^ = 9kQ, Ry = 45kQ,


R4 = 90kQ , R5 = 450kQ, R
* = 900 kQ,
4 =4.5MQ, ^=9X40]
Fig.21.64 Diagram for problem 21.3.
680 Electrical and Electronic Measurements and Instrumentation

5. The waveform shown in Fig. 21.66 is observed on a the deflecting voltage required to cause the beam
CRT screen. If the time/div switch is set 10 gs and to strike a deflecting voltage and find the
the volt/div switch is set to 200 mV. Determine the corresponding deflection of the screen.
frequency and peak-to-peak amplitude of the [Ans. 200 V ; 4 cm]
signal. [Ans. 33.33 kHz, 600 mV]
11. Voltage £[ is applied to the horizontal input and E^
to the vertical input of a CRO. and have same
frequency. The trace on the screen is an ellipse. The
slope of major axis is negative. The maximum
vertical value is 3 divisions and the point where
the ellipse crosses the vertical axis is 2.6 divisions.
The ellipse is symmetrical about horizontal and
vertical axis. Determine the possible phase angle of
E^ with respect to El. [Ans. 120° or 210°]
12. A Lissajous pattern on an oscilloscope is stationary
Fig.21.66 CRT display for problem 5. and has 5 horizontal tangencies and 2 vertical
tangencies. The frequency of horizontal input is
6. What value should Q have for Vo to be equal to 1000 Hz. Determine the frequency of vertical input.
0.1V in the circuit shown in Fig. 21.67 ? [Ans. 2500 Hz]
13. In the oscilloscope pattern shown in Fig. 21.68 the
signal connected to the vertical plates has the same
frequency in parts (a) and (b). If the input to the
horizontal plates is a 50 Hz voltage :
(a) state the unknown frequency, and
(b) explain what causes the difference in appear­
ance of two patterns. [Ans. 25 Hz].

[Ans. 5 pF]
7. If the vertical amplifier of an oscilloscope has a
bandwidth of 15 MHz, what is the fastest rise time
that an input may have to be displayed without
distortion ? [Ans. 23.33 ns]
8. A high-impedance probe with 9MQ resistance and
4 pF capacitance is connected to an oscilloscope
with an input resistance of 1 MQ. If the effective Fig. 21.68 Diagram of problem 13.
capacitance decreased to 3.6 pF when the probe
was connected. What is the capacitance of the 14. Prove that the Lissajous patterns produced by
oscilloscope alone ? [Ans. 36 pF]
voltages applied to Y and X plates as shown in
Figs. 21.69 (a) and (b) have the following frequency
9. In a cathode ray tube the distance between the
ratios :
deflecting plates cm, the length of the
deflecting plates is 4.5 cm and the distance of the (fl) 5 : 4 (ft) 8:1.
screen from the centre of the deflecting plates is
33 cm. If the accelerating voltage supply is 300 volt,
calculate deflecting sensitivity of the tube.
[Ans. 2.48 mm/V]
10. An electrostatically deflected cathode ray tube has
plane parallel deflecting plates which are 2.5 cm
long and 0.5 cm apart, and the distance from their
centre to the screen is 20 cm. The electron beam is
accelerated by a potential difference of 2500 V and
is projected centrally between the plates. Calculate Fig. 21.69 Diagram of problem 14.
Cathode Ray Oscilloscope (CRO) 681

15. Find the frequency of the vertical plates if the 18. Calculate the ratio of vertical to horizontal
frequency applied to horizontal plate is 50 Hz for frequencies for an oscilloscope which displays the
the patterns shown in Figs. 21.70 (a) and (b). following Lissajous figures shown in Fig. 21.73.
[Ans. (a) 25 Hz, (b) 100 Hz] The input in the case of (d) is applied to Z-axis.
@ o

(«) (b) («) (W

Fig. 21.70 Diagram of problem 15.


16. A sinusoidal input is applied to the vertical input
of an oscilloscope starting at t = 0. The following
Lissajous patterns are obtained when a sinusoidal
input is applied to the horizontal terminals. Estimate
the phase shift between vertical and horizontal
inputs. [Ans. 0°, 90°, 180°, 45°, 60° leading]
S3 O (C) W

Fig. 21.73 Lissajous Figures of problem 18.


[Ans. A /fy: (a) 2 : 1 (b) 1 : 2 (c) 2 : 3 (d) 1 : 8]
19. Determine the phase shift between two sine waves,
which is indicated by the pattern shown in
Fig. 21.74. [Ans. 53°]
(a) (b) (c)

(<0 (e)
Fig. 21.71 Lissajous figures of problem 16. Fig. 21.74 CRT display for problem 19.
17. If the horizontal frequency is 51 Hz, determine the
vertical frequency for the Lissajous pattern shown 20. A sampling oscilloscope is being used to observe a
in Fig. 21.72 400 MHz sine wave. A sampling pulse occurs
every 3 ns. Draw five cycles of the 400-MHz signal
and place a dot at the sampled point on each of the
five cycles.

Fig. 21.72 Lissajous figure for problem 17.

Objective Type Questions T

Tick (^) the most appropriate answer : (b) a barium and strontium oxide coated cathode
1. The source of emission of electrons in a CRT is : (c) accelerating anodes

(a) PN function diode (d) post-accelerating anodes.


682 Electrical and Electronic Measurements and Instrumentation

2. In a CRT the focusing anode is located 10. The bandwidth of a CRO is from 0 - 20 MHz. The
(a) between pre-accelerating and accelerating anodes fastest rise time a sine wave can have to be
accurately reproduced by the instrument is
(b) after accelerating anode
(a) 35 ns (b) 35 ps (c) 17.5 ns (d) 0.175 ps.
(c) before pre-accelerating anode
11. The horizontal amplifier should be designed for
(d) none of the above.
3. The deflection of an electron beam on a CRT screen (a) high frequency signals with a fast rise time
is 10 mm. Suppose the pre-accelerating anode (b) high amplitude signals with a slow rise time
voltage is halved and the potential between (c) high amplitude signals with a fast rise time
deflecting plates is doubled, the deflection of the
(d) low amplitude signals with a fast rise time.
electron beam will be :
12. A vertical amplifier for a CRO can be designed for:
(a) 80 mm (b) 40 mm (c) 20 mm (d) 10 mm.
4. If the distance of screen from a CRT to centre of (a) only a high gain (b) only a broad bandwidth
deflection plates is 15 cm, the length of deflection (c) a constant gain times bandwidth product
plates is 2 cm, the distance between plates is 1 cm (d) all the above.
and accelerating voltage is 500 V, the deflection 13. In an oscilloscope, when the unknown signal
sensitivity is : applied to the vertical plates is being synchronized
(a) 33.2 V/cm (b) 0.03 cm/V with the sweep signal applied to horizontal plates,
(c) 60.4 V/cm (d) 0.015 cm/V. the pattern seen on the CRO screen moves towards
5. Post acceleration is needed in a CRO if the the right. It means that,
frequency of the signal is : (a) the frequency of the signal is lower than that of
(g) less than 1 MHz (b) more than 1 MHz the sweep signal
(c) more than 10 MHz (d) more than 10 Hz. (b) the frequency of the signal is greater than that
6. phsophor material is used for display in CRTs for: of the sweep signal
(c) the frequency of the signal is equal to the
(a) photographic applications
frequency of sweep signal
(b) general purpose applications
(d) none of the above.
(c) television applications (d) all the above.
14. An oscilloscope has an input capacitance of 50 pF
7. An aquadag is used in a CRO to collect : and a resistance of 2 MQ and the voltage divider
(a) primary electrons ratio of 10. The parameters of a high impedance
(b) secondary emission electrons probe are
(c) both primary and secondary emission electrons (a) q =5.55 pF, Rj = 18MQ
(d) none of the above. (b) q = 3.33 pF, Rj = 9MQ
8. A thin aluminium film is usually deposited on the (c) q = 11.1 pF, Rt =18MQ
non viewing side of the phosphor because, (d) q = 5.55 pF, Rj = 9MQ
(a) it acts as a heatsink and prevents phosphor bum 15. In a digital storage oscilloscope, the input signals are
(b) the light scatter from the phosphor is reduced (a) directly applied to the oscilloscope
(c) it does not allow the screen to be negatively (b) multiplexed, converted to digital form and
charged stored and applied to oscilloscope
(d) all the above. (c) multiplexed, converted to digital form stored,
9. During the retrace time, the electrons forming the converted to analog form and applied to oscil­
horizontal beam loscope
(fl) move from left to right on the screen (d) applied to amplifier, stored as analog signals,
(b) move from right to the left on the screen multiplexed, converted to digital form, stored
(c) mo"P from bottom to top of screen in digital form, converted to analog form, and
applied to CRO through an amplifier.
(d) move from top to bottom of screen.

1. (b) 2. (fl) 3. (b) 4. (b) 5. (c) 6. (b) 7. (b) 8. (d) 9. (b) 10. (c)
11. (b) 12. (c) 13. (fl) 14. (fl) 15. (d)
Instruments for Generation
of Waveforms

22.1 SIGNAL SOURCES 22.2 OSCILLATORS


Signal sources provide a variety of waveforms for An oscillator is one of the most basic and useful of
testing of electronic circuits at low power levels. The the electrical and electronic measuring instruments.
waveforms generated by several different kinds of Oscillators provide a convenient source of power of
instruments, which range in complexity from simple test voltage for practically measurements. An oscillator
fixed frequency sine-wave oscillators to highly provides a sine-wave signal output. Depending upon
sophisticated instruments such as those used for the oscillator design or application, terms such as
testing complex communication circuits. Some of the oscillator, test oscillator, signal generator or function
applications of signal generators include measuring generator are used. An oscillator is the basic element
the frequency response of amplifiers and the of all signal sources and generates a sine-wave signal
alignment of radio receivers. There are various types of known frequency and amplitude. This instrument
of signal generators, but the following types of covers a frequency range from a few Hz to GHz. A
characteristics are common to all types : simple oscillator has two basic blocks (z) an oscillator
(z) the frequency of the signal should be well and (z'z) an attenuator. The oscillator's performance
known and stable, depends upon the functioning of these two blocks.
(z'z) the amplitude of the output should be The accuracy, stability and freedom from distortion
controllable from very small to relatively depends upon the design of oscillator, while the ampli­
large values, tude accuracy depends upon the design of attenuator.

(zzz) the output signal should be free from 22.2.1 Classification of Oscillators
distortion.
Oscillators may be classified in several ways. Here
There are many variations of the above require­ they are classified on two bases : (z) the design
ments, especially for specialized signal generators principle used, and (zz) the frequency range over which
such as function gerenators, pulse generators and they are used.
pulse frequency generators. The classification based on design principle
Sine-wave generators, both in the audio and radio categorises the oscillators as : (z) Feedback oscillators
frequency' ranges are called Oscillators. Although, the and (zz) Negative resistance oscillators.
terminology is not universal, the term Oscillator is The classification based on frequency band is :
generally used for an instrument that provides only a
sinusoidal output signal. The term Function (0 Audio frequency (AF) 20 Hz - 20 kHz.
Generator is applied to an instrument that provides oscillators
several output waveforms, including sinewave, ('■'■) Radio frequency (RF) 20 kHz - 30 MHz..
oscillators
square wave, triangular wave and pulse trains as well
(iii) Video frequency (VF) d.c. - 5 MHz.
as amplitude modulation of the output signal.
oscillators
Although we speak of oscillators as 'generators' of (iv) High frequency (HF) 1.5 MHz - 30 MHz.
signals, it should be emphasized that no energy is oscillators
created, it is simply converted from a dc source into ac (v) Very high frequency 30 MHz - 300 MHz.
energy at some specified frequency. (VHF) oscillators

(683)
684 Electrical and Electronic Measurements and Instrumentation

Feedback oscillators are more commonly used. In general the operating conditions for regenerative
They use an active device such as an amplifier action or oscillations of feedback oscillators are :
whose output is feedback in phase to its input 1. The frequency of a sinusoidal feedback
(positive feedback) to cause regenerative action oscillator is determined by the condition that
resulting in oscillations. the loop phase shift is zero. Another way of
Negative resistance oscillators use an active device explaining it would be that the phase shift
with an inherent negative resistance charac­ between the amplifier input signal and the
teristics which provide its own regeneration and signal which is returned through amplifier
hence oscillations. and the feedback circuit must be equal to
zero.
22.3 FEEDBACK OSCILLATORS 2. The product of voltage of amplifier, G, and
A basic feedback circuit is shown in Fig. 22.1. the feedback factor H must be equal to or
greater than 1.
These two conditions for sustained oscillations
are called Barkhausen Criteria.
It is not necessary to supply an input signal to
initiate oscillators. A noise voltage or a transient is
sufficient to start oscillations. It is usually desirable for
Fig. 22.1 Basic feedback circuit the loop gain to be greater than unity to start
oscillations. However, if the loop gain is too large, the
Voltage gain of amplifier
output waveform will not be sinusoidal as the circuit
G=— ...(22.1) will operate under conditions of saturation in this
Ei case.
where E-] and Eo are phasors and G is a complex
22.4 TYPES OF FEEDBACK OSCILLATORS
function of frequency. A portion E of output voltage
Eo is feedback into the input. Feedback oscillators are classified into three
categories :
Feedback factor H = —— ...(22.2) (z) RC (resistance-capacitance) oscillators,
Eo
(zz) LC (Inductance-capacitance) oscillators, and
where H is a complex quantity.
(zzz) Crystal oscillators.
Now amplifier input voltage E-j is the sum of the
signal Ef and feedback voltage Ey. 22.4.1 RC Oscillators
RC oscillators may be one of the two following
Ei “ Ei+ E/ ...(22.3)
forms :
Amplifier output voltage (z) Wien's bridge oscillator, and
Eq = GE] = G(E; + Ez) = G(E. + HE0) (zz) Phase shift oscillator.
or E ...(22.4)
0 1-GH 1. Wien's Bridge Oscillator
.'. Gain with feedback The Wien's bridge RC oscillator has become the
G = E° - G standard circuit for variable frequency test circuits in
...(22.5)
f E; 1-GH the audio frequency range. This oscillator is simple
and stable in operation. Also it is far less cumbersome
Equation 22.5 indicates that if the complex quantity
than the LC oscillator.
called Ic'yp gain GH is positive, as it approaches unity
the gain increases without limit. Hence, if the value of A Wien's bridge oscillator is an audio oscillator is
GH = 1 + /0, the overall gain is infinite. Physically useful for testing equipment that operates in the
interpreted, this situation means that the signal input audio-frequency range. Such instruments always
voltage E, can be reduced to zero without affecting the produce a sine-wave signal, variable in both
output voltage. Thus the amplifier can provide an amplitude and frequency and usually provide a
output without an input and such a self-excited source square-wave output as well. The maximum amplitude
is termed as a Feedback Oscillator. of the output waveform is typically on the order of
Instruments for Generation of Waveforms 685

25 V whereas the range of frequencies covers at least The values of Rt and R2 are kept equal through a
the audio-frequency range from 20 Hz to 20 kHz. The mechanically operated switch.
most common output impedances for audio oscillators The first condition for oscillations requires that
are 75 Q and 600 Q. the phase shift between and E2 to be zero. This can
The two most common audio oscillator circuits only happen when the bridge is balanced and as per
are the Wien's bridge oscillator and the phase-shift the analysis given in Art. 16.8.1, the resonant
oscillator both of which employ RC feedback frequency f = -—as shown in Eqn. 22.10.
networks. The Wien's bridge offers some very
attractive features, including a straightforward Equation 22.10 indicates that bridge resonance is
design, a relatively pure sine-wave output and a very independent of the value of other two bridge arms i.e.
stable frequency. R3 and R4.
The Wien's bridge oscillator is essentially a The second condition for oscillations requires that
feedback amplifier in which the Wien's bridge serves the product of amplifier gain G, and the feedback
as the phase-shift network. The Wien's bridge is an a.c. factor H should be equal or greater than 1. As the
bridge the balance of which is achieved at one bridge is to be used as a feedback network for the
particular frequency. The basic Wien's bridge oscillator, voltage E2 should not be equal to zero in
oscillator is shown in Fig. 22.2. magnitude but the phase shift between Ej and E2
should be zero.
This means that the value of H should not be
equal to zero.
The impedances Z4 and Z2 have the same phase
angle at resonance since R3 and R4 are pure resistance.
At resonance, therefore :

wC
and , nr 1
Z, =---------------- = (1/-,- i)..—R
2 (1/R) + jcoC 2
Zo 1
Thus E =----- 2_=-E1 ...(22.11)
n Z1 + Z2 3 1 V 7

and E, = —E, ...(22.12)


‘ R3 + R4 1

(The above relationships indicate that both Efl and


Ej, are in phase with £2 at resonant frequency.)
As can be seen, the Wien's bridge oscillator
consists of a Wien's bridge and an operational At null point: E2 = Efl - Efc = 0.
amplifier represented by the triangular symbol. The Thus if R4 /(R3 + R4) = i or R3 = 2 R4, the output
conditions for balance are :
Z4Z4 = Z2Z3 ...(22.6) voltage E2 will be equal to zero. We do not want to
make E2 equal to zero and therefore, the ratio,
= +S ...(22.7)
R4(R3 + R4) should be smaller than - .
R4 r1 c2

and fn =--- , * ...(22.8) Let — =----- -— = - - — where a > 3.


E4 R3 + R4 3 a
E2 _ B?__ Eb _ Eff p 1
If Q = C2 = C and R} = ft, = R, Eqns. 22.7 and 22.8 Then E1 E1 Ej ^3 aj
simplify to
=2 ...(22.9) ...(22.13)
R4
At resonant frequency En I E1 = - and hence
C = —— Hz ...(22.10)
70 2nRC V ' H=l/a.
686 Electrical and Electronic Measurements and Instrumentation

Thus if the gain, G, of the amplifier is made equal distortion sinusoidal output. However, when using
to a, the condition GH - 1 is satisfied. transistors, a peak detector circuit is used to provide
It is clear from above that : negative feedback signal proportional to the output
(0 The frequency of oscillations is equal to the voltage. This is because the current drawn by an
resonant frequency^. incandescent lamp is too much of drain on the power
supplies and thus is incompatible with the transistors
(n) At any frequency other than resonant
and battery power sources. Also the use of an
frequency Efl is not in phase with and
incandescent lamp causes a thermal lag.
therefore E2 =Efl -E;, is not in phase with Er
Thus the condition for oscillations that E2 be Advantages
in phase E1 is satisfied only at resonant (i) The Wien bridge oscillator provides a stable
frequency. low distortion sinusoidal output voltage
(Hi) The relationship GH = 1 is satisfied only at over a wide range of frequency.
resonant frequency. (ii) The frequency range can be selected by using
simple decade resistance boxes.
Capacitors, C, are variable air capacitors mounted
(iii ) The frequency of oscillations in an RC
on the same shaft. Continuous variation of frequency
network is /0=l/2nRC. Thus unlike L-C
is accomplished by varying the value of C. Resistors R
networks where the resonant frequency
are provided with switches, so that different
varies inversely as square root of C, the
frequency ranges may be selected.
frequency of Wien bridge oscillator varies
In order that the amplitude of oscillations be held inversely as C itself. Thus frequency
steady over a wide range of frequencies, a negative variation greater than 10 : 1 is possible with
feedback is necessary. This negative feedback is single sweep of an air dielectric tuning
provided by resistors R] and R2. Resistance R2 is often capacitor.
a temperature sensitive resistor with a positive (iv ) With the addition of a power amplifier to
resistance temperature co-efficient. R2 is usually an isolate the oscillator from the load, the circuit
incandescent lamp operated at a temperature lower is used to provide test signals for a variety of
than its illumination level. Since the resistance of the applications.
lamp changes with amplitude of driving signal, thus
(n) The range of this oscillator is 2 Hz to
changing the voltage division of the bridge arms
100 kHz. The upper frequency limit is fixed
accordingly. Thus, as the amplitude of the oscillations
by the amplitude and the phase shift
increases, the resistance R2 increases. This reduces the characteristics of the amplifier. The lower
negative feedback which reduces the gain of the limit of frequency is fixed by tire practical
amplifier and thus amplitude of oscillations is limits on the size of range selecting resistors.
restored to normal.
The above statement may be elaborated further as Practical circuit of Wien bridge oscillator.
under : Figure 22.3 shows the circuit diagram of a two state
Wien bridge oscillator. Two stages are necessary in
The amplitude of oscillations depends upon how
order to obtain a phase shift of 0° as a single stage
much the value of GH is greater than unity. If the
amplifier shifts by 180°.
feedback factor H is constant, the amplitude is
determined by the value of gain, G of the amplifier. If
G increases, the amplitude increases till the increase is
limited by the non-linear characteristics of transistors.
If the output of the amplifier tends to increase, the
increased current through R2 increases its resistance.
This reduces the value of H and would then tend to
keep the value of product GH constant. Thus the
amplitude is regulated.
A vacuum tube Wien bridge oscillator depends
upon a temperature sensitive element R2. This allows
the operation of the amplifier within its linear region
of its transfer characteristics. Thus we get a low Fig. 22.3 Practical Wien bridge circuit.
Instruments for Generation of Waveforms 637

The four arms of the Wien bridge are : 1


Now
(/) R and C in series 2nRC
(ii) R and C in parallel Capacitance
(in) resistor R and C = -~—
(iv) resistor R^. 2^/0
=-------- - -1------------- ? =354 kHz
Qi and Q2 are two transistors and the positive
27tx30xl03xl5xl0-3
feedback is applied from the collector of Q2 through
the coupling capacitor C1 to top of the bridge circuit. 2. Phase Shift Oscillator
Example 22.1 Determine the frequency of oscillations When an audio-frequency (AF) oscillator is to be
shown in a Wien's bridge circuit of Fig. 22.4 ; where used over comparatively a limited frequency range, or
q = q = 6 kD, q = q = C =3 nF, on a specified frequency, simplified RC networks are
R3 = 12 kD, R4=6 kD usually employed. For example, Fig. 22.5 shows a
simple "phase shift oscillator", which can be used for
generation of sinusoidal voltages upto a frequency
range of several hundred kHz. The circuit consists of a
single transistor amplifier stage, followed by three
cascaded arrangements of resistor R and capacitor C to
provide the necessary positive feedback.

Fig. 22.4 Wien's bridge oscillator for Example 22.1.

Solution. The conditions of a wien bridge are


fulfilled with :
q/q=12/6=2.
Fig. 22.5 Phase shift oscillator.
With Ry = R, = R and q = C2 = C, the output The single transistor amplifier shifts the phase of
frequency is, any voltage by 180° which appears et its base. The RC
network is necessary to provide an additional phase
------------- 5----------- 5- = 8.84 kHz
2nRC 2ttx6x10*3x3x10 9 shift of exactly 180°. This additional phase shift is
obtained from the RC network only at a specific
Example 22.2 Design a Wien's bridge oscillator
frequency. Hence, at a frequency where the RC
around thefollowing specifications, f =15 kHz, Vcc =±10V,
network provides exact 180° phase shift, the total
phase shift from the base of transistor, around the
Solution. /K4 =100 Ijn = 100z 1 pA= lOOpA. circuit, back to the base is exactly 0°. Thus the feedback
If the peak value of the sinusoidal waveform is voltage is in phase with the voltage input to the
90% of Vcc, we have : amplifier at that frequency. Thus, if the amplification
0.9 Vrr 0.9x10 of the transistor is sufficiently large, the circuit will act
R, + R. =------- = 90 kQ. as an oscillator at that frequency.
3 4 * 100 XlO’6 100 xlO"6*
The phase shift network for the phase oscillator
For balance in a Wien's bridge, R^=2 R4.
is shown in Fig. 22.6. It consists of equal valued
2R4+R4 =90 kQ or R4 =30 kQ. capacitors and resistors connected in cascade. Each
and q = 60 kQ. stage provides a phase shift 60° with the total phase
It is generally convenient to have shift being 180°.
R.1 = qZ = R=30kQ
4
The phase shift oscillator is analysed by ignoring
or R = 30 kQ. any loading effects produced by the amplifier.
688 Electrical and Electronic Measurements and Instrumentation

The expression for feedback factor is : Solution. From Eqn. 22.16, frequency of
__________ 1_______ oscillations is :
5 ,F 1 6 1 1
/n = = 50 Hz
(coRC)2 l(coCR)3 wCR 276kRC 2^6nx 13 x 100 x IO"6

...(22.14)
Advantages and Disadvantages
(/) The circuit of a phase-shift oscillator is much
simpler than the Wien bridge circuit. This is because
this circuit dispenses with the negative feedback and
the stabilization arrangements.
(n) The phase shift oscillator may be used over a
wide frequency range, from a few Hz to several kHz.
The upper frequency range is limited since at high
frequencies, the impedance of phase shifting RC
network becomes very small, with the result the
amplifier is heavily loaded. To offset this, corrective
measures have to be taken which complicate the
circuit and thus the advantage gained on account of
simplicity of the circuit is lost.
(iii) The circuit is particularly suited for low
frequencies, say of the order of 1 Hz, since these
frequencies can be obtained easily from the circuit by
using large values of R and C which are commercially
available.
Fig. 22.6 Phase shift audio oscillator.
(iv) The phase shift oscillator has decidedly an
In case the phase shift of the feedback oscillator is advantage over L-C tuned oscillators in the low
180°, imaginary components of the Eqn. 22.14 should frequency range as in this frequency range the large
be zero. inductors required are cumbersome and
———5-————=0 ...(22.15) impracticable.
(coRC)3 co CR
(v) The frequency stability is not as good as that
This requires that the frequency of oscillations is, of Wien bridge oscillator. The distortion is greater and
co 1 changing of frequency is in convenient as each
Hz ...(22.16) capacitor is to be individually adjusted.
2 7t 2^6tiRC
22.4.2 L.C. Oscillators and R.F. Oscillators
and con = , 1 — rad
ai/ s -1 ...(22.17)
0 x6rc The first oscillators were the L.C. variety. These
oscillators are particularly suited for the radio
Substituting the value of coQ in Eqn. 22.14, we have, frequency (R.F.) range. There are many types of (L.C.)
1 oscillators and they use a conventional type of L.C.
H=
Vo 1-5/(1/6)4-7(676-6^6) tank (R.F.) circuit. Their principle of operation is
simple and almost similar for any of the many
...(22.18)
variations of the basic circuit. The opeation of the basic
or H= = —— = ...(22.19) circuit is as follows :
Vo 1-5x6 29
An L.C. oscillator uses a tank circuit which
^=-29^ ...(22.20) comprises of an inductor L in parallel with a capacitor
C. The parallel combination of L and C is excited into
which means that the gain of the amplifier must be at
oscillations and the output voltage of the tank circuit
least 29 if the circuit is to sustain oscillations.
is amplified by either a transistor or a vacuum tube
Example 22.3 Determine the frequency of oscillations amplifier. A fraction of the amplified voltage is fed
of a phase shift oscillator with a three section feedback back into the tank circuit by either an inductive or a
network consisting of 13 Q resistors and 100 pF capacitors. capacitive coupling. This is done in order to
Instruments for Generation of Waveforms 689

compensate for the power loss in the tank circuit. This decreases and therefore the induced voltage
"regenerative feedback" results in a constant decreases. Capacitor Cg/ therefore, discharges through
amplitude output voltage at the resonant frequency of resistance Rov. This makes the grid negative and
the tank circuit. The resonant frequency of the tank therefore a chain reaction is set off.
circuit is With the grid becoming negative, the plate
/0=l/2n7IC. current starts decreasing and as a result the magnetic
The Barkhausen criteria must be satisfied to field of L2 is wiped out. This induces a negative
produce sustained oscillations. voltage in coil Lp The capacitor, Cg is still discharging
and this drives the tube to a cutoff region and
It is evident from the expression for/,, that these
therefore the plate current becomes zero. The negative
circuits are particularly suited for radio frequencies
induced voltage across causes C\ to discharge and
since the value of inductance required to produce
then charge up again to the peak value of the negative
oscillations is quite small at these frequencies. The L-C
induced voltage.
oscillators can operate at very high frequencies, up to
several hundred MHz. The tube is cut-off and Q is charged with a
negative polarity at its top plate. During the next half
Armostrong Oscillator
cycle, the tank circuit takes out the tube from its
Figure 22.7 shows the basic circuit of an Armstrong cut-off region. Q starts to discharge through Llz and
oscillator. Inductor, Llz and capacitor, Clz form the tank the grid is raised in potential till the tube starts
circuit of the oscillator. Coil Ly is inductively coupled to conducting again. As soon as the tube conducts, the
a coil L2. When the plate supply voltage Ebb is first energy is transferred from the plate circuit to the tank
connected, current flows through the plate circuit. circuit. Capacitor, C\ charges up again to the peak
This current flows through coil and since L2 is value of the positive voltage. The entire cycle repeats
inductively coupled to coil Lp a voltage is induced in itself as described earlier. The output is taken from coil
the later. This voltage is in such a direction that grid of L3 which is inductively coupted to coil L^.
the tube is driven positive. This increases the plate
Hartley Oscillator
current at a faster rate and the induced voltage in coil
Li, increases further. Thus a high positive voltage is Figure 22.8 shows the transistorized version of a
built up across the tank circuit. Capacitor Clz charges Hartley oscillator. This circuit uses only one coil
with a positive polarity on its top plate. During this instead of two coils as Armstrong circuit. The coil has
process, since the grid is positive the grid current a tap which corresponds to the common a.c. ground of
charges the capacitor C$ to the peak value with a the Armstrong circuit. The variable (tuning) capacitor
polarity as shown. As the plate current approaches its Cj is now in parallel with the entire coil The
saturation point the rate of rise of plate current output is now available through an RC coupling
instead of an inductive coupling as in Armstrong
circuit.

Fig. 22.8 Hartley oscillator circuit.


Phase shift network consists of a tapped conductor
consisting of sections and L2 and an justable
capacitor Q to vary the frequency of oscillations.
Fig. 22.7 Armstrong oscillator circuit. The feedback factor is H = -1^ / L, ...(22.21)
690 Electrical and Electronic Measurements and Instrumentation

The negative sign means that there must be a 180 Solution. Frequency of oscillations
phase shift across the amplifier. This is accomplished
/ - r 1
by connecting the amplifier in the inverting configu­ 0 2 71^(1^ + fy)C1 2 71^280 xl0~6x 0.001
ration as shown in Fig. 22.9.
= 300 Hz.
The minimum gain of the amplifier is computed
using Eqn. 22.23 as,
G . = _k=™=_27
mm Lj 10

Using Eqn. 22.24 the value of the feedback resistor


Ifyis
Kf = Gminfy =27x 15 = 405 kQ

Colpitts Oscillator
The Colpitts circuit in its transistorized form is
shown in Fig. 22.11. It is, in fact, another form of basic
Armstrong circuit. The tank circuit consists of a fixed
inductor Lp and two variable capacitors Q and C2 in
series. This circuit differs from the Hartley's circuit
only in the form of tapping. In the Hartley's circuit, the

Fig. 22.9 Basic Hartley oscillator.

The circuit must satisfy the Barkhausen criteria


for sustained oscillations for which GH > 1 or
GH>1/H ...(22.22)
or G^-fy/^ ...(22.23)
Equation 22.23 states that the gain of the amplifier
must be greater than or equal to L2/fy. Using the
equation for the gain of an inverting amplifier
A= -Rf/Rj ...(22.24)
Example 22.4 Determine the frequency of oscillations
Frequency
and the minimum value of Rj to sustain oscillations in a tuning
Hanley oscillator shown in Fig. 22.10.
Fig. 22.11 Colpitts oscillator.

inductor is tapped while in the Colpitts circuit, the


tapping is from the capacitors. The capacitors Cj and
C2 are ganged. The amount of feedback depends upon
the relative values of Q and C2. The smaller the Q the
greater the feedback. As the tuning is varied, the
values of both capacitors increase or decrease
simultaneously, but the ratio of the two capacitances
remains the same.

22.4.3 Crystal Oscillators


In crystal oscillators, the usual electrical resonant
circuit is replaced by a mechanically vibrating crystal.
These crystals exhibit Piezo-electric properties, i.e.,
when mechanical stresses are applied on the opposite
Fig.22.10 Hartley oscillator circuit for Example 22.4.
Instruments for Generation of Waveforms 691

faces of the crystal, electrical charges appear at some similar to those of LC oscillators. Figure 22.13 shows
other faces and vice-versa. Thus when an alternating the circuit of a Pierce oscillator. This circuit is similar
voltage is applied to appropriate faces, mechanical to that of the Colpitts oscillator.
vibrations are produced at some other faces. Therefore
Advantages and Disadvantages
if the frequency of alternating voltage is equal to the
frequency of mechanical vibrations, the vibrations will (1) The frequency stability of the oscillations
be intense. obtained from a crystal vary and that too over a long
period of time.
Piezo-electric properties are exhibited by a
number of natural crystal substances like quartz, (2) The frequency of oscillations generated by
Rochelle salt, etc. Quartz is the normally used material other RF oscillators using electrical resonating (tank)
for crystal oscillators because of its frequency control circuits is effected considerably by changes in supply
in oscillators which comes from its permanence, low voltage, load and temperature. Crystal oscillators do
temperature co-efficient and high mechanical Q. not face these problems and thus their frequency
Quartz is preferred to Rochelle salt on account of the stability is high. Hence these oscillators are the
above-mentioned qualities even though Quartz has a standard means of maintaining frequency.
lesser Piezo-electric effect. (3) The vibrating crystal structure has a high
The electrical equivalent of a crystal is shown in value of Q. Values in excess of 20,000 are always to be
Fig. 22.12(b). The circuit has two resonant frequencies expected and values over 1,000,000 have been
obtained under special circumstances.
(z) Series resonant frequency = 1/2tiVlC
(4) Since the ratio of capacitance C/C' is very
(zz) Parallel resonant frequency
small, the coupling between crystal resonator and
f _ 1 ll+C/C external circuit is inherently small.
LC (5) The crystal oscillators have a very limited
tuning range (or not at all). The crystal oscillators are
used for frequencies greater than 100 kHz. Their use is
mainly at radio frequencies (RF).

22.5 BEAT FREQUENCY OSCILLATOR (B.F.O.)


Before the advent of the Wien bridge oscillator,
the beat frequency oscillator was the most commonly
oscillator to obtain variable frequency output in the
(«) W
audio frequency (AF) range. Now the use of B.F.O. has
Fig. 22.12 Crystal and its equivalent electric circuit. decreased. It is mostly used when it is desired to cover
a very large frequency range with a single dial rotation
It is apparent that /2 is greater than /p But the
(without the need for switching bands).
value of ratio C/C' is very small and hence the two
frequencies and /2 are close to each other. A beat frequency oscillator circuit is shown in
Fig. 22.14 in block diagram form. One of the oscillators
Because of the close analogy between a crystal
generates a fixed frequency and the other a variable
and an LC network, the crystal oscillator designs are
frequency. The outputs of these oscillators are fed to a
mixer. The frequency of output of the mixer is the
difference of two frequencies. Thus if the fixed

dial

Fig. 22.14 Beat frequency oscillator.


Fig. 22.13 Crystal oscillator circuit.
692 Electrical and Electronic Measurements and Instrumentation

frequency oscillator generates a frequency of 100 kHz difficulty of making their base regions and because of
and the variable frequency oscillator has a frequency relatively slow motion of minority carriers in semi­
range of 100 kHz to 120 kHz, the output frequency conductors. Frequencies of the order of 100 - 500 MHz
varies between 0 to 20 kHz. Therefore, frequency dial are the upper limit. The tunnel diode operates because
is marked 0 to 20 kHz (i.e., the difference of the two of majority carriers and hence is thus very fast. It is
frequencies). This then is an audio frequency (AF) possible to make tunnel diode oscillators which
output. This output is amplified by an AF amplifier. operate in the microwave frequency region. A simple
It is obvious that circuit of a B.F.O. is much more circuit shown in Fig. 22.15 can be made to oscillate if
complicated than that of a Wien bridge oscillator and the correct bias current is applied to the tunnel diode.
hence B.F.O. is becoming obsolete. The tunnel diode circuits may use negative feedback
for bias stability, just as feedback oscillators do.
22.6 NEGATIVE RESISTANCE OSCILLATORS
Many negative resistance elements are available
for use in negative-resistance oscillators. These are
tetrodes, tunnel diodes, neon tubes, and unijunction
transistors. There are two types of negative resistance
oscillators used in practice.
(i) Dynatron. Its action is based upon the negative
resistance region of the characteristics of a tetrode
coupled with an LC tank circuit.
(zz) Tunnel diode oscillator. This is solid state
equivalent of the dynatron with additional
advantages of very high speed operation. Therefore
this oscillator is very useful in the very high frequency
(VHF) and ultra high frequency (UHF) ranges.

Figure 22.15(a) shows the characteristics of a tunnel


diode. The characteristics are stable if driven by a
voltage source. The voltage current response can be
used to cause or sustain oscillations. The load lines as
shown are unstable ; that is there are two stable Fig. 22.15 Characteristics and circuit of a
intersections and one unstable intersection. If the tunnel diode oscillator.
tunnel diode is connected to an impedance which may
be represented by Z = R + jX, we find that the criteria 22.7 SQUARE-WAVE AND PULSE GENERATORS
for instability is : R + RD = 0. Thus the device Square-wave and pulse generators are often used
resistance, RD, must cancel out the resistance of the with an oscilloscope as the measuring device. The
remainder circuit, and the reactance at the frequency waveforms displayed either at the output and or at
of oscillations must be zero. In order to obtain some specific points in the system under test provide
sinusoidal oscillations, the above conditions must be qualitative as well as quantitative information about
satisfied. If the resistance of the device (tunnel diode) or device under test. The fundamental difference
were linear, there would be no limit to the amplitude between a square-wave generator and a pulse
of the oscillations. The amplitude of the oscillations generator is dependent upon duty cycle. The Duty
can be controlled, if the positive resistance line is Cycle is defined as the ratio of average value of the
almost tangent to the negative resistance line. The pule over one cycle and the peak value are inversely
negative resistance will then have an average value related to their time duration. Since the average value
over one complete cycle which is equal to positive and peak value are inversely related to their time
resistance. duration, the duty cycle can be defined in terms of
The tunnel diode, which has a negative resistance, pulse width and the period or pulse repetition time.
is very useful as it may operate at very high
pulse width
frequencies. It has been difficult to obtain high Duty cycle = £------- :-------- ...(22.25)
frequency operation of transistors because of the
Instruments for Generation of Waveforms 693

A square wave voltage is shown in Fig. 22.16(a) short duration pulses reduce power dessipation in the
T/2 component under test. For instance, measurements of
:. Duty cycle for a square wave = = 0.5 transistor gam can be made with pulses having short
duration which does not cause overheating of
or according to another definition
junctions.
_ average value
Square wave generators are used whenever low
peak value
frequency characteristics of a system are being
_ 1 / 2 peak value investigated. These applications include testing of
peak value audio-systems. Square wave pulses are also preferred
to short duration pulses if the transient response of a
system requires a comparatively high settling time.
Average value Peak
= 1/2 peak value value 22.7.1 Pulse Characteristics and
Terminology
T/2 The characteristics of a pulse are shown in
Period Fig. 22.17. The base Une is referred to the d.c. level and
is the line at which the pulse starts and finishes. The
(a) Square wave shift of this line from zero volts, or the expected value,
Pulse is called the base Une offset. The amplitude of the pulse is
width measured from the base line to the steady state pulse
value.
1. Pulse rise and fall time. The pulse rise time is the
time needed for the pulse to go from 10% to 90% of its
amplitude, and the fall time is the time for the trailing
edge to go from 90% to 10%. These times are also
called leading edge and trailing edge transition times.
2. Linearity. The linearity of the pulse is the
deviation of an edge, from the straight line drawn
(b) Pulse wave through the 10% and 90% points, expressed as a per
cent of the pulse amplitude.
Fig. 22.16 Square and Pulse waveforms.
3. Pulse preshoot. The pulse preshoot is the g 22
Thus square wave generators produce an output deviation prior to reaching the base line at the start of
voltage with equal on and off periods, their duty cycle the pulse. The overshoot is the maximum height
is 0.5 or 50%, as the frequency of oscillations is varied. immediately following the leading edge.
This is on account of the fact that whatever may be the 4. Ringing. It is the positive and negative peak
frequency, the positive and negative half cycles distortion, excluding overshoot.
always extend over half of the total period. 5. Settling time. It is the period needed for pulse
For a pulse the on and off periods can be varied. ringing to be within a specified percentage of the pulse
amplitude, measured from the 90% point of the
Duty factor for a pulse wave = y
leading edge.
According to the other definition. 6. Pulse droop or sag. It is the fall in pulse
amplitude with time. Pulse rounding is the curvature
Duty cycle for a pulse wave
of the pulse at the leading and trailing edges.
_ average value _t / T x peak value _
— — t / 1 j 7. Pulse ividth. The width of the pulse is measured,
peak value peak value in units of time, between the 50% points on the leading
The duty cycle of a pulse generator may vary and trailing edges. The pulse period is the time
(normally between 50% to 95%) depending upon the between equal points on the waveform.
duration of the on period. The very short duration 8. Pulse repetition rate. The pulse repetition rate is
pulses give a low duty cycle. The pulse generator a measure of how frequently a pulse occurs. It is equal
generally can supply more power during its on period to the reciprocal of the pulse period, and is measured
than a corresponding square wave generator can, in units of frequency.
694 Electrical and Electronic Measurements and Instrumentation

9. Duty cycle. The duty cycle of the pulse is the


ratio of its width to its period, usually expressed as a
percentage.
10. Pulse jitter. Pulse jitter is a measure of short
term instability of one event with respect to another,
for example, instability in the starting time, or the
pulse width, or the pulse amplitude. It is usually
expressed as a percentage of the main parameter.

22.8 PULSE AND SQUARE WAVE CIRCUITS


Sine wave
The pulse and square wave generating circuits are output
of two types : D.C. bias
(i) Passive circuits. These circuits shape an input 0V
waveform, usually a sine wave to give a
pulse output. Output
(n) Active circuits. These circuits include a pulse
blocking oscillator or a multivibrator.
The circuits are briefly described as below :
Fig. 22.18 Generating a pulse from a sine wave :
22.8.1 Pulse Shaping Circuit (a) circuit diagram, (b) waveforms.
Figure 22.18 illustrates a circuit in which a total resistance in series with capacitor C and the
sine-wave, from an oscillator, is converted into a pulse transformer has a step down ratio of 1 : N
train. The amplifier is saturated between two limits,
N+l
by the sine wave, and the sharpness of the pulse f— ...(22.27)
depends on the magnitude of the sine wave relative to
the d.c., bias, and the gain of the amplifier. This circuit +V
can also be used to 'clean up' a pulse input having
overshoots, undershoots, rings and so on.

22.8.2 Blocking Oscillator


The circuit, shown in Fig. 22.19 operates by
driving transistor Qj hard into and out of saturation,
so that the output is a series of pulses. The transistor
gives a 180° phase shift, and the transformer provides
a further 180° shift, so the circuit free runs and
maintains oscillations. R} adjusts the frequency of the
oscillator, which is given by Eqn. 22.27, where R is the Fig. 22.19 A blocking oscillator pulse generator.
Instruments for Generation of Waveforms 695

The width of the pulse is determined by the Qt This means that the collector voltage of Q}(ecf)
characteristics of the transformer, mainly the self drops more rapidly than the collector voltage of
inductance and self capacitance of the primary Q2(e.2). The decrease in exl is applied to R2 C2 network,
winding. because the charge on C2 cannot change
instantaneously, the full negative-going change
22.8.3 Multivibrators
appears across R2. This decreases the forward bias on
There are three types of multivibrators : bistables
Q2 which in turn decreases the collector current of
or flip flops, which provide an output pulse for every
Q2(zc2), and the collector voltage of Q2 rises. This rise
two input trigger signals; monostables which give a
in Q2 collector voltage is applied via the R1C1, network
timed pulse out for a trigger input; and astable or free
to the base of increasing its forward bias. Qj
running multivibrators, which give a string of pulses.
therefore conducts even more heavily and its collector
The period and pulse widths of these circuits can be
voltage drops still more rapidly. This negative-going
readily adjusted by timing resistors and capacitors.
They are usually made as integrated circuits, and are change is coupled to the base of Q2, further decreasing
widely used in industry. its collector current. The entire process is cumulative
until Q2 is entirely cut off and Q-j conducts heavily
Astable Multivibrator
(bottoms).
The astable or free running multivibrator is widely
With Q2 cut off, its collector voltage practically
used for the generation of pulses. It can be made to
equals the supply voltage, Vcc, and capacitor C]
produce either square wave or pulses depending on
charges rapidly to Vcc through the low-resistance
the choice of circuit components. A typical
path from emitter to base of the conducting transistor
free-running multivibrator is shown in Fig. 22.20.
Qp When the circuit action turns fully on, its collector
potential drops to approximately 0 V and since the charge
on C, cannot change instantaneously, the base of Q2 is
at least — Vcc potential, driving Q2 deep into cutoff.
The switching action now begins. C2 begins to
discharge exponentially through R2. When the charge
on C2 reaches 0 V, C2 attempts to charge up to the
value of + VBB, the base supply voltage. But this action
immediately places a forward bias on Q? and this
transistor starts to conduct. As soon as Q2 starts
conducting, its collector current causes a decrease in
collector voltage e^. This negative-going change is
coupled to the base of Q] which starts to conduct less,
Fig. 22.20 Astable or free-running multivibrator. i.e., it comes out of saturation. This cumulative action
repeats until Q1 finally cuts off and Q? conducts
Essentially, the circuit consists of a two-stage heavily. At this instant, the collector voltage of Qj
RC-coupIed amplifier, with the output of the second reaches its maximum value of Vcc. Capacitor C2
stage (Q?) coupled back to the input of the first stage charges to the full value of Vcc, and a full cycle of
(QP via capacitor Cr Similarly, the output of Qj is operation has been completed.
coupled via C2 to the input of Q2. Since the coupling The waveforms appearing at the base and the
between the two transistors is taken from the collector of each transistor are the result of a
collectors, the circuit is known as a collector-coupled symmetrical or balanced operation : The time constants
astable multivibrator. and R2C2, the transistors themselves, and the
The usual qualitative analysis of the circuit supply voltages are all identical. The conducting and
proceeds as follows : When the power is first applied non-conducting periods are therefore of almost the
to the circuit, both transistors start conducting. same duration. The waveforms for each of the two
Because of small differences in their operating transistors are given in the waveform diagram of
characteristics, one of the transistors will conduct Fig. 22.21.
slightly more than the other. This starts a series of Assume that at time t=l, transistor Qj is fully
events. Assume that Qj initially conducts more than turned on and transistor Q2 is cut off. This makes the
696 Electrical and Electronic Measurements and Instrumentation

collector voltage ec] of Qj a minimum (practically 0 V) starts conducting again. Obviously, one complete
and the collector voltage ec2 of Q2 a maximum (Vcc). cycle of operation, from time t =1 to time f=3,
Capacitor Q is charging through the emitter-to-base depends on the time required for the base voltage of
resistance of (ff toward the supply voltage VqC and the cutoff transistor to reach the forward bias value.
reaches its full charge rapidly (low emitter-to-base This time depends on two things : the magnitude of
resistance). Since ecl is 0 V, capacitor C2 begins to the reverse bias (-V^c) an<^ ^e time constant of the
charge exponentially through R2 toward the base capacitor charging circuit involved, namely, RpCp or
supply voltage VBB with a time constant equal to R2C2. R2C2.
The analytical evaluation of circuit operation
proceeds as follows :
During its non-conducting period, the collector
voltage of Qi equals
eci = vcc<1-e’1/’3) -<22-28)
where r3 = R3C2-
When Q-, switches on, its collector voltage is at
ground potential and the base voltage of Q2 becomes
— Vcc with respect to ground. The subsequent rise in
Q2 base voltage, through the R2C2 charging circuit, is
described by
+ Vcc )- Vcc ...(22.29)
where r2 = R2C2.
Q2 remains cut off until eb2 reaches the value of 0 V
(by good approximation) and the off time interval T2 of
1 23456789
Q2 can be determined by setting eb2 in Eqn. 22.29 to
Time
zero and solving for t, so that
Fig. 22.21 Waveforms for the astable multivibrator
of Fig. 22.20. o = (VBB + Vcc )(1 )- vcc ...(22.30)
Since the early part of the exponential charging
curve is almost linear, the increase in Q2 base voltage and T2 = t2 In ...(22.31)
(eb2) is indicated by a linear slope on the graph of V +V
VBB + vcc J

Fig. 22.21. Similarly, when Q2 is off and Q2 is bottomed, the


At time t = 2, ec2 reaches a value of approximately collector voltage of Q2 can be described by
0 V placing a forward bias on the base of Q2 which ecl = VCcO-f‘l'"> ...(22.32)
then starts to conduct. Within a very short time, the
where r4 = R4Cr
collector current of Q2 reaches its maximum and
collector voltage ec2 drops to 0 V. When Q2 starts to When now Q2 is switched on, its collector voltage
draw current, the base of Qj becomes negative and Q2 drops to 0 V and the base voltage of Q1 is given by
is quickly driven into cutoff. Its collector voltage, ecl,
reaches the Vcc value and collector current icl en = (VbB + Vcc )(1 ) ...(22.33)
becomes zero. Within a very small fraction of the total where Tj = ^Cj.
Q2 conducting time, capacitor C2 is hilly charged to Solving for the off time interval of Qa by setting
Vqq through the low-resistance emitter-to-base path of ebl in Eqn. 22.33 to zero, we obtain.
q2-
Between times t =2 and t =3, transistor (ff is cut 0 = (VDB + VCcXl-e_'/'1)-Vcc ...(22.34)
off, and its collector current and voltage remain
constant. Similarly, the collector voltage and current and 7] = Tj In ...(22.35)
for Q2 remain constant. Only capacitor Q is charging V +V
VBB+ VCC 7
and the base voltage ebl of 'Qi is rising exponentially
The total period of oscillation is given by,
toward VBB. At time t=3, the base voltage of Qj
exceeds the cutoff value (approximately 0 V) and t=t1+t2 ...(22.36)
Instruments for Generation of Waveforms 697

600 Q
output

50 Q
output

Trigger
output

output

Fig. 22.22 Block diagram of a pulse generator.

In the case of symmetrical operation, when time The basic generating loop, which is redrawn for
constants RjCj and R2C2 are equal, the waveform is a greater clarity in Fig. 22.23 consists of two current
symmetrical square wave. By making time constant sources, the ramp capacitor, the Schmitt trigger circuit,
RjCj larger than time constant R2C2, the output and the current-switching circuit (indicated by a
waveform becomes a pulse train because the off time
of Qj will be larger than the off time of Q2.

22.8.4 Laboratory Square-wave and


Pulse Generator
The block diagram of a typical general-purpose
generator providing negative pulses of variable
frequency, duty cycle, and amplitude is given in
Fig. 22.22. The frequency range of the instrument is
covered in seven decade steps from 1 Hz to 10 MHz,
with a linearly calibrated dial for continuous
adjustment on all ranges. The duty cycle can be varied Fig. 22.23 Simplified current source operation.
from 25 per cent to 75 per cent. Two independent
outputs are available: a 50 Q source that supplies simple switch). The two current sources provide a
pulses with rise-and fall times of 5 ns at 5 V peak constant current for charging and discharging the
amplitude, and a 600 Q source that supplies pulses ramp capacitor. The ratio of these two currents is
with rise-and fall times of 70 ns at 30 V peak determined by the setting of the symmetry control
amplitude. The instrument can be operated as a which then determines the duty cycle of the output
free-running generator or it can be synchronized with waveform. The frequency dial controls the sum of the
external signals. Trigger output pulses for two currents from the current sources by applying
synchronization of external circuits are also available. appropriate control voltages to the bases of the current
698 Electrical and Electronic Measurements and Instrumentation

control transistors in the current generators. The size triggering the Schmitt circuit by an external
of the ramp capacitor is selected by the multiplier synchronization pulse.
switch. These last two controls provide decade The unit is powered by an internal supply that
switching and vernier control of the frequency of the provides regulated voltages for all stages of the
output. instrument.
The upper current source, supplying a constant
current to the ramp capacitor charges this capacitor at 22.9 PULSE GENERATOR CIRCUIT
a constant rate, and the ramp voltage increases Pulse generators are instruments that produce a
linearly. When the positive slope of the ramp voltage rectangular waveform similar to a square wave but
reaches the upper limit set by internal circuit with a different duty cycle.
components, the Schmitt trigger (a bistable There are many applications in systems and
multivibrator) changes state. The trigger circuit output component testing in which pulse generators, used in
goes negative, reversing the condition of the current conjunction with an oscilloscope, are very useful. The
control switch, and the capacitor starts discharging. most basic pulse generator is the astable multivibrator
The discharge rate is linear, controlled by the lower which generates symmetrical square waves with
current source. When the negative ramp reaches a variable pulse repetition frequency (PRF) and
predetermined lower level, the Schmitt trigger provides for the relative lengths of the positive and
switches back to its original state. This now provides a negative excursions of the waveform to be adjusted. A
positive trigger circuit output that reverses the schematic for a basic pulse generator, consisting of an
astable multivibrator and a monostable multivibrator,
condition of the current switch again, cutting off the
is shown in Fig. 22.24. The output of the astable
lower current source and switching on the upper
multivibrator is a square wave. The square wave will
current source. One cycle of operation has now been
be symmetrical if both capacitors, C, and both
completed. The entire process, of course, is repetitive
resistors, R, have the same values.
and the Schmitt trigger circuit provides negative
pulses at a continuous rate. The frequency, which is given as

The output of the Schmitt circuit is passed to the


trigger output circuit and to the 50 Q and 600 Q
amplifiers. The trigger output circuit differentiates the can be changed, while maintaining the symmetry of
square-wave output from the Schmitt trigger, inverts the waveform, by changing both resistors or both
the resulting pulse, and provides a positive triggering capacitors. The duty cycle of the square wave can be
pulse. The 50 Q amplifier is provided with an output varied by changing the value of one capacitor or one
attenuator to allow a vernier control of the signal resistor. Common practice would be to use several
output voltage. In addition to its free-running mode of switch-selectable capacitors to provide different
operation, the generator can be synchronized or ranges of frequency and to provide for vernier control
locked into an external signal. This is accomplished by by making one of the resistors variable.

Fig. 22.24 Basic pulse generator circuit.


Instruments for Generation of Waveforms 699

The purpose of the "one-short," or monostable Solution. The frequency of the square wave

multivibrator, is to provide a short-duration output generated by the astable multivibrator is computed as


pulse each time a pulse is applied to its input. The _ 0.693
monostable multivibrator has one stable state and one * ~ RC
unstable state. The circuit operates in its stable state = ______ °^3______ = 924 Hz
until an input pulse drives it into its unstable state, 75 x 103 x 0.01 x 10-6
thus producing an output pulse. The duration of the
The pulse width is computed as
output pulse is computed as
t = 0.69 R3C3
R] + R3
t = R3C3 In ...(22.38) = 0.69x 50xl03x0.005xlO-6
-
= 0.173 ms
If resistors R] and R2 are equal in value, as they To compute the duty cycle, we must first compute
frequently are, then the expression simplifies to the pulse period which is
t = 0.693 R3C3 ...(22.39)
T = - =—-— = 1.08 ms
The circuit C2R^ is called a differentiator. Its f 924 Hz
purpose is to provide a short-duration "spike" to
We can now compute the duty cycle as
trigger the monostable multivibrator. The purpose of
diode D3 is to bypass to ground the negative spike _ , Pulse width
Duty cycle =-----------------x 100%
from the differentiator. The waveforms at points A, B, Pulse period
0,173 x!0~3
’ 1.08 xlO'3
= 16%
Some of the typical specifications of a
laboratory-quality pulse generator given below :
A Pulse repetition rate : 0.3 Hz to 20 MHz, five
ranges.
A Pulse width : 25 nsec to 3 sec in five ranges.
A Maximum duty cycle : greater than 80%
from 0.3 Hz to 1 MHz; greater than 50% form
1 to 10 MHz.
Fig. 22.24.
A Output impedance: 50 Q.
Example 22..A Determine the frequency, pulse width, A Amplitude continuously variable from 0.03
and duty cycle of the circuit shown in Fig- 22.25. to 10 V.

Fig. 22.25 Pulse generator circuit for Example 22.6.


700 Electrical and Electronic Measurements and Instrumentation

22.10 TRIANGULAR WAVESHAPE which simplifies to


GENERATOR ^=VCCR1 -(22-42)
The circuit diagram of a triangular waveshape
generator is shown in Fig. 22.26. At the first stage, From Eqn. 22.42 we can determine the maximum
which is a voltage comparator generates a square amplitude of the triangular output, VQ2, which is
wave output. expressed as :
2^
The output of Aj is driven to saturation; therefore, V«1 = VCC~ -<22«)
the square wave is either at + Vcc or • The second «2
stage, A2, is an integrator which generates a triangular
When the output voltage V02 reaches the
output. The square wave is applied to a square-to-sine
amplitude given by Eqn. 22.43, the output of the
wave converter that filters out the odd harmonics
making up the square wave while passing on only the comparator changes states and the triangular wave
fundamental sine wave. begins to decrease linearly. Since the output is
symmetrical about 0 V. Equation 22.43 also expresses
The operation of the circuit can be analyzed by
the minimum value of VQ2 at which switching occurs.
starring at the output of the comparator, which is at
The wave-forms at V *, VQ1, and V02 are shown in
either + Vcc or -V^c. Consider V01, to be at -V^c. The
voltage VOi will remain at -V^c until the voltage at the Fig. 22.27 for the situation in which R1 = R?.
inverting input of exceeds the voltage at the The frequency of the circuit is controlled by the
noninverting input, which in this case is at zero volt. RC time constant of the integrator. To obtain an
The noninverting input voltage, Vx, is due, in part, to expression for the frequency, we begin with the
the voltage V01, and, in part, to the voltage V02, expression relating capacitor current, charge and time
according to the expression of change:
v = -V __ ____ + v ___ ^2__ ...(22.40) q = ict ...(22.44)
x cc R^ + R, 02 Rl + R2
The rate of charge of the capacitor is
the output VQ1 changes states when Vx =0; therefore,
dq = icdt ...(22.45)
we can say
which can be written as :
0 = -^cc ...(22.41) ic = dq/dt ...(22.46)

flfl. 22.26 Circuit for a basic function generator.


Instruments for Generation of Waveforms 701

The development of Eqn. 22.54 began with Eqn.


22.44, which allows us to compute the charge on a
capacitor after a period of time t. Equation 22.44 is
valid only if the initial charge and, therefore, the initial
voltages on the capacitor are zero. Therefore, the time
t in Eqn. 22.54 is the time for the capacitor to charge
from 0 V until switching occurs, which is at one-four th
cycle as shown in Fig. 22.27. Since t = T/4, Eqn. 22.54
becomes
/
T = 4RC ...(22.55)
I

The frequency, which is the reciprocal of the


period, is now expressed as
Fig. 22.27 Waveforms for the function generator
of Example 22.6. f = —( ...(22.56)
J 4RC^R1 J
As the capacitor charges, the relationship between
charge, capacitance, and voltage across the capacitor Example 22.6 Compute the frequency and the peak
plates is amplitude of the triangular output of the circuit shown in
Fig. 22.28.
q=CVQ2 ...(22.47)
Substituting Eqn. 22.47 into Eqn. 22.46 yields
i=C^^ ...(22.48)
c dt
Since the input resistance of the operational
amplifier is very high, the current through resistor R is
approximately equal to the charging current of the
capacitor. Therefore, we can write
zR = C^^ ...(22.49)

In addition, since the voltage gain of the


operational amplifier is very high, the voltage at the
input to the amplifier is very nearly zero.
Therefore,
Fig. 22.28 Function generator for Example 22.7.

The output frequency can be computed using


Substituting Eqn. 22.50 into Eqn. 22.49, we obtain, Eqn. 22.56 as :
...(22.51) -MAI
4RC1R1>
Integrating both sides of Eqn. 22.51, we obtain
_________ _1__________ (100xl03>
Vm = — [ Vni dt = . (22.52) 4(500xl03)(0.004xl0-6)X^ 60xlO3 ,
02 RCJ 01 RC(t)

Substituting Eqn. 22.43 into Eqn. 22.52, gives = 208 Hz


V ^ = Yoit ...(22.53) The amplitude of the triangular waveform can be
cc r2 rc computed from Eqn. 22.43 as

Since VQ1 = V£c, Eqn. 22.53 simplifies to : 60xlQ3


^02 = 15x
D 100 xlO3
t=RC-± ...(22.54)
^2 = 9 V.
702 Electrical and Electronic Measurements and Instrumentation

22.11 FUNCTION GENERATOR 100 kHz. The frequency control network is governed
A function generator is a versatile instrument that by the frequency dial on the front panel of the
delivers a choice of different waveforms whose instrument or by an externally applied control
frequencies are adjustable over a wide range. The voltage. The frequency control voltage regulates two
most common output waveforms are the sine, current sources.
triangular, square, and sawtooth waves. The The upper current source supplies a constant
frequencies of these waveforms may be adjusted from current to the triangle integrator whose output voltage
a fraction of a hertz to several hundred kilohertz. increases linearly with time. The output voltage is
The various outputs of the generator may be given by the well-known relationship.
available at the same time. For instance, by providing
...(22.57)
a square wave for linearity measurements in an audio
system, a simultaneous sawtooth output may be used An increase or a decrease in the current supplied
to drive the horizontal deflection amplifier of an by the upper current source increases or decreases the
oscilloscope providing a visual display of the slope of the output voltage. The voltage comparator
measurement results. The capability of the function multivibrator changes state at a predetermined level
generator to phase lock to an external signal source is
on the positive slope of the integrator's output
another useful feature. One function generator may be
voltage. This change of state cuts off the upper current
used to phase lock a second function generator, and
supply to the integrator and switches on the lower
the two output signals can be displaced in phase by an
current supply.
adjustable amount. In addition, one generator may be
phase locked to a harmonic of the sine wave of another The loiver current source supplies a reverse
generator. By adjusting the phase and the amplitude current to the integrator so that its output decreases
of the harmonics, almost any waveform may be linearly with time. When the output voltage reaches a
generated by the summation of the fundamental predetermined level on the negative slope of the
frequency generated by the one function generator output waveform.
and the harmonic generated by the other function
generator. The function generator can also be phase 22.12 SIGNAL GENERATING INSTRUMENTS
locked to a frequency standard, and all its output The previous sections of this chapter have
waveforms are then generated with the frequency considered the circuits which go to produce the sine,
accuracy and stability of the standard source. pulse and square waveforms used in signal generating
The function generator can supply output equipment. In the remainder of this chapter the
waveforms at very low frequencies. Since the low instruments are introduced. These are categorised as
frequency of a simple RC oscillator is limited, a signal generators, swept frequency generators,
different approach is used in the function generator of synthesizers, pulse generators; and function
Fig. 22.29. This instrument delivers sine, triangular, generators, although the dividing lines between these
and square waves with a frequency range of 0.01 Hz to groups are often blurred.

Fig. 22.29 Basic element of a function generator.


Instruments for Generation of Waveforms /03

22.12.1 Signal Generators output is a difference frequency (/2_/i) so a slight


These instruments usually produce a fixed change in one of the frequencies, when they are close
frequency sine wave, whose output can be frequency to each other, can give a wide drift in the output. The
or amplitude modulated by another signal. The output also has a considerable amount of noise and
instruments cover a frequency range of 0.001 Hz-50 GHz, spurious signals.
but not from the same device. Figure 22.30 shows a signal
generator circuit. Frequency modulation is achieved by
varying the voltage across a variable capacitance
diode in the tuning circuit of the oscillator. This gives a
system with low output distortion, for modulation
depths below 1% of the carrier frequency. Above this
modulation level the waveform applied to the tuning
diode needs to be deliberately distorted, in order to
compensate for its non-linear characteristics. During Fig. 22.31 A heterodyne oscillator.
frequency modulation manual or automatic methods
Multiplier and divider techniques can be used to
may be used to keep the amplitude of the output
extend the frequency range available from a single
constant.
instrument. In the multiplier generator, shown in
Amplitude modulation is most conveniently done
Fig. 22.32, the output from the fixed frequency
by varying the supply voltage to the oscillator. This
oscillator is fed through a series of tuned multipliers,
method is, however only suitable for small
each having a non-linear amplifier which produces
modulation depths, up to about 50%. It also gives
harmonics. The output from each stage is fed to a
phase modulation due to the effect on the components
tuned filter which selects the high frequency output.
used within the oscillator circuit. Feedback can be
Frequency modulation may be applied to the master
used to reduce output distortion, as shown in
oscillator, and amplitude modulation is achieved by
Fig. 22.30, by detecting the output to obtain the
varying the d.c. supply to the last multiplier stage. The
modulation envelope, comparing this with the
disadvantage of the multiplier generator is that it
amplitude modulation input, and then amplifying
produces a large amount of spurious signal around
and feeding back the difference as the modulation
the desired frequency.
signal. This technique is known as envelope feedback.
The divider signal generator uses a high
The output amplifier shown in Fig. 22.30 provides
frequency master oscillator, whose output is divided
the required signal level, and buffers the oscillator
down by a series of electronic stages. The outputs
from changes in the load impedance. The attenuator is from the divider stages are square waves, which need
needed to give low level output signals. to be filtered to produce sine waveforms. Frequency
The heterodyne principle is sometimes used, as in modulation is applied to the main oscillator, if
Fig. 22.31, to give a continuously variable, wide required, and amplitude modulation is obtained by
frequency range output from a single instrument. The diode modulators in the output amplifier. The divider
signal quality is good, but the frequency stability is generator does not produce any spurious signal at
poor, especially at low frequencies. This is because the frequencies below the required frequency.

Fig. 22.30 A signal generator using envelope feedback for amplitude modulation.
704 Electrical and Electronic Measurements and Instrumentation

Input for
amplitude
modulation

Optional Optional
Output
output output

Fig. 22.32 A multiplier frequency generator.

22.12.2 Swept Frequency Generator Two modes are used to set the swept frequency
A swept frequency generator, or sweeper, is a range :
special type of signal generator in which the output (i) The stop start or (f2~ /T) mode. In this mode
frequency is cyclically swept through a range of the stop and start frequencies are set from
frequencies. The instrument may have a display built the front panel, and the instrument sweeps
into it, to show the variation of amplitude with between these limits. This mode is used for
frequency, or it may provide signals for use with an wide sweep widths.
external oscilloscope. (ii) Delta frequency mode (bf). In this mode the
Figure 22.33 shows the block diagram of a typical centre frequency, and the maximum
sweep generator. The time base is usually adjustable, excursion about this frequency, are set from .
to give output sweep times in the range from 10 ms to the front panel. It is used for narrow sweep
greater than 100 s. It is also often possible to control widths.
the sweep manually from the front panel of the The frequency range of the swept frequency
instrument. The time base is frequently a triangular or generator usually extends over three bands, 0.001 Hz -
sawtooth waveform. 100 kHz (low frequency to audio), 100 kHz-1500 MHz

Manual control

fig. 22.33 Block diagram of a sweep generator using a heterodyne frequency generator.
Instruments for Generation of Waveforms

Manual control

Detector input
(from device
under test)

Fig. 22.34 Block diagram of a sweep generator using a heterodyne frequency generator.

(r.f. range), and 1-200 GHz (microwave range). Three The graticule on the display is not accurate
approaches may be used to cover a wide band of enough to be able to interpret the frequency being
swept frequencies from a single instrument. displayed, and some swept frequency generators have
(z) Manually switching between different the facility for putting a series of accurately known
frequency oscillators. The problem occurs when the frequency markers onto the screen, to act as a
frequency range needed overlaps two bands. reference. One method of generating the marker
(zz) Stacked switching. In this the bands are frequencies, as shown in Fig. 22.33, is to pass the
automatically selected by electronic switches, so that output from a stable crystal oscillator through a
one can sweep the whole instrument range as one harmonic generator, which gives a series of narrow
continuous band. pulses, spaced at harmonic intervals. These are mixed
with a sample of the r.f. output from the sweeper. The
(zzz) Heterodyne control. In this method,
mixer will produce a series of low difference
illustrated in Fig. 22.34, two high frequency signals are
mixed to give a lower difference frequency output, as frequency output bursts, called birdies, as the sweeper
one continuous band. If, for example, generator 1 can frequency approaches and passes through each
be varied over a frequency range 3-5 GHz (i.e., a harmonic frequency. The birdies are shaped and
change of 2 : 3), and generator 2 has a fixed frequency amplified, and then combined with the signal received
output of 3 GHz. then the output can vary from 1 to back from the device under test (via a detector), before
2000 MHz (i.e., a change of 2000 : 1). Since the tuning being fed to the display. This gives a composite
range needed for this wide output variation small, the picture of the frequency response characteristic, of the
instrument has good linearity. However, a non-linear device being tested, and the frequency calibration
mixer will produce many other frequencies in markers. An alternative method, of generating a
addition to the required one. marker, is to pass a single fixed frequency signal to the
mixer, to give a spot of accurately known frequency
Output level control is used in a swept frequency on the screen.
generator to keep the output amplitude to the-value
set on the front dial. Usually an r.f. detector monitors 22.12.3 Sweeper Errors
the r.f. amplitude of the output. This is compared with Several errors occur in swept frequency generator
a signal corresponding to the required amplitude, and measurements, due to imperfections in the output of
the error is fed back to an electronic attenuator circuit, the generator. Any harmonics produced in the
to keep the output constant. frequency source will be swept along with the
706 Electrical and Electronic Measurements and Instrumentation

fundamental, but at a faster rate. For example, the nth generators have many uses, such as testing of radio
harmonics will be swept at n times the rate of the and radar for signal reception in the presence of noise,
fundamental. These harmonics may be within the pass and intermodulation and crosstalk tests in communi­
band of the device being tested, for part of the sweep, cation systems.
and out of the pass band for the rest. This would give a Two parameters are of interest in random noise
change in the response of the device, which could be generators :
falsely interpreted as a change in its characteristics.
The Noise-Power Spectrum
For example Fig. 22.35, illustrates the effect of the nth
harmonic on a test to measure the flatness response of Usually one is only interested in noise which has a
a low pass filter. A bump is generated when the nth bandwidth greater than that of the circuit under test.
harmonic is present, and several overlapping Within this frequency range the noise power spectral
harmonics will produce a series of bumps. Generally density can follow one of three curves, as shown in
only the 2nd and 3rd harmonics are of concern, since Fig. 22.36. White noise has a constant power spectral
the higher harmonics have a much lower amplitude. density from 20 Hz to 25 kHz. Pink noise has higher
amplitudes at lower frequencies (its amplitude varies
inversely as the square root of frequency) and is given
its name because of the similarity of its spectrum with
that of red light. It is used in bandwidth analysis. The
spectrum of USASI noise is approximately equal to the
energy distribution of speech and music frequencies,
and it is used for testing audio systems.

Fig. 22.35 Effect of sweep generator harmonics on a


low pass filter response measurement.

The accuracy of the output, and the change in its


amplitude with frequency, is another important
consideration. The amplitude varies with the sweep
rate, since the amplitude feedback correction circuits
have less time to work at faster sweep rate. The effect
of this error can be minimised by sweeping at the
lowest rate acceptable for the device being measured,
and by making readings with respect to the markers, Probability Distribution Function (PDF)
since these are also affected by a change in amplitude. The power spectrum of a noise signal generator
The output from a sweeper is not linear, which gives its frequency content, but does not characterise
means that the spacing between markers is unequal, its waveshape; this is done by its p.d.f. The p.d.f. is the
so that one cannot easily estimate a value at a statistical calculation of the portion of time which the
frequency between two markers. The error is usually signal spends at the various amplitudes. Usually, for
naturally occurring random signals, this function
very small, but if an exact reading is required then the
follows a Gaussian curve.
sweeper is set to its manual mode. Its frequency can
then be changed until the desired frequency appears Random noise can be generated by using a gas
on the display, and this can be read with an external discharge tube or a zener diode generator. A
accurate frequency meter. semiconductor noise diode gives an output frequency
in the band from 50 kHz to 200 kHz. This can be
22.12.4 Random Noise Generators amplified and modulated to produce an audio
A random noise generator produces a signal frequency band. The signal is then passed through a
output whose instantaneous amplitude varies at filter to get one of the spectra shown in Fig. 22.36.
random, and has no periodic frequency component. The disadvantage of the random noise generation
Generators are available which cover frequency bands method is that sometimes the total output is subject to
from low audio to microwave. Random noise long term variations, giving an unpredictable power
Instruments for Generation of Waveforms 707

spectrum. An alternative method is to generate a signal proportional to ± f2. The low pass filter
pseudo-random noise, such as by clocking a long shift selects the signal, which is then amplified and fed to
register having several feedback loops. The noise the voltage controlled oscillator (VCO). The output
pattern will now be repeatable, but within a cycle the from the VCO is fed to a programmable divider, and
signal behaves as truly random noise. The output from then back to the phase detector. The system drives f2
the shift register is fed through a low pass filter to give towards/j until they are equal, and at this stage there
the pseudo-random analogue noise signal. is still a phase difference between the two frequencies,
which produces sufficient d.c. signal to keep/2-locked
22.13 FREQUENCY SYNTHESIZER to f[. The output frequency is Nf and this can be
Frequency generators are of two types : changed by varying N, usually through a keyboard
from the front panel of the instrument.
(i) Free running, in which the output can be
tuned continuously, over a frequency range, The switching speed, between frequency setting,
by mechanical or electronic methods. These of the frequency generator depends on the time
are the types described so far, and they have needed for/2 to lock to/j and on the setting time. For
good overall performance, although their large frequency differences the lock time can be long.
frequency, accuracy and stability are It is minimised by setting the VCO close to its final
relatively poor. value, usually under microprocessor control, and then
letting the phase lock loop pull it to its exact value. The
(ii) Synthesizer, which has its output derived
setting time is the time, after lock, for the output to
from a fixed frequency, highly stable
reach within a given range of the final output setting:
oscillator, and covers the range in a series of
steps. it is typically a few milliseconds.
The problem, with the basic system shown in
Synthesizers are of two types, direct and indirect. Fig. 22.37 is that for a wide frequency range, with
The direct synthesizer uses a stable crystal oscillator, small incremental steps, a low reference frequency,
followed by a series of harmonic multipliers and and a VCO which can cover a wide range, are required.
mixers, to provide the range of different output
The programmable divider also needs a large
frequencies. The indirect frequency synthesizer uses a
divider ratio, and this produces noise within the loop
phase locked loop to give an output which is a fraction bandwidth. A multiloop frequency synthesizer, as
of that of a stable crystal oscillator. The synthesizer shown in Fig. 22.38, overcomes these disadvantages.
technique provides a generator with excellent The low frequency synthesizer has a small range, and
frequency accuracy and stability, but careful design is can operate with small steps, whilst the high
needed to overcome the problem of spurious signals frequency synthesizer operates over a wide range
in the output. with wide steps. Combining these two signals, in a
Figure 22.37 shows the basic indirect frequency mixer, gives a system which can work over a wide
synthesizer. The output from the phase detector is a frequency range, with small steps.

Select
setting

Fig. 22.37 An indirect frequency synthesizer.


708 Electrical and Electronic Measurements and Instrumentation

Fig. 22.38 An indirect multiloop frequency synthesizer.

22.14 PROGRAMMABLE DECADE


FREQUENCY SYNTHESIER
Figure 22.39 shows an alternative system, which is called a decade synthesier as the fixed dividers have a ratio
of 10. The range of ratios needed from each programmable divider is much less. The fixed divider frequency
synthesier uses a high reference frequency allowing a wide loop bandwidth, and thereby giving a faster lock
time and better noise performance commercial synthesiers are available which cover frequency ranges from
10 kHz to 3000 MHz and from 10 MHz to 30 GHz and with modulation capabilities for use a signal generator
from the same instrument, and by their flexibility. The three basic types of outputs produced are sine, square and
triangular wave, over a frequency range of 1-50 MHz, although some function generators also have the
capability of producing pulses, the performance of the output from a function generator is usually lower than
that from a dedicated instrument.
Select

pig. 22.39 A fixed divider (decadic) indirect frequency synthesizer.


Instruments for Generation of Waveforms 709

Review Questions
1. What are signal sources? What are the desirable 11. Explain the principle of working of a negative
characteristics of a signal? Give an overview of resistance oscillator and describe the frequency
different signal sources used. range over which it is used.
2. Describe the meaning of an oscillator. Describe the 12. Define the duty cycle for a pulse wave. Describe
classification of oscillators the characteristics and terminology associated
(zj depending upon the principle they operate with pulse waves. 1
(if) depending upon the frequency range used by 13. Distinguish between the active and passive circuits
them. used for production pulse and square waveshapes.
3. Describe the Barkhausen criteria for production of Describe a passive pulse shaping circuit.
oscillations. Describe how the Barkhausen criteria 14. Describe in details the circuit and working of an
is met within Wien's bridge oscillator. astable multivibrator.
15. Describe the circuit diagram and working of a
4. Describe the principle of working of a Wien's bridge
laboratory type square wave and pulse generator.
oscillator. Derive the conditions for its balance.
Explain the working of a current source.
5. Describe the practical circuit for a Wien's bridge
16. Explain with a neat circuit diagram how a
oscillator and list its advantages and disadvantages.
triangular waveshape is produced using a square
6. Explain the circuit of a R.C. phase shift oscillators.
wave.
Describe how Barkhausen criteria is satisfied in this
17. Describe the circuit of a square wave generator
oscillator. Give its advantages and disadvantages.
which generates square, triangular and sine
7. Prove that in a R.C. shift oscillator the gain of the waveshapes.
amplifier should be at least 29 to sustain oscillations.
18. Describe the following :
8. Describe the circuit of a Hartley oscillator for
(z) A signal generator using envelop feedback for
production of radio frequencies. Derive the
amplitude modulation.
condition for production of oscillations.
(zz) A heterodyne oscillator.
9. Explain the circuit of a Colpitts' oscillator for
19. Describe the working of a sweep frequency
production of radio frequencies. Compare its
Operation with that of Hartley's oscillator. generator. What are the sweeper errors ?

JO. Describe the working of a crystal oscillator. List its 20. What is a frequency synthesizer? Describe its
circuit details.
advantages and disadvantages.

Unsolved Problems
1. Determine the gain with feedback A for the
amplifier in Fig. 22.40, for sustained oscillations.
[Ans. 19.96]

2. Determine the values of Cj and C2, that will cause


the circuit in Fig. 22.41 to oscillate at 12 kHz.
Assume C, = Q [Ans. 1.32 nF]
Fig. 22.41 Circuit for Problem 2.
710 Electrical and Electronic Measurements and Instrumentation

3. Determine the frequency of oscillations of a RC phase shift oscillator with a three section feedback network
consisting of 20 Q resistors and 1.5 nF capacitors. [Ans. 2.16 MHz]

4. Determine the value of L2 in the circuit shown in Fig. 22.42 if the frequency of oscillations is to be 100 kHz.
[Ans. 153 pH]

5. Determine the minimum value of 1^ in Fig. 22.42 to sustain oscillations if - 125 pH. [Ans. 83.3 kQ]

Fig. 22.42 Circuit for Problem 4. Fig- 22.43 Circuit for Problem 6.

6. Determine the frequency of the triangular output of the circuit shown in Fig. 22.43. [Ans. 11.72 kHz]
7. Determine the peak to peak amplitude of the output signal for the circuit shown in Fig. 22.43. [Ans. 8 V]
8. Determine the frequency and duty cycle of the astable multivibrator output in the circuit of Fig. 22.44
9. Determine the pulse width of the output pulse VQ in the circuit of Fig- 22.44 [Ans. 69 ps]

Fig. 22.44 Circuit for Problem 8.

10. Design a Wien bridge oscillator around the following specifications :


fQ = 10 kHz Vcc = 115 V, ;in = 1HA Ir4 +
(Refer to Fig- 22-4)
[Ans. R, =90 kQ, R4 = 45 kQ, c = 35-4 pk]
Instruments for Generation of Waveforms 711

Objective Type Questions


Tick (/) the most appropriate answer The frequency of oscillations is :
(a) 8.84 kHz
1. The desirable characteristics of a signal source are:
(b) 4.42 kHz
(a) the frequency of the signal should be well
known and stable (c) 8.84 MHz

(b) the amplitude of a signal source should be (d) No oscillations occur as Barkhausen criteria is
controllable from very small to relatively large not satisfied.
values 6. In an RC phase shift oscillator having three stages
(c) the output signal should be free from distortion which have equivalued resistors and capacitors,
the amplifier, in order to satisfy, Barkhausen
(d) all the above.
criterion should have a minimum gain of
2. In signal generators
(a) 6
(a) energy is created
(b) 29
(b) energy is generated
(c)31
(c) energy is converted from a simple d.c. source
(d) none of the above.
into a.c. energy at some specific frequency
7. In a Hartley oscillator, the inductor is divided into
(d) all the above.
two parts. The inductor which is on the input of the
3. A video frequency oscillator has a frequency range
amplifier has a value of 20 pH while that connected
of:
to the output side of the amplifier has an
(a) 20 Hz to 20 kHz (b) 20 kHz to 30 MHz inductance of 300 pH If the resistance connected to
(c) d.c. to 5 MHz (d) 2 MHz to 30 MHz. the input of the amplifier is 15 kQ, what should be
4. Feedback oscillators have a closed loop gain of the value of resistance in the feedback path of the
amplifier ?
G G
W 1-GH (i?) 1 + GH (a) 300 kQ

G H
(b) 225 kQ
(c)l±GH (d)l+GH (c) 1 kQ
(d) 15 kQ.
where G and H are respectively the gains of
forward and feedback paths. 8. The pulse rise time is defined as the time taken by
the pulse
5. A Wien's bridge oscillator circuit is shown in
Fig. 22.45. (a) to go from 10% to 90% of its amplitude
(b) to go from 0% to 100% of its amplitude
(c) to go from 0% to 90% of its amplitude
(d) to go from 10% to 100% of its amplitude.
9. The duty cycle of a pulse shown in Fig. 22.46 is

Fig. 22.45 Fig. of Q. 5.


Fig. 22.46 Fig. of Q. 9.
The values of components (fl) 33% (b) 25%
K = 6kQ, C = 3nF, Kg=6kQ, R4 = 12kQ. (c) 300% (d) 75%.
712 Electrical
and Electronic Measurements and Instrumentation

10. An astable multivibrator fisced frequency output of 3 GHz, the output


(«) gives one output pulse for every two input frequency varies from
pulses (a) 0 to 2 GHz
(b) gives a timed output pulse for a trigger input (b) 6 GHz to 8 GHz
(c) gives a train of output pulses for a trigger input (c) 0 to 3 GHz
(J) gives four output pulses for a single input (d) 0 to 5 GHz.
pulse. 14. A random noise generator produces a signal
11. An astable multivibrator uses a resistance of (a) whose amplitude varies randomly
100 kfl and capacitance 0.01 pF. The frequency of
(b) which has no periodic frequency
square waves generated by it is
(c) has an unpredictable power spectrum
(a) 924 Hz (b) .593 Hz
(d) all the above.
(c) 693 Hz (d) 110 Hz
15. A voltage controlled oscillator (Vco) is an instrument
12. A triangular waveshape is obtained :
(a) whose frequency is dependent upon the ampli­
(a) by integrating a square wave
tude of input signal
(b) by differentiating a in wave
(b) whose frequency is independent of the ampli­
(c) by differentiating a square wave
tude of the input signal
(J) by integrating a sine wave
(c) whose frequency is dependent upon the
13. In a sweep frequency generator, two oscillators, frequency of the input signal
one with frequency range of 3 GHz to 5 GHz is
(d) none of the above.
heterodyned with a second oscillator having a

1. (d) 2. (c) 3. (c) 4. (fl) 5. (d) 6. (b) 7. (b) 8. (a) 9. (b) 10. (c)
11. (c) 12. (fl) 13. (a) 14. (fl) 15. (a)
Signal Analysers I
I
f
I
'>

23.1 INTRODUCTION complex waveform can be done by electrical means


The analysis of electrical signals is used in many using a band pass filter network to single out the
applications. The different instruments which are various harmonic components. Networks of these
used for signal analysis are wave analysers, distortion types pass a narrow band of frequency and provide a
analysers, spectrum analysers, audio analysers and high degree of attenuation to all other frequencies.
modulation analysers. A wave analyser, in fact, is an instrument
All signal analysis instruments measure the basic designed to measure relative amplitudes of single fre­
frequency properties of a signal, but they use different quency components in a complex waveform. Basically,
techniques to do so. A spectrum analyser sweeps the the instrument acts as a frequency selective voltmeter
signal frequency band and displays a plot of which is tuned to the frequency of one signal while
amplitude versus frequency. It has an operating range rejecting all other signal components. The desired
of about 0.02 Hz-250 GHz. A wave analyser is a frequency is selected by a frequency calibrated dial to
voltmeter which can be accurately tuned to measure the point of maximum amplitude. The amplitude is
the amplitude of a single frequency, within a band of indicated either by a suitable voltmeter or a CRO.
about 10 Hz-40 MHz. There are two types of wave analysers, depending
Distortion analysers operate over a range of upon the frequency ranges used,
5 Hz-1 MHz and give a measure of the energy present (?) Frequency Selective wave analyser, and
in a signal outside a specified frequency band. They (n) Heterodyne wave analyser.
therefore tune out the fundamental signal and give an
indication of the harmonics. An audio analyser is 23.2.1 Frequency Selective Wave Analyser
similar to a distortion analyser but can measure This wave analyser is used for measurements in
additional functions, such as noise. Modulation the audio frequency range (i.e., from 20 Hz to 20 kHz).
analysers tune to the required signal and recover the The analyser has a filter section with a very narrow
whole amplitude modulated (a.m.), frequency passband which can be tuned to the frequency of
modulated (f.m.) or phase modulation (p.m.) envelope interest. The block diagram of this wave analyser is
for display or analysis. shown in Fig. 23.1(a).
The waveform to be analysed in terms of its
23.2 WAVE ANALYSERS separate frequency components is applied to an input
It is well known that any periodic waveform can attenuator that is set by the meter range switch on the
be represented as the sum of a d.c. component and a front panel. A driver amplifier feeds the attenuated
series of sinusoidal harmonics. Analysis of a wave­ waveform to a high-Q active filter. The filter consists
form consists of determination of the values of of a cascaded arrangement of RC resonant sections
amplitude, frequency, and sometime phase angle of and filter amplifiers. The passband of the total filter
the harmonic components. Graphical and mathe­ section is covered in decade steps over the entire
matical methods may be used for the purpose but audio range by switching capacitors in the RC
these methods are quite laborious. The analysis of a sections. Close-tolerance polystyrene capacitors are
(713)
714 Electrical and Electronic Measurements and Instrumentation

Variable frequency high-Q active filter


R's varied with frequency. Control
C's varied with frequency multiplier

Normalized frequency
(b) Attenuation characteristics

Fig. 23.1 Audio-range wave analyser and Characteristics of the active filter.

generally used for selecting the frequency ranges. 23.2.2 Heterodyne Wave Analyser
Precision potentiometers are used to tune the filter For frequency measurements in the megahertz
to any desired frequency within the selection pass­ range, a heterodyne wave analyser construction is
band. used, as shown in Fig. 23.2. The input signal is fed
A final amplifier stage supplies the selected signal through an attenuator and amplifier before being
to the meter circuit and to an untuned buffer mixed with a local oscillator. The frequency of this
amplifier. The buffer amplifier can be used to drive a oscillator is adjusted to give a fixed frequency output
recorder or an electronic counter. The meter is driven which is in the passband of the i.f. amplifier. This
by an average-type detector and usually has several signal is then mixed with a second crystal controlled
voltage ranges as well as a decibel scale. oscillator, whose frequency is such that the output
The bandwidth of the instrument is very narrow, from the mixer is centred on zero frequency. The
typically about 1 per cent of the selected frequency. subsequent active filter has a controllable bandwidth,
Figure 23.1(b) shows a typical attenuation curve of a and passes the selected component of the frequency to
wave analyzer. the indicating meter.
Signal Analysers 715

Frequency
control

Input
signal

Bandwidth
control

Fig. 23.2 Block diagram of a heterodyne wave analyser.

Good frequency stability in a wave analyser is however, the output waveform is not an exact replica
obtained by using frequency synthesisers, which have of the input waveform because of various types of
high accuracy and resolution, or by automatic distortions that may occur.
frequency control (AFC). In an AFC system the local Distortion may be a result of the inherent
oscillator locks to the signal, and so eliminates the drift non-linear characteristics of different components
between them. used in an electronic circuit. Nonlinear behaviour of
circuit elements introduces harmonics in the output
23.2.3 Applications of Wave Analysers
waveform and the resultant distortion is often referred
Wave analysers have very important applications to as harmonic distortion (HD).
in the following fields :
Types of Distortion
(z) Electrical measurements Distortion is caused by many devices and
(zz) Sound measurements and components which form an electronic circuit. In this
(zzz) Vibration measurements section the different types of distortions caused by
amplifiers are considered. The various types of
The wave analysers are applied industrially in the
distortions which occur are :
field of reduction of sound and vibrations generated
by rotating electrical machines and apparatus. The (z) Frequency distortion. This type of dis­
tortion occurs because the amplification
source of noise and vibrations is first identified by
factor of the amplifier is different for
wave analysers before it can be reduced or eliminated
different frequencies.
A fine spectrum anylsis with a wave analyser shows
(zz) Phase distortion. This distortion occurs on
various discrete frequencies and resonances that can
account of the energy storage elements in the
be related to the motion of machines. Once, these
system which cause the output signal to be
sources of sound and vibrations are detected with the
displaced in phase with the input signal. If
help of wave analysers, ways and means can be found signals of all frequencies are displaced by the
to eliminate them. same amount, the phase shift distortion
would not be noticed. However, in actual
23.3 HARMONIC DISTORTION ANALYSERS
practice, signals at different frequencies are
The application of a sinusoidal input signal to an shifted in phase by different angles and
electronic device, such as an amplifier should result in therefore the phase shift distortion becomes
generation of a sinusoidal output waveform. Generally, noticeable.
716 Electrical and Electronic Measurements and Instrumentation

(nz) Amplitude distortion. Harmonic distortion Percentage harmonic distortion


occurs due to the fact that the amplifier = ^D2+ D2+ D2+... x 100 ...(23.4)
generates harmonics of the fundamental of
the input signal. Harmonics always give rise
to amplitude distortion, for example, when xlOO ...(23.5)
an amplifier is overdriven and clips the
input signal. In a measurement system noise is read in addition
(m) Intermodulation distortion. This type of to harmonics, and the total waveform, consisting of
distortion occurs as a consequence of the harmonics, noise and fundamental, is measured
interaction or heterodyning of two freque­ instead of the fundamental alone. Therefore the
ncies, giving an output which is sum or measured value of total harmonic distortion (THDM)
difference of the two original frequencies. is given by Eqn. 23.6.
(v) Cross-over distortion. This type of E(H)2+(N)^
distortion occurs in push-pull amplifiers on {Z(F)2 + (H)2 + (N)2}1/2
account of incorrect boas levels as shown in
where H = Harmonics; N = Noise ; F = Fundamental
Fig. 23.3.
Equation 23.6 leads to below 1/2% error, due to
Crossover
/ \ distortion / \ the approximation for values of THD below 10%.
Figure 23.4 shows a harmonic distortion analyser
which is used to measure THD. The signal source has
very low distortion and this can be checked by reading
its output distortion by connecting directly into the
analyser. The signal from the source is fed into the
Fig. 23.3 Illustration of crossover distortion. amplifer under test. This generates harmonics and the
original fundamental frequency. The fundamental
23.4 TOTAL HARMONIC DISTORTION frequency is removed by the notch filter.
A non-linear system produces harmonics of an In the manual system of Fig. 23.4(a) the switch Sw
input sine wave, the harmonics consisting of sine is first placed in position 1 and the total content of
waves with frequencies which are multiples of the fundamental and harmonics (Er) is measured. Then
fundamental of the input signal. Total harmonic
distortion (THD) is measured in terms of the harmonic
content of the wave, as given by Eqn. 23.1.
Tun [^(Harmonics)2]172
ItlU =---------------------------------- ...(23.1)
Fundamental
A measure of distortion represented by a
particular harmonic is simply the ratio of amplitude of
harmonic to that of the fundamental Harmonic
distortion. (HD) is then represented by,
E~ e_ e.
D2 = -2-z D, =^, D.=-±
2 £ 3 p 4 E
^1 ci ^1

When Dn (n =2,3,4,...) distortion of nth harmonic


and En represents the amplitude of nth harmonic.
is the amplitude of fundamental.
Total harmonic distortion (THD) is defined as :
THD = /d| + D2 + D2 +... ...(23.2)

Je2 + e2 + e2+...
= 5----- 4----- ...(23.3) Fig. 23.4 Simplified block diagrams of fundamental
£i suppression harmonic distortion analysers ;
(a) manual reading, (b) ratio reading.
Signal Analysers 717

the switch is moved to position 2 to measure just the two signal source (/j and /2) frequencies as 7 kHz and
harmonics EH. The value of THD is then found from 60 Hz, having amplitudes in the ratio 1: 4 respectively.
Eqn. 23.6 as in Eqn. 23.7. The lower frequency has a much higher amplitude
since a low degree of non-linearity is expected. In
THD xlOO ...(23.7)
Ej. Europe the values of and f2 are defined by DIN
45403 as 8 kHz and 250 Hz again having an, amplitude
The meter can be calibrated by putting the switch ratio of 1 : 4.
in position 1 and adjusting the reading for full scale
The output from the amplifier under test is fed
deflection. With the switch in position 2 the meter
into a high pass filter, giving a waveform at point A
reading is now proportional to THD. Figure 23.4(1?)
which consists of the high frequency carrier and is low
shows an alternative arrangement, where the values
frequency modulation as in Fig. 23.6. The waveform
of Et and EH are read simultaneously and their ratio
can be observed on an oscilloscope and inter­
calculated and displayed as THD on the indicator. For
modulation distortion (IMD) calculated from (Eqns.
good accuracy the notch filter must have excellent
23.8 or 23.9)
rejection and high pass characteristics. It should
attenuate the fundamental by 100 db or more and the IMD = AmPlitude of modulation( ) * 1QQ
harmonics by less than 1 db. The filter also needs to be Amplitude of carrier (Ac)
tuned accurately to the fundamental of the signal ...(23.8)
source. This is difficult to achieve manually and most
= $-^xl00 ...(23.9)
distortion analysers do this automatically. A common
P
form of notch filter is a Wien bridge. This balances at
one frequency only and at this frequency the output
voltage at the bridge null detector is minimum.

23.5 INTERMODULATION DISTORTION


When a high frequency (fj signal and a low
frequency (f2) signal are mixed in a linear circuit the
output contains the two frequencies only. When
mixed in a non-linear circuit, for example, an amplifier
having distortion, modulation occurs. The output now
contains the original frequencies (^ and /2) and also
the sum and difference frequencies (fA +f2) and
*n addition to several harmonics and their
sum and difference frequencies.
Figure 23.5 shows an instrument for measuring
intermodulation distortion. The SMPTE (Society of Fig. 23.6 Waveform output from an intermodulation
Motion Picture and Television Engineers) defines the distortion meter.

Fig. 23.5 Simplified block diagram of an intermodulation distortion meter.


Electrical and Electronic Measurements and Instrumentation

Fig. 23.7

The analyser shown in Fig. 23.5 is capable of 23.7 SPECTRUM ANALYSERS


calculating IMD directly and displaying its value on Spectrum analysis is defined as the study of
an indicator. The r.m.s. value of the waveform at A is energy distribution across the frequency spectrum of a
measured by r.m.s detector 2. The waveform at A is given electrical signal. The study gives valuable
also demodulated by a detector and the high frequency information about bandwidth, effects of different
components removed by the low pass filter, leaving types of modulation and spurious signal generation.
the low frequency distortion components. This is
The knowledge of the above quantities and pheno­
measured by r.m.s. detector 1 and the ratio is taken of
mena are useful in the design and testing of radio
the outputs from detectors 1 and 2 before being
frequency (RF) and pulse circuitry.
displayed on the indicator.
The spectrum analysis is divided into two major
A special case of intermodulation distortion is
categories on account of instrumentation limitations
difference frequency distortion, also called CCIF
and capabilities. They are :
intermodulation distortion. Figure 23.7 shows the
cons- traction of a difference frequency analyser. A (z) Audio frequency (AF) analysis, and
low pass filter is used after the amplifier under test so (zz) Radio frequency (RF) spectrum analysis.
that the meter is sensitive to the low (difference)
The RF spectrum analysis covers a frequency
frequencies only.
range of 10 MHz to 40 GHz, and hence is more
23.6 TRANSIENT INTERMODULATION DISTORTION important, because it includes the vast majority of
communication, navigation, radar, and industrial
Transient intermodulation distortion occurs
instrumentation frequency bands.
because an amplifier is not able to respond rapidly to
changing inputs. It can be tested by applying a square The spectrum analysers are sophisticated instru­
wave signal with a superimposed high frequency sine ments which are capable of portraying graphically the
wave, to the input of the amplifier, as in Fig. 23.8(a) amplitude as a function of frequency in a portion of RF
and observing the output on an oscilloscope. If spectrum. These instruments find wide applications
transient intermodulation distortion is present then for measurement of attenuation, FM deviation, and
the output will have an initial portion of the high frequency in pulse studies.
frequency signal missing as at Fig. 23.8(b), and the 23.7.1 Basic Spectrum Analyser
value of T can be noted.
The basic spectrum analyser is designed to
represent graphically, a plot of amplitude versus
frequency of a selected portion of the frequency
spectrum under study. The modem spectrum
analyser basically consists of a narrow band
superheterodyne receiver and a CRO. The receiver is
electronically tuned by varying the frequency of the
(a) Input signal (b) Output signal
local oscillator. A simplifed block diagram of a
Fig. 23.8 Waveforms to measure transient swept frequency spectrum analyser is shown in
intermodulation distortion. Fig. 23.9.
Signal Analysers 719

Fig. 23.9 Basic swept receiver spectrum analyser.

The circuit incorporates a sawtooth generator


which supplies a ramp voltage to the frequency
control element of the voltage tuned local oscillator.
The local oscillator then sweeps through its frequency
band at a recurring linear rate. The same sawtooth
voltage is simultaneously applied to the horizontal
Fig. 23.10 A three-dimensional presentation of ampli­
plates of the CRO. The RF signal to be tested is applied tude, frequency, and time (a) The addition of a funda­
to the input of the mixer stage. mental and its second harmonic, (b) View seen in the
The sawtooth generator makes the local oscillator Time-Amplitude plane. On an oscilloscope, only the
composite \ + 2f\ would be seen, (c) View seen in the
sweep through its frequency band to beat with the input
Frequency-Amplitude plane. The components of the
signal to produce the desired intermediate frequency composite signal are clearly seen.
(IF). An IF component is produced only when the
corresponding component is present in the RF input 23.7.3 Spectra of Different Signals
signal. The resulting IF signals are amplified, and then Let us consider some of the commonly used
detected. After that they are applied to the vertical signals in order to illustrate the spectra which are
deflection plates of the CRO, thereby producing a displayed on the CRO when they are applied to the
display of amplitude versus frequency on the screen. spectrum Analyser.
23.7.2 Spectral Displays 1. Continuous wave (CW) signals. When a
continuous wave (CW) input signal is slowly swept
Normally a CRO is used for display of electrical
through by the spectrum analyser's local oscillator, the
signals with respect to time, with the X-axis of CRO
response displayed on the screen is a plot of the IF
calibrated to read directly the rise time, pulse width,
amplifier pass band. Since the CW signal has energy at
and the repetition rate. Such measurements are said
only one frequency and therefore the display on the
to be in Time Domain. However, in the case of screen is a single spike. This occurs in case the total RF
spectrum analysers, the signals are broken down into sweep width or Spectrum Width is wide as compared
their individual frequency components and displayed to the IF band width in the analyser.
along X-axis of the CRO which is calibrated in terms of
2. Amplitude modulation. When a continuous
frequency. Therefore, the signal amplitude is displayed
wave signal of frequency, is amplitude modulated
versus frequency. These measurements are then said
by an input signal of frequency, f, two side-band
to be in the Frequency Domain. frequencies of fc + fs and fc - fs are produced. The
A three dimensional representation of a funda­ display on the spectrum analyser screen is a signal of
mental frequency,^ and its second harmonic, f2 -2 fv
is shown in Fig. 23.10. The two signals and the sum
instantaneous values, as displayed on a CRO screen
are shown in Fig. 23.10(b). The two signals are shown
in Fig. 23.10 (c) on amplitude-frequency plane and are
portrayed on the CRO as two components of the
composite signal, as the window of the spectrum
analyser sweeps across the frequency range of the
Fig. 23.11 A single-tone amplitude modulation.
signal. (a) Time-amplitude plot, (b) Frequency-amplitude plot.
720 Electrical and Electronic Measurements and Instrumentation

frequency with two side-band frequencies whose generally referred to as spectral lines on the analyser
magnitude relative to the carrier frequency depends display. There will be twice as many sidebands or
upon the percentage of modulation. This is illustrated spectral lines as there are harmonic frequencies
in Fig 23.11. contained in the modulating pulse. Figure 23.13(d)
If the frequency, the spectrum width, and the shows the spectral plot which results from rectangular
vertical response of the analyser are calibrated, the pulse modulation of a carrier. The individual lines
display on the CRO screen provides the following represent the modulation product of the carrier and
information in numerical terms, (fl) carrier frequency, the modulating pulse frequency with its harmonics.
(b) modulation frequency, (c) modulation percentage, Therefore, the lines will be spaced in frequency by an
(d) non-linear modulation, (e) incidental frequency
modulation, (/) Spurious signal location and strength.
3. Frequency modulation. If a CW signal fc is
frequency modulated at a rate f, it will produce an
infinite number of sidebands. These are located at
intervals of fc + nfr, where n = an integer, 1, 2, 3... In
Time —>
practice, only the sidebands containing significant
(«)
power are usually considered. A frequency
modulated display is shown in Fig. 23.12.

Frequency, j —>

Fig. 23.12 Amplitude spectrum of single-tone (b)


frequency modulation.

4. Pulse modulation. An idealized rectangular


waveform, with zero rise time and no overshoot is
shown in Fig 23.13(a). This pulse is shown in the time
domain, but when its frequency spectrum is to be
analyzed, it must be broken down into its individual
frequency components. This is shown in Fig. 23.13(b),
where a constant voltage, a fundamental frequency,
and its third harmonic are added algebraically to form
a wave which eventually becomes a square wave as
more odd harmonics are added in phase with the
fundamental. The pulse becomes perfectly rectangular
if an infinitely large number of odd harmonics are
added. A spectral plot, in the frequency domain,
would have the form given in Fig. 23.13(c), where the
amplitude and phases of an infinite number of
harmonics are plotted, resulting in a smooth envelope
as shown.
When this pulse is used for amplitude modulating Fig. 23.13 Pulse modulation : (a) Periodic rectangular
a carrier, the sums and differences of the carrier and pulse train, (b) Addition of a fundamental cosine wave
and its harmonics to form rectangular pulses, (c) Spec­
all harmonic components contained in the pulse are
trum of a perfectly rectangular pulse. Amplitudes and
produced. The harmonic frequencies therefore phases of an infinite number of harmonics are plotted
produce multiple sidebands, in exactly the same resulting in smooth envelope as shown (d) Resultant
manner that the modulating signal in amplitude spectrum of a carrier amplitude, modulated with a
modulation does. These multiple sidebands are rectangular pulse.
Signal Analysers 721

amount equal to the pulse repetition rate of the


original pulse waveform. The main lobe in the centre
and the side lobes are shown as groups of spectral
lines extending above and below the base line. For a
perfectly, rectangular pulse, the number of side lobes
is infinite. The main lobe contains the carrier frequency,
represented by the longest line in the centre.
There are a number of other useful applications of
this instrument. A few typical examples are cited :
Fig. 23.15 Illustration of standing wave
1. Pulse-width and repetition rate measure­ ratio (SWR = U> / 14)
ments.
2. Tuning a parametric amplifier. In most systems some of the forward wave will be
3. FM deviation measurement. absorbed by the load, for example, in the case of an
4. RF interference testing. antenna some of the forward energy is radiated into
space. Therefore the reflected wave has a lower
5. Antenna pattern measurements.
amplitude than the forward wave, resulting in a
23.8 STANDING WAVE RATIO standing wave similar to that in Fig. 23.15. The
When a transmission line is terminated by an standing wave ratio (SWR) is then given by the ratio of
impedance different from its characteristic impedance the maximum and minimum amplitudes of this wave.
some of the forward signal wave is reflected back. This Both voltage and current ratios can be used and these
reflected wave mixes with the forward wave and the are referred to as VSWR and ISWR respectively and
resultant amplitude at any point is the algebraic sum are given by Eqns 23.10 and 23.11.
of the amplitudes of the two waves, as in Fig. 23.14.
VSWR = ...(23.10)
Amplitude Forward Reflected
wave ( wave ISWR = i ...(23.11)
Reflection
j point
' Distance The standing wave ratio is also given by
(Eqns. 23.12 and 23.13) using impedance, power, or
forward and reflected voltage as the parameters.
Node Antinode
SWR = 7 (for
' ZCr > Z,
L7
) ...(23.12)
Fig. 23.14 Illustration of standing wave.

The nodes and antinodes do not move relative to


or SWR = — (for ZL >ZC) ...(23.13)
the transmission line i.e., they are stationary and the
waves are called standing waves.
where Zc is the characteristic impedance of the
An important consideration in transmission line
transmission line and ZL is the characteristic impe­
and antenna design is the standing wave ratio (SWR) dance of the load.
and this is illustrated using Fig. 23.15 which
represents the standing wave on a line. If the line is 1 + (P./Pr)1/2
SWR =—V R F' ...(23.14)
open circuited or short circuited (load impedance 1-(PR/PF)1/2
being infinite or zero respectively), then all the
where Pp is the forward power from the source to
forward wave is totally reflected back to the source
the load and PR is the power reflected back from the
and V1 is zero. For an open circuit line the antinodes
load.
are located at half wavelengths along the line, starting
back from the open circuit point. For a short circuited 1+VVF
SWR = ...(23.15)
line the nodes are located at half wavelengths, starting 1-VR/VF
back from the short circuit point. Therefore although
open and short circuited lines give identical standing where VF is the voltage component of the forward
wave patterns, they are shifted relative to the wave, and VR is the voltage component of the reflected
positions of the nodes and antinodes. wave.
Electrical and Electronic Measurements and Instrumentation

When the load impedance equals the value of resistor Rx is chosen depending on the
characteristic impedance of the transmission line, the antenna type, i.e., a 50 ohm or a 75 ohm system. When
forward wave is totally absorbed in the load, and there making measurements switch Sw is first set to
is no reflected or standing wave. Now equals V2 in
Fig. 23.15 and the system is ideal having a standing
wave ratio defined as 1 : 1.

23.8.1 Measurement of Standing Wave Ratio


The standing wave ratio (SWR) is one of the most
fundamental parameters as it checks for mismatch
between the transmitter, transmission line and
antenna. The SWR varies with frequency, since the
impedance of the source, line and load are unlikely
to change by the same amount with frequency ; so
the SWR should always be measured at several
frequencies.
Any of the equations (23.10 to 23.15) can be used
to find SWR and meters are available which are based
on all ol these. Most low cost instruments use voltage Fig. 23.16 An SWR meter arrangement based
on a Wheatstone bridge.
or current as their measurement parameter. These
instrumimts give correct readings only when the load position 1 and the value of Ry is adjusted to give full
is not loo reactive that is the SWR ratio being scale defection on the indicator. Having calibrated the
measured is low. The meters must also be connected instrument switch SW is moved to position 2, and the
to take readings at the load feed point or at half value of the SWR is read directly from the indicator.
wavelength distances along the transmission line,
Figure 23.17 shows an alternative SWR meter
back from the load. This usually means that the
construction. The pick up unit consists of three
transmission line needs to have a length which is an
conductors, the centre conductor being the core of the
integral value of its electrical length. The electrical
transmission line and the outer two conductors are
length for half wavelength and quarter wavelength
spaced equally from this. Usually this pick up unit is
transmission lines is respectively given by Eqns. 23.16
and 23.17
El =150F// ...(23.16)
= 75 F / f ...(23.17)
where f = frequency of antenna and
F = velocity factor whose value varies from
0.67 to 0.97 depending upon the dielectric
used.
SWR meters based on power measurements give
results which are independent of length. A true RF
wattmeter is used fitted with a directional coupler so
as to measure the forward and reflected power
separately and the SWR is calculated from Eqn. 23.14.
The radiation resistance of an antenna can also be
measured using a resistance bridge and the SWR
found from (Eqn. 23.12) or (23.13). Figure 23.16 shows
a Wheatstone bridge arrangement in which resistors
Rg, and Rx form three of the bridge arms, and the
radiation resistance of the antenna forms the fourth
arm. The supply to the bridge is a low power R.F Fig. 23.17 An SWR meter arrangement based on
signal generator, which provides the excitation. The forward and reflected wave measurements
Signal Analysers ' '

made as a pattern on a printed circuit board. Resistors noise bridge, which is a low cost instrument for
Rx and Ry are selected to have values equal to the measuring several parameters on antennas and
characteristic impedance of the line i.e., 50 ohms or transmission lines ; The noise source produces a wide
75 ohms. In operation any R.F. signal on one of the spectrum of pseudogaussian noise. This is amplified
pick up wires will be rectified by diodes D1 or D2 and then goes through an isolator before being
smoothed by capacitors or C2 and then applied as coupled to three output. The detector gives an
d.c. to the indicator. The forward signal from the indication of the received signal and shows a high
transmitter to the antenna picked up by the reflected noise level at all frequencies which are separate from
wire, is absorbed by resistor Rx so no forward voltage the resonant frequency of the device under test
appears on the pick up line after point X. However (usually an antenna or a transmission line). The
this signal remains on the forward pick up wire, and is electrical length of a transmission line differs from its
applied to the indicator. Similarly, the reflected signal physical length due to the velocity factor F. The noise
is picked up by the reflected pick up wire, but is bridge can be used to find the correct physical length
absorbed in resistor Ry of the forward pick up circuit. which gives an electrical length equal to half a
When making measurements with the meter wavelength. The transmission line should initially be
shown in Fig. 23.18 the switch is set to position 1 and cut to a length slightly greater then that given by
Rz is adjusted to give a reading at a set calibration (Eqn. 23.16) of (23.17). Rx is set to it low value, and one
point. The switch is then moved to position 2 and the end of the cable under test is connected to the test
SWR read from the indicator. socket, whilst test is connected to the test socket whilst
A good antenna has a standing wave ratio which the other end is short circuited. The detector is set to
varies between 1 : 1 and 1 : 1.2. Poor antennas have the-frequency at which the cable is to be half a
ratios greater than 1 :1.5 and most SWR meters do not wavelength in length and the frequency is slowly
give a reading beyond 1 : 3. decreased until null is reached. Now the cable is
Instruments are usually multipurpose for gradually trimmed in stages. After each trim its end is
example, combining a standing wave ratio meter with again short circuited and it is re-tested until
a field strength meter and power output meter. eventually the null point occurs at the required
Figure 23.18 shows a block diagram of an impedance frequency.

Fig. 23.18 Simplified block diagram of an impedance noise bridge.

Review Questions
I

1. Describe the circuits and working of wave 2. Describe the engineering applications of wave
analysers used for audio frequency and megahertz analysers.
ranges.
724 Electrical and Electronic Measurements and Instrumentation

3. Explain the different types of distortions caused by (ii) amplitude modulated signals
amplifiers. (iii) frequency modulated signals
4. Explain the term "total harmonic distortion". (iv) pulse modulated signal.
Describe the functioning of a total harmonic
8. Explain the following terms
distortion meter.
(i) SWR
5. What is Intermodulation Distortion ? Describe the
working of an intermodulation distortion meter (ii) VSWR
with the help of a block diagram. (iii) ISWR
6. Describe the working of a difference frequency 9. Explain the term Electrical length for transmission
distortion analysers with the help of a block lines. How is this affected by the operating
diagram. frequency and the velocity factor ?
7. Describe the basic circuit of a spectrum analyser. 10. Describe an arrangement for measurement of
Explain how the spectra of the following is Standing wave ratio.
displayed :
(i) continuous wave signals
High Frequency Measurements

measurements in this frequency range, the methods


24.1 INTRODUCTION used involve use of very simple apparatus which may
In this chapter methods for measurement of include :
inductance, capacitance and resistance in the (i) a set of inductors from 1.0 pH to 10 mH.
frequency range of 0.1 to 100 MHz are described. No (ii) a variable air capacitor of maximum
doubt, some of the methods described earlier i.e., capacitance 1000 pF,
bridge methods can be used in this frequency range (iii) a decade resistance box of resistance upto
but these methods give erroneous results on account 100 Q,
of the fact that these methods do not take into (iv) a thermo electric ammeter or an electronic
consideration the special effects which are inherent in voltmeter (EVM) in order to indicate
any measurement at high frequencies. Some of these resistance conditions.
special effects which are predominant at high
frequencies are : 24.2 RESONANCE METHODS
Most of the methods used for high frequency
(z) high reactance due to stray series inductance
measurements employ a series RLC resonant circuit as
and its variation with frequency ;
shown in Fig. 24.1. The emf for the circuit is obtained
(n) low reactance due to stray shunt capacitance
by coupling the coil L with the output coil of an
and its variation with frequency ;
oscillator. A loose coupling is desirable in order that
(iii) increase of effective resistance with the changes in conditions of the RLC circuit do not
frequency on account of skin effects ; effect the oscillator circuit. A tight coupling on the
(iv) increase of effective resistance on account of other hand, will affect the oscillator.
proximity effects;
(v) increase of effective resistance due to
production of eddy currents ;
(vi) variation of dielectric loss of capacitors with
frequency.
These effects make the high frequency
measurements complex. If bridge circuits are used at
Fig. 24.1 RLC resonant circuit.
high frequencies, elaborate shielding arrangements
have to be adopted to nullify the effects of factors
Resonance is obtained either by variation of
listed above. frequency of the oscillator or by variation of the
The normal approach for measurements at high capacitance and keeping the frequency constant. The
frequencies is to adopt such methods which are as resonance conditions are indicated by the thermal
simple as possible so that the effects of stray electric milli-ammeter connected in the circuit. (A thermal
and magnetic fields are easy to remove. Since errors of milli-ammeter is necessary as it can be worked at high
the order of 1 to 5 percent are acceptable in frequencies.) Under resonance conditions the

(725)
726 Electrical and Electronic Measurements and Instrumentation

milli-ammeter indicates maximum reading. However, variably capacitor, C, is used for obtaining resonance
in practice it is convenient to use an electronic conditions which are indicated by an electronic
voltmeter connected across the capacitor C to indicate voltmeter connected across it
the resonance conditions since the ammeter At resonance LC = 1/co2 =1/4k/2 ...(24.1)
introduces an extra series resistance in the circuit
which may be undesirable under many circumstances. Thus knowing the value of frequency, f and
The resonance is indicated when the voltmeter reads capacitance, C, the value of inductance can be
maximum. However, it is interesting to note that the calculated. The above method apparently looks very
circuit conditions which give maximum current are simple but the value of inductance obtained this way
not the same as those which show maximum voltage is rarely accurate. The reasons are discussed below :
across capacitor C, but, for high Q circuits the The above method does not take into account the
difference between the two conditions is negligible. additional capacitances which are introduced into the
Hence, in general, it is assumed that when the circuit. Also errors are caused due to the fact that the
voltmeter connected across C shows maximum dial setting of capacitor, C, may not be accurately
reading, the circuit is having maximum current i.e., the known. To offset the influence of above effects and to
circuit is working under resonance conditions. get improved accuracy a number of readings are taken
Another assumption, which is made for resonance by varying the frequency, f.
methods, is that the resonance curve is symmetrical Let Ca be the value of additional capacitance.
about its maximum ordinate. This condition is valid Therefore, for resonance at any frequency
only when : L(C+Cfl) = 1/4k2/2 ...(24.2)
(z) the curve is highly peaked i.e., highly 1 / /2 = 4k2L(C + Ca )= 4k2LC + 4K2LCfl
selective which is the case when the circuit
has a high Q factor ; ...(24.3)
(zz) the readings are taken in the vicinity of the From Eqn. 24.3, it is clear that if a curve is plotted
resonance point. To satisfy the condition between 1 / /2 and C, we get a straight line as shown in

(zzz) measurements should be limited to values of Fig. 24.2 (b). The line produced back cuts the x-axis at
O'. The slope of the line is tan 0 = 4k2L
current greater than 50% of the maximum
value of current. L=tan0/4zi2 ...(24.4)
Thus knowing the value of slope of the line, the
24.3 MEASUREMENT OF INDUCTANCE value of inductance, L, may be obtained.
Figure 24.2(a) shows a simple circuit for
The value of additional capacitance, C , is equal to
measurement of inductance, L, of a coil. This coil the intercept OO'. Now,
under measurement is loosely coupled to the output
coil of a variable frequency oscillator. The dial of the Ca=C4+Cr ...(24.5)
oscillator is calibrated in terms of frequency. A where Crf = self-capacitance of the coil,
Cv = capacitance of voltmeter.
Equation 24.5 is true only when the calibration of
capacitance C includes the effects of its screen
capacitances. Under condition, the method affords a
means of measurement of self-capacitance of coils.
In the above analysis we have assumed that the
value of inductance, L, remains constant. However,
this is not strictly true as the effective value of
inductance changes with frequency. Thus this method
gives erroneous results for coils whose effective self­
inductance varies considerably over the range of
frequency used. It is clear from Eqn. 24.3 that we get a
straight line relationship between C and 1 / f2 only if L
remains constant over the range of frequency used.
Thus if the value of self-inductance, L, changes with
Fig. 24.2 Measurement of inductance. frequency we do not get a straight-line but a curve.
High Frequency Measurements 727

Therefore, in order to determine the value of L at a The input capacitance of the electronic voltmeter is
particular frequency, a tangent is drawn to the curve 4.5 pF and the lead capacitance is 1.5 pF.
at a value the frequency where the value of inductance Solution. The method has been explained in Art.
is required. The value of inductance is determined 24.3. A curve is drawn between l//2 and C. The
from the slope of this curve using Eqn. 24.4. points for the graph are :
Example 24-i A capacitance of 250 pF produces ........................

C, pF f, kHz l//2
resonance with a coil at a frequency of (2/n)x IO6 Hz,
while at the second harmonic of this frequency resonance is 132 350 8.16 xlO’12
produced by a capacitance of 50 pF. Calculate : 98 400 6.25 xlO“12
(fl) the self-capacitance, and 73 450 4.93 x1012
(b) the inductance of the coil. 55 500 4.0x10 12
Neglect the effect voltmeter capacitance and also other 31 600 2.78x10'12
stray capacitances.
The graph is plotted in Fig. 24.3.
Solution. Suppose
C = capacitance of variable capacitor,
Cd - Self-capacitance of coil, and
L = inductance of coil.
At resonance,
L=l/4n2/2(C+Crf)
=_____________ 1_______________
4k2 x (2 / ti)2 x 1012(250 + Cd)x IO-12

= ------- - ------- •••(/)


16(250+Crf)
and also
L=__________ 1__________
4n2 x (4 / ti)2 x 1012(50 + Cd )x 10“12
Fig. 24.3 Plot of 1 / f2 v/s C for Example 24 2.
=-------- ------ •••(”) From the graph, additional capacitance,
64(50+ Crf)
From (z) and (ii), 16(250 + Cd) =64(50 + Cd) q=20 pF.
Capacitance of voltmeter and leads
or self-capacitance Crf = 16.67 pF.
= 4.5+ 1.5 =6.0 pF.
From (z), .’. Self-capacitance of coil
inductance, L =------------------- H = 234 uH Cd =20-6 =14 pF.
16(250 + 16.67)
Slope of line
Example 24.2 A coil of unknown inductance and self- . Q ab 4xl0~12 4
capacitance is connected in series with a standard variable tan 0 =---- =------------- =__
O'b 75xl0“12 75
capacitor C. An electronic voltmeter is connected across C.
A variable frequency oscillator is loosely coupled to the coil From Eqn. 24.4, inductance of coil
and the circuit is adjusted to resonance for each different L=^=± 1 H = 1.35mH
oscillatorfrequency fby adjusting C, as in thefollowing table: 4z? 75 4n2

f, kHz 350 400 450 500 600 24.4 MEASUREMENT OF CAPACITANCE


C, pF 132 98 73 55 31 Substitution method is normally used for
measurement of capacitance at high frequencies.
Determine :
Figure 24.4 shows a simple circuit for measurement of
(a) the inductance, and capacitance. C is the unknown capacitance connected
(b) the self-capacitance of the coil. in parallel with a variable standard capacitance Cs.
728 Electrical and Electronic Measurements and Instrumentation

The standard capacitance is of the same order as the


unknown capacitance.

Standard

Oscillator

Fig. 24.4 Measurement of capacitance by


substitution method.

With switch S open, resonance is obtained by


varying the standard capacitance Cs. The resonance
condition is indicated by an electronic voltmeter
connected across Cg.
Let Fig. 24.5 Measurement of effective resistance by
f = frequency at resonance ; Hz, resistance variation method.
L = inductance of coil coupled the output of
Let
oscillator; H,
E = induced emf in the resonant circuit,
Csl = setting of standard capacitor with
switch S open ; F. IQ = current in the resonant circuit,
R = effective resistance of resonant circuit
/. We have Csl = 1/4ti2/2L ...(24.6)
excluding resistance R^
Now the switch S is closed putting unknown R1 = additional resistance inserted in the
capacitance in parallel with Cg. Keeping the frequency resonant circuit,
f the same as earlier, resonance conditions are L = inductance of coil,
obtained by varying C .
C = capacitance,
Let Cs2 = setting of standard capacitor with V = reading of voltmeter connected across C,
switch closed.
and f0 = resonant frequency.
.-. We get C + Cs2 = 1 / 4k2/2L ...(24.7)
r E
At resonance L =--------- ...(24.9)
From Eqns. 24.6 and 24.7, value of unknown 0 R+R1 7
capacitance is :
I F
u = C
C Si -Cs2' .-. Voltage V = —2— --------------------
2tt/0C (R+R1)2n/0C
Thus the value of unknown capacitance is directly
...(24.10)
known from the difference of the two settings of the
standard capacitor. or R+ Rx = E/2nfQCV ...(24.11)
If the values of E, f0 and C are kept constant, we
24.5 MEASUREMENT OF EFFECTIVE get
RESISTANCE R+R^K/V ...(24.12)
There are two methods of measurement of
where K = a constant = E /2 nfQC ...(24.13)
effective resistance of a series RLC resonant circuit.
(i) Resistance variation method, and Thus, under the above conditions if a curve is
(n) Reactance variation method. plotted between 1/V and Rp we get a straight line as
shown in Fig. 24.5(1?).
24.6 RESISTANCE VARIATION METHOD The effective resistance is given by the intercept
The effective series resistance of a resonant circuit OO' provided that the loss resistance of capacitor and
is measured by adding extra resistance from outside. the resistance of the leads is negligible.
High Frequency Measurements 729

From the above analysis we conclude : The reactance is then adjusted so that the current
(z) We can determine the value of effective I=/0/V2
resistance of a resonant circuit by inserting an external /= 1_=A=_£_
variable resistance Rr The circuit is set into resonance Zr2 + x2
by varying the value of C. This value of capacitance C
is kept constant afterwards. We go on increasing the Hence, Vr2 + X2 = /2R or X=±R
value of Rr This does not disturb the resonance condi­
...(24.15)
tions and its effect is only to decrease the value of reso­
There are, therefore, two values of reactance X
nance current IQ and to decrease the value of voltage V
which give a current, I = /0 / >/2 .
across the capacitor C. Thus an increase in the value of
Rj decreases V and increases the value of 1/V. The difference of these two values or reactance
gives twice the value or effective resistance i.e., 2R.
(ii) The analysis is valid only if the value of
induced emf E in the resonant circuit is constant. Also As mentioned above there are two ways of
the resonant frequency fQ is constant. This requires varying the reactance :
that the coupling between the oscillator circuit and the (i) variation of capacitance, C, and
resonant circuit be very loose otherwise changes in the (ii) variation of frequency/
resonant circuit would affect the oscillator circuit. In
The two methods are discussed below :
order to ensure that the value of induced emf and the
resonant frequency remain constant for all the 1. Variation of capacitance. In this method, the
readings, the oscillator should possess a high degree value of frequency / is kept constant and the
of frequency and amplitude stability. capacitance is varied. Let the value of capacitance
(iii) We have explained above that the value of which gives the maximum current be CQ and the two
induced emf E in the resonant circuit should remain values of capacitance which give a current of / /2 be
constant. This can be checked by connecting a thermal Cj and C2.
ammeter in series with the output coil of the oscillator Therefore,
or by connecting an electronic voltmeter across the 2R = 1/wC2-1/coC1
output coil of the oscillator.
r _ C1 -C2 C -C
= H ^2
or ...(24.16)
24.7 REACTANCE VARIATION METHOD 2cdC1C2 4ti/C1C2
The effective resistance of a resonant circuit can be Now if the circuit loss is small and the resonance
measured by variation of reactance of the circuit. is peaked and symmetrical,
Figure 24.6 shows the circuit for such a measurement. C c » C2
*"1^2 0
...(24.17)
C__
r~ H C -C
-C2 ~ H ^2 ...(24.18)
" 2coC2 ~ 4n/C2

Also Q = coL/ R =1/ coC0R ...(24.19)


. 2Co
...(24.20)
C] -C2
Fig. 24.6 Measurement of effective resistance by
reactance variation method. The accuracy of this method depends upon the
accuracy with which the difference Q - C2 can be
In this circuit a thermal ammeter is used to known. For accurate determination of this difference a
measure the value of current flowing. The circuit micrometer capacitor is put in parallel with the main
reactance is varied by either varying the value of capacitor. From Eqn. 24.20 it is clear that the circuit Q
capacitance C or the frequency f. The reactance is can be determined in terms of two values of capacitance.
adjusted till the ammeter reads maximum value. This
2. Variation of frequency. In this method the
happens when the circuit is under resonance conditions.
capacitance C is kept constant and the frequency is
The maximum value of current: varied. Let a>0 be the angular frequency to give
I0 = E/R ...(24.14) maximum current IQ and od1 and co2 be the two
where E = emf induced, and frequencies which give a value of current
R - effective value of resistance. I=!o/22.
730 Electrical and Electronic Measurements and Instrumentation

2 R = (WjL-1/ w1C)-(oj2L-1/ co2C) same scale reading I for the tzoo values ofC - namely 37 and
= (cd1 -co2)L+1/ C(l/ <£>2 — 1 / CDj ) 45 pF, one being below and the other above the capacitance
n CQ needed to produce maximum current.
= (cOj - w2)L+ (DqL
The capacitor is now adjusted to a value and an
= (a>1 -co2)/(co1co2) as co0L-1 / C.
added series resistance of 75 Q is found to reduce the current
Now for a peaked and symmetrical resonance to previous value of I. Determine values ofL and R.
curve :
Solution. Let the two values of capacitance which
(Op = co1co2 ...(24.21) produce the same current be Cf and C2.
9 CO. - (0o
2 R «(co.1 -coz.2)L+ cOpL
U
—----- - ~2(co.i -co7z )L Cj = 45 pF and C2=37pF
Let the resonance curve be peaked and
or R «(w1 -w2)L ...(24.22)
symmetrical and therefore capacitance required for
(Oj -o>2 resonance is :
...(24.23)
WqC _ q + C2 _ 45+37
= 41pF.
°~ 2 2
and ...(24.24)
R co1 - co2 At resonance, inductance

The frequency variation method has a ___ ____ —________________________J_J


4k2/2C0 4k2 x (3x 106)2 x 41 x 10’12
disadvantage because it is quite difficult to keep the
value of induced emf E constant with change of = 69.2 pH.
frequency. But the error introduced on this account is
When the capacitance in the circuit is Cr the value
small and can be allowed for.
of current
Example 24.3 A coil of inductance L is tuned by a
capacitance C to resonate at 1 MHz. A standard capacitor of a/r2+(coL-1/coC1)2

230 pF is tuned to resonate with the same coil at 2 MHz. If


the resonance is indicated by a VTVM of capacitance 8 pF in
parallel with the circuit, calculate the values ofL and C.
2kx 3x 106 x 69.2 x 10-6
Solution. We have
r2 + 1
C+Cv = 1/4k2L(1x106)2 2kx3x106x45x1012J

or (C + 8)x IO-12 = 1/(4k2Lx 1012) ...(z)


and (Cs + CJxlO“12 = 1/4k2L(2 x 106)2 •••(0
£
or (230+ 8)x 10~12 = 1/(16k2Lx 1012) ...(zz) At resonance, I =-------- ...(ii)
R + 75 ' ’
where Cv = capacitance of voltmeter Equating (?) and (zz) we get, R = 70.8 Q.
= 8x10-12F
Example 2445 Calculate the Q factor and effective
and Cs = capacitance of standard capacitor
resistance of a circuit tuned to a frequency of 1.5 MHz and
= 230x10-12F
having an effective capacitance of 150 pF. In this circuit the
From relation (zz), we get, current falls to 70.7% of its resonant value when the
Inductance frequency of an emf of constant magnitude injected in series
with the circuit deviates from the resonant frequency by
1 5 kHz.
L= H = 26.6 pH
238 x16 k2
Solution. The analysis of this problem has been
From relation (zz), capacitance done in Art. 24.6.
From Eqn. 24.24,
C = —-------8=944pF.
4k2 x 26.6
Q factor =----- -— = —.
Example 24.4 A constant rms voltage of frequency fz
3 MHz is applied to a coil of inductance L and resistance R,
a calibrated variable capacitor C, and a thermojunction all Resonant frequency
in series. The uncalibrated ammeter is found to show the fQ = 1.5 MHz =1.5x 106 Hz
High Frequency Measurements 73 I

The two frequencies at which the current is 70.7% ZD in parallel is equal to zero. The other impedances
of or 1 / x/2 time the current at resonant frequency are : i.e., ZR, Zr r, Zrr do not affect the null conditions.
d C', Zt'
= (1.5 x 106 + 5 x 103) Hz Therefore at null point:
and /2 = (1.5x106-5x103)Hz
—+ —=0 or Z.+Zn=0
.-. =10xl03 Hz
Hence, Q = ^=150 or ZA = -ZD ...(24.25)
10xl03 But
From Eqn. 24.23, resistance 2^ = 21 + Z2 + Z] Z2 / Z3
(Qj, -o>2 _ f1~ f2 and ZD = Z4 + Z5 + Z4Z5 / Z6
w2C 2 n /02C Therefore, at null point,
Z] + Z2 + Z1Z2 / Z3 = -Z4 - Z5 - z4z5 / Z6
...(24.26)
A common parallel T network for measurement of
24.8 T NETWORKS frequency is shown in Fig. 24.8. Comparing Fig. 24.8
T networks are very useful for measurement of with circuit of Fig. 24.7(a).
inductance, capacitance, resistance and frequency in
the high frequency range. These networks employ null
techniques.
There are two types of T networks used :
(z) Parallel T networks, and
(zz) Bridge T networks.

24.9 PARALLEL T NETWORK


Figure 24.7(a) shows a parallel T network with
source null detector. In fact there are two Ts formed by Fig. 24.8 Parallel T network for measurement
impedances Z]z Z2, Z3, and Z4, Z5, Z6 in parallel. The of frequency.
network can be transformed into another form shown
in Fig. 24.7(b) by star delta transformation. A null is We have Z^ = Rv Z2 = Rr Z3 = -j / coC1,
indicated by the detector if the admittance of and 24 = -j I «C2, Z5 = -j I coC2 and Z6 = R2.
Applying Eqn. 24.26 for null conditions, we have,
R2 z i 1
R, + R, 4--------------- -------- 1------- 1—=—=—
1 -j/wCl wC2 mC2 CO2C2R2

or 2 R. + = -t- + ? \—
171 1 coC2 co2C|R2

Equating real and imaginary terms we get,


2R: =l/co2CfR2 or w2=l/2R1R2C2
...(24.27)
and R12coC1 = 2 / coC2 or co2 =2 / R2C1C2
...(24.28)
This circuit (Fig. 24.8) therefore can be used to
measure frequency. From Eqns. 24.27 and 24.28, we
have,
1/2R1R2C2=2/R2C1C2

This condition is fulfilled if

Fig. 24.7 Parallel T network. R}- 4 R^ and = C2.


732 Electrical and Electronic Measurements and Instrumentation

24.10 BRIDGE T NETWORK The T networks have an advantage over ordinary


Bridge T network is a simplification of parallel T bridge circuits. It is clear that these networks have a
network. If the impedance Z3 is infinite and the common point between source and detector. This
combination of impedances Zp and Z2 is made into a point may be earthed and thus the screening of whole
single impedance Zr it becomes a bridge 'T network. of the circuit becomes easy.
Figure 24.9 shows a bridge T network. Example 24.6 Two variable capacitors
mechanically coupled so that their capacitances are always
equal, are connected in series between terminals A and B to
form the series arms of a bridge 'T' network. Their junction
is connected through a variable resistor R2 to the terminal
C. A coil of unknown inductance L and resistance R is
connected across terminals A and B. The input voltage is
applied between A and C, and a detector is connected
between B and C. Under what conditions will the detector
show an output voltage approaching zero ? Evaluate R and
Fig. 24.9 Bridge Network.
L if this balance condition is obtained at a frequency of
From Eqn. 24.26 for a bridge T network 5 MHz with Cj = 45 pF and R1 = 4.2 Q.
^1 = ~^4 ~%5~ Z4Z51Z6 ...(24.29) Solution. The network is similar to the one shown
as Zo is infinite and Z. + Z- = Z,. in Fig. 24.10. The analysis of the network is carried out
in Art. 24.10.
Figure 24.10 shows a bridge network for measure­
Equations 24.30 and 24.31 give conditions for no
ment of effective resistance R, inductance L and Q of a
output voltage as :
coil.
Coil
(o2C2Rj

j 7 2
and L=—x—
<o2Cj
R 1
~ 4n2 x (5 x 106)2 x (45 x 10'12)2 x 4.5

= 0.115 MQ
Fig. 24.10 Bridge T network for measurement of ____________ 2___________
effective resistance, inductance and Q. ~ 4n2x(5xl06)2x45xl0’12
For this circuit
= 45 pH.
Zj = R + /wR Z4 = -j I coCr
Z5 — y / wCp z6 = Rp 24.11 Q METER
At null point we have The determination of the storage factor Q is one of
~ ~^4 ~ Z5 — Z4Z5 / Z6 (Eqn. 24.29) the most widely used means in the laboratory for
testing radio frequency coils (RFC), inductors and
R u. W - _7 .
capacitors. The storage factor is equal to Q = co0L/ R
coCT coCj co2C2Rj
where co0 is the resonant angular frequency, L is the
inductance and R is the effective resistance of a coil.
The effective resistance, R, is never determined
directly since its value depends upon the value of
Equating real imaginary terms, we get,
frequency. As is well known, the value of effective
R = 1/oj2C12R1 ...(24.30) resistance with a.c. differs from its d.c. value on
and L - 2 / (a2C1 account of skin effect and eddy current losses. Since
...(24.31)
the value of R varies in a complex manner with
...(24.32) frequency, it is indirectly determined by measuring
1 the value of Q.
High Frequency Measurements 733

The Q meter is an instrument which is designed to If the input voltage E is kept constant, the voltage
measure the value of Q directly and as such is very appearing across the capacitor is Q times E and a
useful in measuring the characteristics of coils and voltmeter connected across the capacitor can be
capacitors. calibrated to read the value of Q directly.
Principle of working. The principle of working of
Practical Circuit
this useful laboratory instrument is based upon the
well-known characteristics of a resonant series R, L, C A practical Q meter circuit is shown in Fig. 24.12.
circuit. Figure 24.11(a) shows a coil of resistance R and It consists of a self-contained variable frequency RF
inductance L in series with a capacitor C. oscillator. This oscillator delivers current to a low
value shunt resistance Rsh. This low resistance is of the
order of a hundredths of an ohm ; a typical value may
be n 02 fi. Through this resistance a small value of
voltage E is injected into the resonant circuit. This
voltage is measured by a thermocouple voltmeter.
Since the value of shunt resistance is very low, it
introduces almost no resistance into the oscillatory
circuit and therefore represents a voltage source of
magnitude E, with a very small internal resistance. A
calibrated standard variable capacitor C is used for
resonating the circuit. An electronic voltmeter is
Fig. 24.11 A resonant RLC circuit and its
phasor diagram. connected across this capacitor. The coil under test is
connected to terminals and T2.
At resonant frequency fQ, we have,
xc=xL Application
where capacitive reactance Xc = 1 l2itfQC 1. Measurement ofQ. The circuit for measurement
of Q is shown in Fig. 24.12. The oscillator is set to the
and inductive reactance X, =2 7r/oL=wot
desired frequency and then the tuning capacitor is
resonant frequency f° 2itjLC adjusted for maximum value of EQ. As discussed
earlier under these conditions Q = EQ/ E and if the
and current at resonance voltage is kept constant, the voltmeter connected
across the capacitor may be calibrated to read the
The phasor diagram is shown in Fig- 24.11(1’). value of Q directly. This measured value of Q is
Voltage across capacitor, Ec = ZQXC = = ^o0)o^ commonly regarded as the Q of the coil under test.
Input voltage E = IQR However, there is an error. The measured value of Q
is the Q of whole the circuit and not of the coil. There
■jC _ 0'0^ _ _Q are errors caused on account of the shunt resistance
E I0R R and also due to the distributed capacitance of the
E0 = QE -(24.32) circuit.
Thus the input voltage E is magnified Q times.

Figure 24.12 Circuit of a Q meter.


734 Electrical and Electronic Measurements and Instrumentation

Correction for shunt resistance. Measured value, Thus,


f =______1______
= _o0L 2 2^HC2 + Cd)
",eas R+R. sh Now 72=2/,
True value = iaQL/ R • 1 = 2 x 1

2n/UC2 + Cll) 2njL(C^Cj)


*
+ -(24.33)
C. -4C,
or distributed capacitance = 1 —- ...(24.35)
Thus the measured value of Q is smaller than the
true value. Now if coils of high resistance (low Q
5. Measurement of bandwidth. The measurement
coils) are being measured, the difference between the
of bandwidth with a Q meter is similar to the one used
two values may be negligible. But when
for measurement of effective resistance by variation of
measurements are done on low resistance (high Q)
frequency method given on page 730. The bandwidth
coils, the error caused on this account may be serious.
is calculated by using Eqn. 24.24.
Correction for distributed capacitance. The
6. Measurement of capacitance. For measurement
distributed capacitance or self-capacitance of
of capacitance a dummy coil is connected across
measuring circuit modifies the true value of Q. The
terminal an T2 (Fig. 24.12). The circuit is resonated
measured value of Q is less than the true value by a
by varying the value of the tuning capacitor. Let this
factor that depends upon the value of the distributed
value be Cr
(self) capacitance and the tuning capacitor. It can be
shown that The capacitor under test CT, is now connected
f CA across terminals T3 and T4. This puts the test capacitor
’'-true
Q =Q*~>neas 1^1
1 + -M ...(24.34)
' ' in parallel with tuning capacitor. The circuit is
resonated again by varying the value of tuning
where Cd = distributed or self-capacitance, and capacitor. Let this value be C9. Therefore value of
C = tuning capacitance. capacitance under test is CL = CT - C2.
A method for measurement of distributed (self) Th is method is similar to the one given in Art. 24.4
capacitance is given later. on page 727.
2. Measurement of inductance. The value of Example 24.^ A circuit consisting of a coil, a resistance
inductance is given by L = l/4k2/02C. an a variable capacitor connected in series is tuned to
The values /0 and C are known and therefore the resonance using a Q meter. If the frequency is 500 kHz, the
value of inductance may be calculated. resistance 0.50. and the variable capacitor set to 350 pF.
Calculate the effective inductance and resistance of the coil,
3. Measurement of Effective resistance. The value
if the Q meter indicates 90.
of the effective resistance may be computed from the
relation R = ^0L/Q^. Solution. At resonance ;
T J T 1
4. Measurement of self-capacitance. The self­ Inductance L = —n „
capacitance is measured by making two measure­
ments at different frequencies. The capacitor is set to a =_____________1_____________ H
high value, and the circuit is resonated by adjustment 4k2 x (500 x 103)2 x 350 x 10"12
of the oscillator frequency. Resonance is indicated by
= 290 pH
the circuit Q meter. Let the values of tuning capacitor
be Cj and that of frequency be f^ under these Let R be the resistance of coil.
conL
conditions. Therefore, .-. R + 0.5 = —2-
Q
f -______ 1
1 2^L(C1+CI() 2kx500x103x290x10“6
90
The frequency is now increased to twice its initial
value and the circuit is resonated again this time with Effective resistance of coil R =10.25 -0.5 = 9.75 0
the help of tuning capacitor. Let the value of tuning Example 24.8 A coil of resistance 10 O is connected in
capacitor be C2 and that of frequency be f2 under these the Q meter circuit as shown in Fig. 24.12. Resonance
conditions. occurs at a frequency of 1 MHz with the tuning capacitor
High Frequency Measurements 735

set at 65 pF. Calculate the percentage error introduced in Example 24.10 Tests using a Q meter, on a radio
the calculated value of Q if a resistance of 0.02 Q is used tuning coil to find its self-capacitance gave the following
across the oscillator circuit. results :
Solution. True value of Q is : 1. With a radio coil connected normally, the
resonance was obtained at 1 MHz with the
tuning capacitor set at 80 pF.
1 2. With a standard inductor connected in place of
(2nx 106)x (45 x 10-12)710 the radio coil, resonance was obtained at 3 MHz
and this condition was not altered when the radio
Measured value of coil was connected in parallel with the standard
inductor. Calculate the self-capacitance of the
'neas cdC(jR+0.02) radio coil.
Solution. Let
=____________ 1____________
(2tix106)x (45 xl(F12)x 10.02 Q = capacitance of tuning capacitor for 1st setting,
coj = angular frequency for 1st setting.
= 244.5
.'. %age error Inductance of coil

= 245----- 5 x 100 = 0.2% high


-244±2 L=1/<^(C1 + Cd)
245 6
In this second setting; the reason for the resonance
Example 24.9 The self-capacitance of a coil is measured conditions to remain unchanged is that a very high
by a Q meter. The circuit is set into resonance azt 2 MHz impedance has been placed in parallel, this being the
and the tuning capacitor is as value of 460 pF. The resonant circuit of the radio coil and its self
frequency is now adjusted to 4 MHz and resonance capacitance. In other words, the radio coil in parallel
conditions are obtained the tuning capacitor at 100 pF. with its self capacitance is resonant at 3 MHz and thus
Calculate the value of self-capacitance of the coil. it may be written
Solution. L=l/co|Crf ...(zz)
The problem has already been analysed in
From (z) and (n) we have, self capacitance of coil is,
previous page.
From Eqn. 24.35, self-capacitance of coil
C -4C d co2-co2
d 3
(l)1
23
400-4x100 _n „ = 80x = 10 pF
=---- 3------- = 20 pF (3)2-(l)2

Review Questions
1. Discuss the problems associated with 4. Discuss the following two methods of
measurement of resistance, inductance and measurement of effective resistance of a coil at
capacitance at high frequencies. high frequencies :
2. Describe the resonance method for measurement (i) Resistance variation method.
of inductance at high frequencies. Discuss the (zz) Reactance variation method.
assumptions made. How is the effect of additional
5. Describe the following methods of measuring
capacitances in the tuning capacitor taken care of ?
effective resistance of a coil using resonant
3. Describe the method for measurement of methods :
capacitance at high frequencies using resonance
(i) Variation of capacitance
method.
(zz) Variation of frequency.
736 Electrical and Electronic Measurements and Instrumentation

6. Explain how frequency can be measured using a 9. Describe how corrections for shunt resistance and
parallel T network. Derive the expression for the distributed capacitance are applied when
frequency in terms of values of components used measuring Q factor of a coil with a Q meter.
in a parallel T network. 10. Describe the measurement of following using a
7. Describe how Q factor of a coil can be measured Q meter :
using a Bridge T network. Derive the expression (/) Q factor (ii) inductance
for Q. (iii) effective resistance (iv) Self capacitance
8. Describe the circuit and working of a Q meter. (v) Bandwidth (vi) capacitance
Describe its applications.

Unsolved Problems "V

1. A capacitor of capacitance 245 pF produces each frequency the capacitor is adjusted to produce
resonance at an angular frequency of 5 x 106 rad/s, a maximum reading on the voltmeter.
while a capacitor of capacitance 50 pF produces The readings are as follows :
resonance with the second harmonic of this
frequency. Calculate : Frequency Capacitance Maximum
kHz PF voltage, V
(a) the inductance, and
200 345 4
(b) the self capacitance of the coil.
300 145 -
[Ans. 154 pH, 15 pF]
400 75 -
2. With a suitable standard inductor connected to a
500 43 -
Q-meter, resonance is obtained with an angular
frequency co with capacitor setting C}; the indicated 600 25 16
Q factor is Qr The unknown impedance is
Determine :
connected in series with the standard inductor and
resonance is re-established by resetting the (a) the inductance of the test coil,
capacitor to C2, the corresponding Q factor being (b) its self capacitance, and
Q2- Calculate : (c) the percentage increase in its resistance as the
(a) the resistive, and frequency varies from 100 kHz to 600 kHz.
[Ans. 1.732 mH, 12 pF, 125%]
(b) reactive components of the unknown imped­
ance when 5. The effective resistance of a coil at a frequency of
q = 208 pF, Q = 170, 0.2 MHz is measured by the reactance variation
method. The coil is connected in series with a
q = 185.5 pF and Q2 = 52.
calibrated variable capacitor and a 10 Q resistor. A
The frequency is 159 kHz. [Ans. 35 Q, 584 Q] constant emf at 0.2 MHz is injected into the circuit.
3. A coil of unknown impedance in series with a An electronic voltmeter connected across the
capacitor of 224 pF and an ammeter of negligible resistor is used to indicate resonance. The capacitor
impedance was connected to a variable frequency settings to reduce the resonant current by one half
of constant voltage and negligible impedance. The are 150 pF and 170 pF. Calculate the effective
frequency was adjusted both above and below resistance of the coil. [Ans. 170 Q]
resonant frequency till the reading of ammeter was
6. A coil of unknown impedance in series with a capa­
reduced to 70.7% of its value at resonance. This
citor of capacitance 0.02 pF and an ammeter of negli­
occurred at frequencies of 876 and 892 kHz.
gible impedance was connected to a variable
Determine effective resistance, inductance and Q
frequency source having constant voltage. The
of the coil. [Ans. 14.5 Q, 0.145 mH, 55.3]
frequency was adjusted both above and below
4. A test coil is connected across a calibrated loss free resonance until the reading of the ammeter was
variable capacitor having in parallel with it a reduced to 0.8 of the value at resonance. This
VIVM of input capacitance 3 pF. A constant occurred at frequencies of 0.97 and 1.01 MHz. Find
current, of varying frequency is passed through an the inductance and resistance of the coil.
injector coil loosely coupled to the test coil, and at [Ans. 1.29 pH; 0.424Q]
High Frequency Measurements 737

7. A test or a radio tuning coil, using Q meter, gave Calculate for the VTVM :
the following results : (a) the input resistance, and
(a) With a radio coil connected normally, reso­ (b) the input capacitance. [Ans. 496 kQ, 10 pF]
nance was obtained at a frequency of 1 MHz
9. A coil was tested using a Q meter and following
with the Q meter capacitor set at 480 pF.
information was obtained. At a frequency of 3 MHz,
(b) With standard inductor connected in place of the value of capacitance to give maximum voltage
the radio coil, the resonance was obtained at across the capacitor was 251 pF while at 6 MHz
5 MHz and this condition was not altered when frequency it was 50 pF. Calculate the self
the coil was connected in parallel with the stan­ capacitance of the coil. [Ans. 17 pF]
dard inductor. Calculate the self capacitance of
10. A coil with a resistance of 0.1 Q is connected in a Q
radio coil. [Ans. 20 pF]
meter circuit as shown in Fig. 24.12. Resonance
8. A Q meter gave the following readings during a occurs when the oscillator frequency is 40 MHz
certain test on a coil : and the tuning capacitor is set at 135 pF. Calculate
Q = 250, C = 150 pF, f = 750 kHz. the percentage error introduced in the calculated
A VTVM was then connected across the standard value of Q by a 0.02 Q shunt resistance connected
capacitor and resonance re-established. The across the oscillator circuit. Comment upon me
corresponding readings now become result. [Ans. 17%]
Q = 150, C= 150 pF, / = 750 kHz.

Objective Type Questions


Tick (^) most appropriate answer : 4. A series RLC circuit is used for measurement of
inductance of a coil using resonant method.
1. The effective reactance of an inductive coil, Resonance conditions are produced by varying the
(a) increase because of stray capacitances as the capacitance. A thermoelectric type ammeter is
frequency increases used in series with the circuit. Also an electronic
(b) decrease because of stray capacitances as the voltmeter is connected across the tuning capacitor
frequency increases (a) The conditions of resonance are more reliably
(c) remains the same irrespective of the increase in indicated when the ammeter reads maximum
frequency even if stray capacitances are present (b) The conditions of resonance are more reliably
(rf) none of the above. indicated when the voltmeter connected across
the capacitor reads maximum
2. The effective reactance of an inductive coil
(c) The conditions of resonance are the same when
(a) increases due to stray series inductances as
both ammeter and voltmeter read maximum
frequency increases
(rf) The conditions of resonance are indicated when
(b) decreases due to stray series inductances as
both ammeter and voltmeter read minimum.
frequency increases
5. If the Q factor of a coil which is w0L/ R is measured
(c) remains the same irrespective of the increase in
by varying the frequency
frequency even if stray series inductances, are
present (a) the plot between Q and frequency is linear

(rf) none of the above. (b) the value of Q initially decreases with increase
of frequency and afterwards increases with
3. The effective resistance of a coil at high frequencies increase of frequency
is more than its d.c. resistance on account of
(c) the value of Q initially increases with increase
(a) skin effects I
of frequency and afterwards decreases with
(b) proximity effects increase of frequency
(c) eddy current losses 1 (rf) the Q factor remains constant irrespective of
(rf) all the above. the value of frequency.
738 Electrical and Electronic Measurements and Instrumentation

6. The Q factor of a coil at the resonant frequency respectively. The value of distributed capacitance
1.5 MHz of an RLC series circuit is 150. The is :
bandwidth is :
(«) (fc)
(rt) 225 MHz (fc) 1.06 MHz
(c) 10 kHz (j) none of the above.
7. The resonant frequency of an RLC series circuit is
1.5 MHz with the tuning capacitor set at 150 pF. 9. The true value of Q of a coil is 245 and the
The bandwidth is 10 kHz. The effective resistance measured value is 244.5. The ratio of distributed
of the circuit is : capacitance to self capacitance of the coil is
(rt) 2.04x1 O’3 (b) 1.002
(a) 29.5 Q (b) 14.75 Q
(c) 9.4 Q (d) 4.7 Q (c) 0.997 (d) none of the above.
10. In a Q meter, the value of shunt resistance
8. In a Q meter, distributed capacitance of coil is
connected across the oscillator is typically of the
measured by changing the capacitance of the
order of
tuning capacitor. The values of tuning capacitor
(rt) Q (b) mQ
are Cj and C, for resonant frequencies f} and 2/t
(c) pQ (d) kQ.

1. (b) 2. (rt) 3. (d) 4. (b) 5. (c) 6. (c) 7. (d) 8. (c) 9. («) io. (b)
Primary Sensing Elements
and Transducers
I

25.1 INTRODUCTION absolutely no energy is extracted from the measurand


The measurand in an instrumentation system during the process of conversion and all the energy
makes its first contact with a Primary Detection Element that is required for conversion is supplied from
or an Input Device. There is a multiplicity and variety outside, so that the measurand is not distorted and the
of measurands to be measured. These include process analogous output of the detector is a faithful represen­
variables like temperature, pressure and flow rate tation of the measurand.
which are widely employed in process and production
plants. The measurands also include electrical 25.2 MECHANICAL DEVICES AS
quantities like current, voltage, resistance, inductance, PRIMARY DETECTORS
capacitance, frequency, phase angle, power and In order to extract information from mechanical
magnetic quantities like flux, flux density, reluctance systems, only mechanical displacement or velocity can
etc. be used, and therefore the importance of mechanical
All these quantities require a primary detection sensing elements is obvious. Some of the commonly
element and/or a transducer to be converted into used mechanical sensing elements are springs - which
another analogous format which is acceptable by the convert a force or a torque into a displacement ; a
later stages of the measurement system. diaphragm, a capsule, bellows or Bourdon tube -
The measurand or the input signal is called an which convert pressure into a displacement, a
information for the measurement system. The bimetallic strip converts temperature into a
information may be in the form of a physical displacement ; a mass damper system is used for
phenomenon or it may be an electrical signal. The measurement of acceleration, velocity and displace­
process of detection and conversion of the information ment. Some input devices may involve more than one
into an acceptable form requires energy. This energy mechanical conversion for example, fluid flow
may be extracted from the measurand, but in that case measurements may involve conversion of fluid rate
it will not be represented in its faithful form as it into pressure differential using an orifice, venturi tube
would be subjected to loading errors. In order that a or pitot tube and then in turn this pressure is
measurand is represented in its faithful form converted into displacement for purposes of
measurement.
undistorted, no energy should be extracted from it
during the process of conversion i.e., it should not be There are a number of mechanical quantities
subjected to any kind of loading effects. In fact, efforts which are to be measured. Some of these quantities are
should be made to supply energy required for listed in Table 25.1 along with their modes of
conversion from outside sources so that the operation for the purposes of measurement.
measurand is not distorted during the process of These mechanical quantities include force,
conversion in order that it be faithfully reproduced in pressure, displacement, flow rate, temperature and
its analogous form. The ideal conversion is where the list is perhaps unending.
(739)
740 Electrical and Electronic Measurements and Instrumentation

Table 25.1 Mechanical quantities and their Modes of Operation


Type Operation
1 A.
Contacting spindle, pin or finger Displacement to displacement

1 B. Elastic Member
1. Prooving ring Force to displacement.
2. Bourdon tube Pressure to displacement.
3. Bellows Pressure to displacement.
4. Diaphragm Pressure to displacement.
5. Spring Force to displacement.

C Mass

i 1. Seismic mass Forcing function to displacement.


i 2. Pendulum scale Force to displacement.
i
3. Manometer Pressure to displacement.
1
s D. Thermal
1. Thermocouple Temperature to electric current.
t
• 2. Bimaterial Temperature to displacement.
3. Temp-strk Temperature to phase.

E. Hydropneumatic

i
1. Static
(a) Float Fluid level to displacement.
s (b) Hydrometer Specific gravity to displacement. 1
1
2. Dynamic I
(zz) Orifice Velocity to pressure. j
j (b) Venturi Velocity to pressure.
(c) Pitot tube Velocity to pressure.
(d) Vernes Velocity to force. i
?
(e) Turbines Linear to angular velocity.
[------- ____ _ __ 1
The initial concept of converting an applied force into a displacement is basic to many types of primary
sensing elements. The mechanical elements which are used to convert the applied force into displacement are
usually elastic members. There are many types of these elastic members. They can be classified into three
categories, although some elastic members may fall into a combination of these categories.
The three categories are :
(z) Direct tension or compression type :
(zz) Bending type ;
(z'z'z) Torsion type.

25.3 MECHANICAL SPRING DEVICES


Most mechanical-input measuring systems employ mechanical springs of one form or another. The
displacements are usually small and engineering approximations for small displacements or deflections are
valid. Various common types of springs are shown in Fig. 25.1. These range from cantilever, helical and spiral
springs to torsion bars, prooving (proof) rings and spring flexture pivots.
Primary Sensing Elements and Transducers 74 1

Fig. 25.1 Spring elements used for sensing force (f) or torque (7).

25.3.1 Cantilever 25.3.2 Helical Spring


A cantilever is shown in Fig. 25.2 which is Figure 25.3 shows a close coiled helical spring
subjected to a force at its free end. subjected to a compressive force F.

Fig. 25.2 Cantilever.

Deflection at the free end


x=Fl3/3EI ...(25.1)
where
Fig. 25.3 Closed coiled helical spring.
F - applied force ; N,
Displacement of spring :
I = length of cantilever, m ;
2 8 FD3n
£ = modulus of elasticity ; N / m x =---- —_ ...(25.3)
/ 2 A 3 Grf4
I = moment of inertia = —
112 J where F - applied force ; N,
b = width of cantilever ; m D= mean diameter of coiled spring ; m,
t = thickness of cantilever ; m d- diameter of spring wire ; m,
Stiffness of cantilever n= number of wires,
G= shear modulus ; N/m2.
K=F/x=3EI/l3 .-(25.2)
742 Electrical and Electronic Measurements and Instrumentation

Stiffness of spring K = F/x Sometimes, notched bars, are used. The notched
bar has the advantage that it has a greater sensitivity
*/8
= Gd D 3n ...(25.4)
on account of its reduced diameter (of the notched
Maximum shear stress, portion).
T = 8FD/(nd3) ...(25.5)
25.3.5 Prooving (Proof) Rings
25.3.3 Spiral Spring They are used for measurement of force, weight
Figure 25.4 shows a flat spiral spring subjected a or load. The applied force causes a deflection which is
torque T. measured with the help of electrical transducers.
Prooving rings are made up of steel and are used
as force standards. They are particularly useful for
calibration of material testing machines in situations
where dead weight standards are impracticable to use

Fig- 25.4 Flat spiral spring.


The deflection of the spring is :
0= Ebt3T/12/, rad ...(25.6)
where E = modulus of elasticity, N/m2,
b - width of spring ; m, Fig. 25.5 Prooving ring.
t = thickness of spring ; m,
I = length of spring ; m, on account of their bulk. A prooving ring is a circular
T = torque ; Nm ring of rectangular cross-section as shown in Fig. 25.5
.'. Stiffness of spring which may be subjected to either tensile or compre­
K = T/e = Ebt3/12 I Nm/rad ...(25.7) ssive forces across its diameter.
Maximum fibre stress The deflection is given by :
Smax=6T/M2N/m2 ...(25.8)
x = (7t/2 ~4-^— F ... (25.10)
Spiral springs are used for production of 16 EI
controlling torque in analog instruments.
where d = outside ring diameter ; m.
25.3.4 Torsion Bars or Shafts The other terms have been explained earlier.
These are primary sensing elements for torque.
The common practice for measurement of
They are made use of in torque meters. The deflection
displacement is to attach a displacement transducer
or twist of the bar is proportional to the applied torque
between the top and bottom of the prooving ring.
and the deformation is used as a measure of the torque.
When the force is applied, the relative displacement
Some torque meters are designed so that the
can be measured. An LVDT is normally used for
angular displacement due to twisting of the bar is
measurement of deflection which is of the order of
measured with the help of a displacement trans­
1 mm or so. Another method is to use strain gauges for
ducers. In others, the strain in the surface of the bar,
measurement of strain caused by the applied force.
which is proportional to the torque, is measured with
The strain, then can be used to compute the applied
the help of strain gauges. The shear strain is a measure
of the torque. force.
Angle of twist 0 = 16T/7tGd3 rad, ...(25.9) 25.3.6 Load Cells
where T - applied torque ; Nm, Load cells utilise an elastic member as the primary
2
G = shear modulus ; N/m , transducer and strain gauges as secondary trans­
d = diameter of bar ; m ducers as shown in Fig. 25.6.
Primary Sensing Elements and Transducers 743

These elastic members are of many types and convert


the pressure into mechanical displacement which is
later converted into an electrical form using a
secondary transducer. These devices are many a time
known as force summing devices. Figure 25.7 shows
some of the commonly pressure sensitive primary devices.
The principle of working of these devices is
explained as : the fluid whose pressure is to be
measured is made to press the pressure sensitive
element and since the element is an elastic member, it
deflects causing a mechanical displacement. 'The
displacement is proportional to the pressure applied.
The displacement is then measured with the help of
electrical transducers. The output of the electrical
Fig. 25.6 Load cells. transducers is proportional to the displacement and
hence to the applied input pressure.
25.3.7 Spring Flexture Pivots
The commonly used pressure sensitive devices
Figure 25.1 shows two different types of arrange­
are described below :
ments for flexture pivots, one for the single spring
flexture pivot and the second for crossed spring flexture 25.4.1 Bourdon Tubes
pivot. The crossed-spring flexture pivot is widely used The Bourdon tubes are designed in various forms.
in measurement work for the following reasons : These are :
(i) it is practically frictionless (z) C type (zz) Spiral
(zz) the pivot sensitivity i.e., the angular deflec­ (zzz) Twisted tube and (zv) Helical.
tion per unit applied torque is virtually The Bourdon tubes are made out of an elliptically
constant for angular relations less than 15°. flattened tube bent in such a way as to produce the
shapes shown in Fig. 25.7. One end of the tube is
25.4 PRESSURE SENSITIVE PRIMARY DEVICES sealed or closed. The other end is open for the fluid to
Most pressure measuring devices use elastic enter. When the fluid whose pressure is to be
members for sensing pressure at the primary stage. measured enters the tube, the tube tends to straighten

Fig. 25.7 Pressure sensitive primary devices.


744 Electrical and Electronic Measurements and Instrumentation

out on account of the pressure applied. This causes a is oval in section is formed into an arc of 250° and
movement of the free end (closed end) and the hence the name C for the configuration which is
displacement of this end is amplified through shown in Fig. 25.8.
mechanical linkages. The amplified displacement of The displacement of tip is,
the free end may be used to move a pointer on a scale
3
calibrated in terms of pressure or may be applied to a x
electrical displacement transducer whose output may
be calibrated in terms of the pressure applied. 2
where E is the modulus of elasticity ; N/m , P, the
Bourdon tube elements have several distinct advan­ applied pressure, N/m and other terms are as
tages and these include low cost, simple construction, depicted in Fig. 25.9.
high pressure range, good accuracy except at low
pressures, and improved designs at the pressure for
maximum safety. Their greatest advantage is that they
are easily adapted for designs for obtaining electrical
outputs.
The disadvantages of Bourdon tubes are : their
low spring gradient which their use limits for pre­
cision measurements upto a pressure of 3 MN/m2, are
susceptible to shocks and vibrations and are subject to
hysteresis.
The Bourdon tube has the following forms :

1. C Type
The C type of Bourdon element is most commonly
used for local indication but it is also used for pressure Fig. 25.9 Geometry of C type Bourdon tube.
transmission and control applications. The tube which
Thus relationship between the displacement of tip
and the applied pressure is non-linear i.e., each
pressure increment does not produce a corresponding
tip movement. However, a linear relationship between
the deflection of the pointer and the applied pressure
is obtained using the geared sector and pinion
movement or other mechanical means. The accuracy
of C type Bourdon tubes varies from ±0.5 to ± 2% or
poorer. However, the normal accuracy is about ± 1%.

2. Spiral Type
It is clear from Eqn. 25.11, that the displacement of
tip varies inversely as the wall thickness and depends
upon the cross-sectional form of the tube. It also varies
directly as the length of arc 'a' which in turn depends
upon the angle subtended by the arc through which it
is bent. Thus in a tube having an arc of 180°, the
displacement of the tube will be twice that of a similar
tube having an arc of 90°. Therefore, the displacement
of the free end (tip) may be increased by increasing the
length of the arc of the tube without changing the wall
thickness.
When the angle through which the arc is bent
reaches 360°, its length can be increased further in two
ways : the tube can be made in the form of a spiral, or it
can be made in the form of a helix. An increased
Primary Sensing Elements and Transducers 745

displacement of the free end can be obtained by


increasing the number of turns in the spiral or helix
thereby avoiding the need for further magnification. It
has been mentioned earlier that magnification is
obtained through the use of geared sector and pinion
is used in C type Bourdon tube. But while using spiral
or helix type Bourdon tubes there is no need for
further amplification and hence the geared sector and
pinion arrangement is not used in their case. The
absence of geared sector and pinion arrangement
eliminates the backlash which tends to occur when
they (geared sector and pinion) become worn owing
to continued use.
A spiral type Bpurdon tube is shown in Fig. 25.10.
Spiral tubes are made by winding several turns of the
tube with its flattened cross-section in the form of a
spiral. Fig. 25.11 Helical type Bourdon tube.

The advantages of helix elements include the high


over-range capabilities which may be in the ratio as
high as 10 :1, its stability in fluctuating pressure appli­
cations, and its adaptability for high pressure service.
The number of coils employed in helix elements
depends upon the pressure to be measured. Helix type
of pressure elements use as few as three coils while
elements used for measurement for high pressures
may have as many as 16 coils or even more. The
accuracies obtainable from helical elements may vary
from ± 0.5% to ± 1% of span.
Bourdon tubes are made of different materials
which include brass, alloy steel, stainless steel, bronze,
Fig. 25.10 Spiral type Bourdon tube. phosphor bronze, beryllium copper, K Monel, Monel
and Ni-span C. The choice of type of material to be
When the pressure to be measured is applied to used depends upon its elastic characteristics suitable
the spiral, it tends to uncoil producing a relatively for the pressure range, process medium, temperature
long movement of the tip whose displacement can be and corrosive resistance to the media to be used.
used for indication or transmission. Phosphor bronze is used in low pressure applica­
The accuracy of spiral tube elements is higher tions where the atmosphere is non-corrosive while in
than that of C type elements on account of absence of applications where corrosion and/or high pressure is a
magnifying elements and is typically about ± 0.5%. problem, stainless steel or Monel are used.
Pressure gauges using bourdon tube elements are
3. Helical Type made with ranges from 760 mm of mercury to
A helical type Bourdon tube is shown in Fig. 25.11. 700 MPa or higher for special applications with the
As explained earlier, a helical bourdon element is minimum span being about 70 kPa. In fact, there is
similar to the spiral element, except it is wound in the quite an overlap of ranges at which the various
form of a helix. The displacement of the tip of a helical bourdon tube elements are designed to operate and
element is larger than that of spiral element. Usually a these ranges differ from one manufacturers, to
central shaft is installed within a helical element and another. One manufacturer, for example, lists spiral
the pointer is driven from this shaft by connecting bourdon tube elements from 0-75 to 0-1500 kPa
links. This system transmits only the circular motion when bronze, beryllium copper and Ni-span C are
of the tip to the pointer which is directly proportional used as the materials for the element. The same
to the changes in pressure. manufacturer uses the aforesaid materials for pressure
746 Electrical and Electronic Measurements and Instrumentation

ranges 1.5-550 MPa while using helical elements. For The displacement of bellows element is given by,
bronze, the upper span limit is 0-30 MPa ; for
, 0.453 Pbn D271-v2
beryllium copper and Ni-span C, 0-40 MPa and for «= ---------------5---------- ...(25.12)
stainless steel 0 -600 MPa. £t3
Another manufacturer uses C type bourdon tube
where P = pressure, N/m2 ;
elements using bronze upto a pressure of 7 MPa and
b = radius of each corrugation, m ;
steel and stainless steel to 150 MPa and still another
which uses these elements upto a pressure of 700 MPa. n = number of semi-circular corrugations ;
It may therefore be concluded that C-type bourdon t = thickness of wall, m ;
tube elements may be used for measurement of D = mean diameter, m ;
pressures ranging from vacuum to about 700 MPa and E = modulus of elasticity, N/m2 ;
even higher, spiral elements from vacuum to 30 MPa and v = Poisson's ratio.
and helical elements from 1.5 MPa to 550 MPa.
Normally a Bellows has the ability to move over a
25.4.2 Bellows greater distance than required in a pressure application
A metallic bellows is a series of circular parts, and, therefore, to give it maximum life and to have
resembling the folds in an accordian as shown in better accuracy, its movement is generally opposed by
Fig. 25.12. These parts are formed or joined in such a a calibrated spring so that only a part of the maximum
manner that they are expanded or contracted axially stroke is used. This system is shown in Fig. 25.13 and
by changes in pressure. The metals used in the is called spring loaded bellows.
construction of bellows, must be thin enough to be
flexible, ductile enough for reasonably easy
fabrication, and have a high resistance to fatigue
failure. Materials commonly used are brass, bronze,
beryllium copper, alloys of nickel and copper, steel
and Monel. Most of the bellows used in pressure
gauges are seamless and are made from drawn tubing
by hydraulic or other methods of rapid forming. These
methods produce uniform walls that give a higher life
expectancy. Other methods such as soldering and
welding of annular sections, rolling, spinning and
turning from solid stock may also be used for
manufacture of bellows.

Deflection of bellows when opposed by a spring is,


d=P—...(25.13)
2
where = effective area of bellows, m ;
K,,
b'sK = stiffness constants of bellows
2 and
spring respectively, N/m ,

.. p
Pressure Pp - d/— P +K^
K-------- ...(25.14)
4
If the bellows assembly operates an electric switch
or some other mechanism, we have,
p = F + ds(Kb + Ks)
...(25.15)
4
where F = force required to operate switch or
mechanism, N ;
d = deflection required to operate switch
or mechanism, m
Fig. 25.12 Bollows element.
Primary Sensing Elements and Transducers 747

There are three main configurations in which The disadvantages of bellows are that are not
bellows elements are used and these are for suited for dynamic measurements on account of their
measurement of absolute, gauge and differential greater mass and longer relative movement. Also they
pressures. These configurations may be illustrated need temperature compensating device to avoid
with the help of Fig. 25.14. errors resulting from changes in ambient temperature.

25.4.3 Diaphragms
The operating principle of diaphragm elements is
similar to that of the bellows. The pressure to be
measured is applied to the diaphragm, causing it to
deflect, the deflection being proportional to the
applied pressure. The movement of the diaphragm
depends on its thickness and diameter. The movement
is small and hence a diaphragm element does not
require any springs as is the case in bellows.
The movement of a diaphragm is a convenient
way of sensing a pressure. The unknown pressure is
applied to one side of a diaphragm. The edge of the
diaphragm is rigidly fixed and causes a deflection on
account of the applied pressure. The displacement of
the diaphragm may be measured to determine the
value of applied pressure, P.
The diaphragm, which is a very thin membrane
under pressure and therefore under radial tension,
may use capacitive or inductive transducers, to
produce an electrical output which is proportional to
Fig. 25.14 Measurement of pressure with bellows.
the output of the transducers. The use of diaphragms
There are two bellows, A and B. The pressure using thin membranes is limited to measurement of
applied to bellows A is P} and that to bellows B is P2. low level pressure fluctuations. Since membranes can
In case it is desired to measure the absolute pressure, withstand only limited values of force (produced by
bellows B is evacuated and the resultant pressure, Pj pressure), diaphragms using thin membranes are
is the absolute pressure. When measurement of gauge suitable for resistance and piezoelectric transducers
pressure is desired, bellows B is opened up to which require greater displacements than are
atmosphere with pressure P2 is equal to the produced by membrane type of diaphragms.
atmospheric pressure and therefore the reading of the Therefore, in place of membranes, it is usual to employ
gauge is the gauge pressure. The measurement of thin circular plates which are either clamped, around
differential pressure P = P1 - P2 is done when pressure
their circumference between two solid rings, or are
P] is applied to bellows A and pressure P2 is applied to machined from a solid piece of metal. In order to
bellows B. maintain linear relationship between deflection of
The advantages of bellows include their simple plates and applied pressure, the deflection of the
and rugged construction, moderate price, their centre of plate should be less than half its thickness or
usefulness for measurement of low, medium and din < 0.51 which are essentially flexible discs.
high pressures, and their applicability for use in The diaphragms, are of two types :
measurement of absolute, gauge and differential
(zj Flat type and
pressures. Bellows elements, like many other
elements have been greatly improved over the past (zz) Corrugated type.
few years. The reduction of drift and hysteresis These are shown in Fig. 25.15.
allows their use in functions requiring ± 0.5% of full
span accuracy. They deliver relatively high forces Corrugated diaphragms, on account of their
and are well adapted to vacuum and low pressure increased surface area, give greater deflections,
measurements. typically 2 per cent of the diaphragm diameter.
748 Electrical and Electronic Measurements and Instrumentation

Lowest natural frequency (for air or gas medium)

% = 201 ,----- ^-5-


E rad/s .. .(25.20)
" D2y3p(l-v2)
a
where p = density of diaphragm material, kg/m
Design equations for the flat diaphragm may use
the following data :
Young's modulus for steel
(b) Corrugated type E = 206 G N/m2 ; v = 0.28
and p = 7800 kg/m3
Fig. 25.15 Single diaphragm elements.
The above relationships are valid for uniform
A flat diaphragm is shown in Fig- 25.16. pressure loading conditions over the entire surface of
the disc and also when the deflection smaller than half
the thickness of disc. Therefore, the response is linear
only when d < 0.5 t. In order to have a linear response
over greater deflections than d > 0.5 t, the diaphragms
are corrugated. A corrugated diaphragm is most
suitable for applications where a mechanical device is
used for measurement of deflection. This is because it
(corrugated diaphragm) gives a relatively greater
output and if combined with a high magnification
Fig. 25.16 Flat diaphragm.
linkage, can be used for direct operation of mechanical
256 Et3 d , indicators.
Pressure P =-------- -—N/m2 ...(25.16)
3(l-v2)D4 In some cases, a diaphragm element may consist
of a single disc ; which in others, two diaphragms are
where E = Young's modulus ; N/m2,
bonded together at their circumference by soldering or
t - thickness of diaphragm ; m, pressure welding to form a capsule. A diaphragm
D = diameter of diaphragm ; m, element may consist of one capsule or two or more
R = radius of diaphragm ; m, capsules connected together with each capsule
deflecting on the application of pressure. The total
v = Poisson's ratio,
deflection is the sum of the deflections of individual
and dm = deflection at the centre of the capsules. Figure 25.17 shows as diaphragm element
diaphragm ; m consisting of three capsules. In this assembly, the
The above relationship between pressure, P and individual capsule is connected axially with the next
the deflection at the centre, dm is linear. But linearity one and is allowed to expand without any restraints.
holds good as long as dm < 0.5 t and not otherwise. The element may be provided with stops for
The deflection at the centre is : over-range and under-range protection.

3P(l-v2) 4
= —-—~R ...(25.17)
16 Et3

The deflection at a radius r from the centre is :


J 3P(l-v2)/ri2
df= l6Et3 ( -(25.18)

Maximum stress at the circumference


. 3 D2p x, , 2
S™=^2-N/m ...(25.19)
Fig. 25.17 Diaphragm element using three Capsules.
Primary Sensing Elements and Transducers 749

Two different arrangements are used for capsular laminar. Viscosities may vary. Pressures may vary from
elements, the convex and the nested type as shown in near vacuum to many times atmospheric, and the
Fig- 25.18. temperatures may vary from cryogenic to hundreds of
degrees of Celsius.
The characteristics and properties which need to
be considered are viscosity, density, compressibility,
temperature and pressure.
In most applications, the knowledge of type of
flow, whether laminar and turbulent is very important.
All liquids have a force of friction, called viscosity;
which tends to inhibit the formation of turbulent
eddies and vortices, and maintain streamline flow.
Viscosity enters into determination of flow constants,
particularly the discharge co-efficient C which is a
function of Reynold's number.
The Reynold's number is given by :
Fig. 25.18 Types of capsules. Re=udp/q ...(25.21)
Materials used for diaphragms include phosphor where v = velocity of flow ; m/s,
bronze, stainless steel, beryllium copper, Ni-span C, d = diameter of pipe throat or orifice ; m,
Inconel, Monel and nickel. Non-metallic materials are p = density of fluid ; kg/m
used for some applications. Buna N rubber, nylon and and q = viscosity ; Ns/m .
Teflon are used in environments that corrode metals.
The Reynold's number is dimentionless, and a
The diaphragm elements find extensive use in
useful criterion by which to compare flows in
applications where measurement of low pressures
geometrically similar installations but with different
including vacuum is involved. The ranges are 0-50 flow conditions. It provides information regarding
N/m2 to 0 -200 kN/m2. Accuracies range from ± 0.5%
conditions where streamline flow ceases and turbu­
to ± 1.25% of full span.
lent flow begins. It has been found by experiments
25.5 FLOW RATE SENSING ELEMENTS that the flow is streamline or laminar for Reynold's
number less than 2000 and turbulence sets in
25.5.1 Introduction increasingly as Reynold's number increases beyond
The measurement of flow is very important in a 2000. Flow becomes completely turbulent for
large number of industries. Flow measurements Reynold's number greater than 105.
account for a high percentage of the process variables Flow measurements are concerned with the
measured in the chemical industries. Flow is the most following :
important variable in plant operation. The plant material (z) velocity of flow ; m/s,
balances, quality control and continuous processes 3
(zz) volumetric flow rate ; m /s,
will be impossible without flow measurements.
(zzz) mass flow rate ; kg/s,
Flow of material in a process or system can be 3
(zu) total volumetric flow ; m , over a set period
measured by a variety of methods depending upon
of time,
the material and its condition, the type of flow, the
volume and the mass of the quantity flowing, the (u) total mass flow; kg, over a set period of time.
range and rangeability, the pressure and temperature 25.5.2 Types of Flow Rate Meters
and the accuracy required. Many accurate and reliable
The flow rate meters can be classified into two
methods are available for the measurement of flow.
categories :
Some of these methods are applicable only to liquids,
some only to gases and vapours and some to both. (i) Inferential meters
Fluids, may be of different types, like they may be (zz) Absolute or positive displacement meters.
clear or opaque, clean or dirty, wet or dry, erosive and In inferential meters, the flow is not directly
corrosive. Fluid streams may be multiphase, vapour, measured but is inferred from measurements of other
liquid or slurries. The flow may be turbulent or quantities which are related. Inferential methods of
750 Electrical and Electronic Measurements and Instrumentation

measurement are based upon differentia! pressure or B = expansion ratio,


head obtained from orifice plate, a venturi tube, a pitot = area at point Y ; m2,
tube or a weir. Flow rate can also be inferred from Ax = area at point X ; m2,
turbine and electromagnetic flow meters.
px = density of fluid at point X, kg/m3,
AT = pressure drop between pts. X and Y; N/m"

For liquids, the fluid density can be considered to


remain constant and therefore expansion ratio B can
be taken as unity. With compressible fluids such as
gases and vapours, the density does not remain
constant and the value of B is less than unity and is a
function of ratios of pressure and areas at points X and
Y and the gas specific ratio.
The pressure drop AP can be measured with a
manometer and from there the input flow rate can be
inferred.
Orifice plates, venturi tubes and nozzles provide a
simple means of inducing a pressure drop. The flow
rate is proportional to the square root of the pressure
drop and hence there is a non-linear relationship
between output (pressure drop) and input (flow rate).
The discharge co-efficient for an orifice plate is
about 0.6 for Reynold's numbers above 10 and the
Fig. 25.19 Flow meters based upon producing permanent pressure loss is 60-70 percent of diffe­
pressure drop using constrictions. rential pressure AP. Therefore, an orifice plate which is
The purpose of this section is to briefly consider easy and cheap to install, may cause six or seven times
the flow rate measuring devices with pressure related the pumping power as compared to a venturi tube
techniques (inferential in nature) which are widely which has a discharge co-efficient of 0.96 for Reynold's
used. The two basic techniques used are : numbers above 10 and less tendency to trap suspended
(?) a pressure drop is produced by means of a material in the fluid being metered. However, venturi
constriction, tubes occupy greater space and are costlier.
(ii) the kinetic energy of the flowing fluid is Pitot tubes are one of the most common forms of
transformed into potential energy in the flow rate measuring devices which work on the
form of a static head. principle of converting kinetic energy of the fluid
Two different forms of producing constriction are being metered into a static head. Pitot tubes are
shown in Figs. 25.19(a) and 25.19(b). Figure 25.19(a) essentially laboratory type of instruments are not
shows a circular orifice plate while Fig. 25.19(b) shows normally used for industrial applications. Figure 25.20
a venturi tube.
The fundamental equation for any flow rate sensing
element which changes the local velocity by using a
constriction or reduction in area of flow is given by :
Q=CEBXy1/2AP/pI ...(25.22)
where Q — flow rate, m/s,
C = co-efficient of discharge
actual flow rate
theoretical flow rate
E= / A*. )2 = vel°city approach factor

...(25.23) Fig. 25.20 Pitot tube.


Primary Sensing Elements and Transducers 751

shows a pitot tube installed in a pipeline for they require mean velocity to be determined by
measurement of flow-rate. The 'tube' actually consists making transverse measurements across the diameter
of two concentric tubes. The open end of the inner of pipe and hence they cannot be used for industrial
tube faces the incoming flow. The outer tube has a applications which require an instant readout. The
closed end, but has a number of holes in its walls. Both other disadvantages are that they produce a low
the tubes contain the same fluid as is flowing in differential head on account of which their sensitivity
the main pipeline. The pressure in the outer tube is the is low, and they require high flow velocities of about
static pressure in the pipeline. The total pressure in 15 m/s to produce measurable heads. The small
the inner tube, however, is greater than this static opening of Pitot tubes may get clogged if fluids
pressure. The additional pressure on the inner tube is containing solid particles are used which may give
on account of the impact of the fluid stream on the rise to wrong results.
stationary inner tube. Weirs and flumes are commonly used devices for
Application of Bernoullis' theorem shows that the open channel measurements. A detailed description of
velocity at this point in the cross-section is : open channel measurements is beyond the scope of
this text. The purpose here is to acquaint the reader
v = J—- ----- ...(25.24) with this type of measurements.
K p )
2 (i) Weirs
where PQ = static pressure ; N/m ,
2 A weir is essentially a dam with a notched
P = pressure at the entrance of the tube ; N/m , opening at the top through which liquid flows as
3
and p = density of liquid ; kg/m shown in Fig. 25.21.
Thus, the velocity at a particular point may be
determined from the pressure differential generated
by the pitot tube, which is measured by an inclined
tube manometer.
The velocity of a fluid across a section of pipe is
not uniform but varies from zero at the pipe surface to
a maximum usually at the centre of the pipe. In order
to find the mean velocity it is necessary to make a
traverse of the pipe i.e., readings are taken by placing
the Pitot tube at various positions at different points
along the cross-section of the pipe. For Reynold's
numbers above 10 , the ratio of mean velocity to the
Fig. 25.21 Rectangular Weir.
velocity at the centre of pipe is 0.82. Once this ratio is
determined exactly for a given situation, the Pitot tube The weirs make use of rectangular V, and
can be placed at the centre of pipe to sense the trapezoidal notches as shown in Fig. 25.22. The rate of
velocity. The mean velocity is, flow is simply measured by measuring the head of
water above the lowest point of the weir opening
28
J-(p-po) through which the liquid flows. This height is
measured by means of a float installed in a box called
[2 stilling well, which is a part of the total structure. The
=C -(P-Po) ...(25.25) float is so placed that it is not disturbed by the velocity
VP
of flow or by turbulence of the stream.
where Cv is the velocity correction factor and is the ratio The discharge through a rectangular notch is,
of mean velocity to the velocity at the centre. 7 _
Q=3CrfL^jH15 ...(25.26)
The advantages of Pitot tubes are that they
produce a negligible loss of head when inserted in the
The co-efficient of discharge, C^, varies with the
pipe and their cost is much smaller than that of venturi dimensions, of the weir and is usually less than 0.7.
meters or orifice meters. The rectangular notch is the most commonly used
The major disadvantage of Pitot tubes is that they notch on account of its simplicity, ease of construction
are laboratory type instruments as for their operation and accuracy.
Electrical and Electronic Measurements and Instrumentation

Fig. 25.23 Venturi flume.

The discharge can be calculated as in the case of a


venturi meter. The actual discharge is,

...(25.29)

= CMy]2gh ...(25.30)

where h - h1 -h2
Fig. 25.22 Types of Notches. M=A2/yll + (A2/A2)2

The discharge through a V notch is, C = venturi flume co-efficient (0.95 to 1)

Q=^c^H2'5‘an| ...(25.27) The maximum value of flow occurs when


h2 = f-) h.
The co-efficient of a discharge varies with notch
2 Uj
angle 0, head H, kinematic viscosity and surface
tension of liquid. Typical values of Cd are 0.58 to 0.7. Maximum discharge is given by,
The V notch offers the widest range for a single Q
^max
= 1.7 2
H1'5 ...(25.31)
size, the small opening at the lower end can
accommodate small flows while the top portion is for
large flows. However, it gives the highest loss of head
because of its shape.
The trapezoidal notch is a combination of the
rectangular and triangular notches as shown in Fig.
25.22(c). The discharge is given by :
9 Q f)

3 13 Z

...(25.28)
mbmwmk
Applications of weirs are primarily in water Fig. 25.24 Elevation and plan of flume.
works and waste and sewage systems. Measurements
can be made from a few litres per minute to millions of Flumes are capable of better accuracies than weirs
litres per day. because the loss of head in improved versions of
flumes is about one fourth of that of weir.
(if) Flumes Weirs and flumes have not previously found
Figure 25.23 shows a venturi flume. It is an open application in process industries, but their use is
channel whose sides are arranged sharply to form a increasing with the increasing demand for water
throat, followed by a fairly long expansion section. pollution control. These measurements are a must
Stilling wells are provided for measuring the head at when large volumes of water are handled in waste
both entrance and throat as shown in Fig. 25.24. treatment facilities.
Primary Sensing Elements and Transducers 753

Example 23.1 A C type bourdon tube made of Monel Example 23.3 A flat circular diaphragm of mild steel
metal is show in Fig. 25.9. The dimensions of the tube are: has a diameter of 15 mm. For mild steel, Young's modulus =
r = 36.5 mm, x = 16 mm, y = 3 mm, t = 0.35 mm 200 GN/m and Poisson's ratio v = 0.28. Find the thickness
of the diaphragm if the maximum stress is not to exceed
Calculate the displacement of the free end if a pressure of 300 MN/m when the applied pressure is 300 kN/m . Find
1500 kPa is applied. The modulus of elasticity for Monel the deflection at the centre for a pressure kN/m .
metal is 180 GN/m2.
Also calculate the natural frequency of the diaphragm if
Solution. From Eqn. 25.11, the displacement of the density of mild steel is 7800 kg/m3.
the tip is given by,
Solution. From Eqn. 25.19, maximum stress,
n / \0.2
a aP I -r I
n nc
Aa = 0.05 — _3D2P
E UJ w 16 t2
71 x36.5xl0“3x 1500xlO3
= 0.05 x---------------------------------- Thickness of diaphragm
180 xlO9
f36.5f'2 (16f2 f 16 f s_ l3D2P _ I3x(15x10~3)2x30x103
<0.35 J I 3 ) <0.35 J yi6S,„ V 16x300xl0 6

= 0.02 m =2 mm = 0.205 xlO-3 m =0.0205 mm

Example 23.2 A differential bellows arrangement for From Eqn. 25.17, the deflection at the centre for a
measuring absolute pressure is shown in Fig. 25.25. It uses pressure of 150 kN/m is,
two bellows each of natural length 50 mm, effective area = 3P(l-v2)
1500 mm , and stiffness = 0.5 N/mm. Bellows A is w 256 Et2
evacuated and a contains a spring of stiffness 3 N/mm. Find
d4 _ 3x 150x 1Q3(1-0,282)x (15x 1Q~3)4
the required natural length of the spring if the bellows are to
be equally compressed to a length 40 mm when a pressure of 256 x 200 x 109 x (0.205 x 10-3)3
100 kN/m absolute is applied to B. Also find the = 47.6 x 10~6 m = 0.0476 mm
displacement of the output point Cfor a change of 10 kN/m
The natural frequency is given by Eqn. 25.20,
in applied pressure.
Spring 20 t | E
n D2 pp(l-v2)
20xQ205xlQ~3 I 200 xlO9
(15xl0“3)2 V 3x7800 (T-0.282)
= 51950 rad/s = 8.3 kHz
Example 23.4 A mild steel torsion bar of 30 mm
bellows, A diameter is used for measurement of a torque of 100 Nm.
Calculate the angle of twist if the shear modulus of mild
Fig. 25.25 Bellows arrangement for measurement
of absolute pressure. steel is 80 GN/m2.

Solution.
Solution. From Eqn. 25.9,
2 Angle of twist
When a pressure of 100 kN/m is used.
Force applied 0= 16T ________ 16x100_______
F = PA = 100 x 103 x 1500 x 10”6 = 150 N nGd3 ~ 7ix 80x 109 x (30x 10-3)3

:. Compression of spring = 150/3 =50 mm = 0.236xlO-3 rad =0.81'


Thus natural length of spring = 50 + 40 = 90 mm Example 23.3 A venturi tube of throat diameter 60 mm
2
When a pressure of 10 kN/m is applied, the force is, is placed in a water pipe of diameter 100 mm to measure the
F = lOx 103 x 1500x 10-6 = 15 N volumetric flow. The volumetric flow rate through the tube
is 0.08 m3/s and the water has a density of 103 kg/m3 and
Total stiffness of spring plus stiffness of bellows viscosity of 10~3 Ns / m2.
= 3 + 2 x 0.5 = 4 N/mm
(a) Determine the Reynold's number for these
Displacement of point C = 15/4 = 3.75 mm conditions.
754 Electrical and Electronic Measurements and Instrumentation

(b) The co-efficient of discharge is 0.99, determine the Solution, (a) For a Pitot tube the velocity,
upstream-to-throat differential pressure.
(c) The differential pressure calculated in (b) is
applied across an unstressed flat diaphragm. The
diaphragm is made of mild steel for which
Young's modulus is 206 GN/m and Poisson's
ratio = 0.28, and has a diameter of 10 mm and a
thickness of 0.2 mm. Determine the deflection of
the diaphragm.
Solution.
(fl) Area of throat
A correction to the result obtained above has to be
= (n / 4) x (60 x 10-3 )2 = 2827 x 10~6 m 2
applied for taking into account the Mach number. The
Velocity through throat Mach number is the ratio of the velocity of an object
moving through a gas, to the velocity of sound in air.
0-08 ,
v =------------- - = 28.3 m / s The velocity of sound in air at atmospheric pressure is
2827 xl0“6
about 330 m/s, and therefore the Mach number for the
From Eqn. 25.21, Reynold's number is, air velocity being measured is 175.4/330 - 05. At this
subsonic Mach number, a small correction of about
n vdP 28.3x60xl0~3xl0~3 .
Re =----- =----------------------------- = 1.7 x 106 3 per cent is applied. Therefore, the correct velocity of
H 10~3
air is (1 -0.03)x 175.4 = 170 m/s.
(b) From 25.23, velocity approach factor, Example 25.7 Water is flowing with a depth of 0.9 m
E = ^1 -(4 / ^ )2 = Vl~(60/100)2 = 1.25 over a rectangular notch 1.2 m wide. Find the depth offlow
over a right angled notch if the same were to pass over it.
From Eqn. 25.22, flow rate, Assume that both the notches have the same co-efficient of
discharge.
Q = CEB
Solution. For rectangular notch, the discharge is,
The value of expansion factor B for liquids is 1

.-. Q = 0.99x 1.25x 1 x—(60x 10-3)2 = 0.08


4 VlxlO3 2 ____
= - crfx 1.2 xVTx 9.81 (0.9)1-5
.•. Differential pressure
AP = 261 kN/m2 = 3.02 C,(7

(c) The deflection at the centre of the diaphragm is For a V notch, the discharge is,
given by Eqn. 25.17 « ,__ . a

_3P(1-u2)D4
” 256 Et3 Q
or 3.02 C, = — Cd 72x9.81 H25 tan 45°
_ 3x261x 103(l-Q.282)x (IQx 10~3)4
H= 1.1 m
256x206x 109x(0.2 xlO"3)3
= 17.1xl0-6m =17.1 pm Example 25.8 A right angled notch has a co-efficient of
discharge of 0.6. If head above the still is 0.5 ± 0.01 m,
Example 2,^.0 A Pitot tube is used for measurement of calculate the discharge.
velocity offlow of water having a density of 1000 kg/m .
Solution. For a right angled Vnotch, discharge is,
(a) Determine the velocity of flow at the head of the
pitot tube if it produces a differential pressure of
10 kN/m~ between its two outlets.
g
(b) The same differential pressure is obtained in air at = — X 0.6 72x9.81 X 0.52-5
an altitude where the density of air is 0.65 kg/m3.
Determine the velocity of air flow. = 0.2506 m3 / s
Primary Sensing Elements and Transducers 755

Now — 8 r-'
dQ- = — C, J2 x 2.5 irl.i
n cx H piezo-electrical crystal and so on. In all cases,
dH 15 * however, the electrical output is measured by standard
dQ, methods, giving the magnitude of the input quantity
or
Q H in terms of an analogous output.

or dQ = ±2.5x0.2506x0.01/0.5 25.7 ELECTRIC TRANSDUCERS


= ±0.0153 m3/s The art of electrical measurements has been
chiefly used for measurement of electrical quantities
Hence, the discharge can be written as 0.2506 ± but its value in making measurements of non­
0.0153 m3/s. electrical quantities in this new era of automation is
rapidly growing. In order to measure non-electrical
25.6 TRANSDUCERS quantities a detector is used which usually converts
An electronic instrumentation system consists of a the physical quantity into a displacement. This
number of components to perform a measurement displacement actuates an electric transducer, which
and record its results. As explained earlier a generalized acting as a secondary transducer, gives an output that
measurement system consists of three major is electrical in nature. The electrical quantity so
components. produced is measured by standard methods used for
(z) an input device, electrical measurements. The result (electrical output)
gives the magnitude of the physical quantity or
(zz) a signal conditioning or processing device, and
condition being measured.
(zzz) an output device.
The electrical signal may be a current or a voltage
The input device receives the measurand or the or a frequency and production of these signals is based
quantity under measurement and delivers a propor­ upon electrical effects which may be resistive,
tional or analogous electrical signal to the signal
capacitive, inductive etc. in nature.
conditioning device. Here the signal is amplified,
The first stage of a measurement system may
attenuated, filtered, modulated, or otherwise
modified in format acceptable to the output device. simply be called a transducer stage instead of detector
transducer stage by redefining a transducer. A trans­
The input quantity for most instrumentation
ducer, in general form, may be defined as a device
systems is a "non-electrical quantity". In order to use
which converts energy from one form to another.
electrical methods and techniques for measurement,
However, this definition has to be restricted, many a
manipulation or control, the non-electrical quantity is
time especially in the field of electrical instrumen-
generally converted into an electrical form by a device
tatiOfl. keeping this restriction in view, a transducer
called a "transducer". We can define a transducer as a
may be defined as a device which converts a physical
device which, when actuated transforms energy from
quantity or a physical condition into an electrical
one form to another.
signal. Another name for a transducer is pick up.
The broad definition of a transducer includes, for
example, devices which convert mechanical force into 25.7.1 AdvflnfagdS of Electrical Transducers
an electrical signal. These devices form a very large There are a number of transducers which trans­
and important group of transducers commonly used form a variety of physical quantities and phenomena
in industrial instrumentation area. The instrumen­ into electrical signals. The reasons for transforming a
tation engineers and technologists are therefore ph sical phenomenon into electrical form are
primarily concerned with this area of instrumentation. numerous. The advantages of converting physical
Many other physical parameters such as heat, intensity
quantities into analogous electrical quantities are :
of light, flow rate, liquid level, humidity and pH value
may also be converted into electrical form by means of (z) Electrical amplification and attenuation can
transducers. These transducers provide an output be done easily and that too with static devices.
signal when stimulated by a mechanical or a (z'z) The mass-inertia effects are minimised. In
non-mechanical input : a photoconductor converts fact, when dealing with electrical or electronic signals,
light intensity into change of resistance, a the inertia effects are due to electrons which have
thermocouple converts heat energy into electrical negligible mass. In many situations, we do not come
voltage, a force produces a change of resistance in a across mass or inertia problems at all.
strain gauge, an acceleration produces a voltage in a (zzz) The effects of friction are minimised.
756 Electrical and Electronic Measurements and Instrumentation

(iv) The electrical or electronic systems can be 25.8 CLASSIFICATION OF TRANSDUCERS


controlled with a very small power level. The transducers can be classified
(t?) The electrical output can be easily used, trans­ (z) on the basis of transduction form used,
mitted and processed for the purpose of measurement.
(ii) as primary and secondary transducers,
(vi) Telemetry is used in almost all sophisticated
(iii) as passive and active transducers,
measurement systems. The entire aerospace research
(iv) as analog and digital transducers, and
and development is based upon telemetry and remote
control. The ever-enlarging field of radio monitoring (v) as transducers and inverse transducers.
in space research has left us with no alternative but to 25.8.1 Classification based upon
resort to electronic means. This completely eliminates Principle of Transduction
the data transmission through mechanical means and
The transducers can be classified on the basis of
hence electrical and electronic principles have to be
principle of transduction as resistive, inductive,
employed for these conditions. The remote indication
capacitive etc. depending upon how they convert the
or recording is an essential part of modern day
input quantity into resistance, inductance or capaci­
instrumentation technology.
tance respectively. They can be classified as piezo­
(vii) There has been an explosive development in electric, thermoelectric, magnetorestrictive, electro-
the field of electronic components and devices. This kinetic and optical. Table 25.2 lists the classification of
development is on account of the fact that electronic transducers based upon principle of transduction.
devices are very amenable to miniaturization.
Components which are compact, have always an, 25.8.2 Primary and Secondary Transducers
advantage. Miniaturization on account of use of ICs Let us consider the case of a Bourdon's tube as
(integrated Circuits) has completely revolutionised shown in Fig. 25.26. The Bourdon tube acting as a
the field of instrumentation. primary detector senses the pressure and converts the
In short, it can be said that the reasons for pressure into a displacement of its free end. The
transforming a physical phenomenon into electrical displacement of the free end moves the core of a linear
form is that the electrical output can be easily used, variable differential transformer, (L.V.D.T.) which
transmitted and processed for the purposes of produces an output voltage which is proportional to
measurement. Modern digital computers make the the movement of the core, which is proportional to the
use of these transducers absolutely essential. displacement of the free end which in turn is
When the definition of transducer is confined to a proportional to the pressure. Thus, there are two
device that covers the entire detector transducer stages of transduction, firstly the pressure is converted
stage, wherein the transducer converts a non-electrical into a displacement by Bourdon tube then the
quantity into an analogous electrical signal, the trans­ displacement, is converted into an analogous voltage
ducer may be thought of consisting of two important by L.V.D.T. The Bourdon tube is called a "Primary
and closely related parts. These two parts are : Transducer" while the L.V.D.T. is called a "Secondary
A Sensing Element, and Transducer".
A Transduction Element.
In addition, there may be many other auxiliary
parts, such as amplifiers and other signal processing
equipment, power supplies, calibrating and reference
sources, and mechanical mounting features.
1. Sensing or Detector Element. A detector or a
sensing element is that part of a transducer which
responds to a physical phenomenon or a change in a
physical phenomenon. The response of the sensing
element must be closely related to the physical
phenomenon.
2. Transduction element. A transduction element
transforms the output of a sensing element to an Fig. 25.26 Measurement of pressure using Bourdon
electrical output. The transduction element, in a way, tube an L.V.D.T. (Example of primary and Secondary
acts as a secondary transducer. transducers).
Primary Sensing Elements and Transducers 757
Table 25.2 Types of Electrical
Electrical parameter
and class of transducer Principle of Operation Typical applications

_______________________ __________ Passive transducers (externally powered)


Resistance

Potentiometer device Positioning of the slider by an external force varies the Pressure, displacement.
resistance in a potentiometer or a bridge circuit.
Resistance strain Resistance of a wire or semiconductor is changed by elongation Force, torque,
: gauge or compression due to externally applied stress. displacement.
Pirani gauge or hot Resistance of heating element is varied by convection cooling of Gas flow, gas pressure.
wire meter a stream of gas. x
Resistance Resistance of pure metal wire with a large positive tempe­ Temperature,
thermometer rature co-efficient of resistance varies with temperature. radiant heat
i Thermistor Resistance of certain metal oxides with negative temperature Temperature, flow
coefficient of resistance varies with temperature.
■ Resistance hygrometer Resistance of a conductive strip changes with moisture content. Relative humidity.
. Photoconductive cell Resistance of the cell as a circuit element varies with incident Photosensitive relay.
light.
Capacitance

j Variable capacitance Distance between two parallel plates is varied by an externally Displacement, pressure.
; pressure gauge applied force.

j Capacitor microphone Sound pressure varies the capacitance between a fixed plate Speech, music, noise.
and a movable diaphragm.
Dielectric gauge Variation in capacitance by changes in the dielectric or Liquid level, thickness.
dielectric constant.
Inductance

| Magnetic circuit Self-inductance or mutual inductance of a.c. excited coil is Pressure, displacement.
transducer varied by changes in the magnetic circuit.
Reluctance pick up Reluctance of the magnetic circuits is varied by changing the Pressure, displacement, ■
position of the iron core of coil. vibrations, position.
Differential The differential voltage of two secondary windings of a Pressure, force,
transformer transformer is varied by positioning the magnetic core through displacement, position
an externally applied force.
Eddy current gauge Inductance of a coil is varied by the proximity of an eddy Displacement,
current plate. thickness.
Magnetostriction gauge Magnetic properties are varied by pressure and stress. Force, pressure, sound.
Voltage and Current

Hall effect pickup A potential difference is generated across a semiconductor plate Magnetic flux, current,
(germanium) when magnetic flux interacts with an applied power.
current.
1
j Ionization chamber Electron flow induced by ionization of gas due to Particle counting,
radio-active radiation. radiation.
Photoemissive cell Electron emission due to incident radiation upon Light and radiation.
photoemissive surface.
Photomultiplier tube Secondary electron emission due to incident radiation on Light and radiation,
photosensitive cathode. photosensitive relays.
Self-generating transducers (no external power)
Thermocouple and An emf is generated across the junction of two dissimilar Temperature, heat flow,
thermopile metals or semiconductors when that junction is heated. radiation.
Moving coil generator Motion of a coil in a magnetic field generates a voltage. Velocity, vibrations.
Piezoelectric pickup An emf is generated when an external force is applied to certain Sound, vibrations, accele­
crystalline materials, such as quartz. ration, pressure changes.
Photovoltaic A voltage is generated in a semiconductor junction device Light meter, solar cell.
when radiant energy stimulates the cell.
758 Electrical and Electronic Measurements and Instrumentation

Let us take another example which is the case of 25.8.3 Passive and Active Transducers
measurement of a compressive force with the help of a Transducers may be classified according to
load cell in conjunction with strain gauges as shown in whether they are passive or active.
Fig. 25.27. (Load cells and strain gauges are explained 1. Passive Transducers. Passive transducers
later in this chapter.) derive the power required for transduction from an
auxiliary power source. They also derive part of the
Force power required for conversion from the physical
quantity under measurement. They are also known as
"externally powered transducers". Typical examples of
passive transducers are resistive, inductive and
capacitive transducers.
Strain A typical example of a passive transducer is a
gauges
'POT' which is used for measurement of displace­
ment. A 'POT' is a resistive transducer powered by a
source voltage e- as shown in Fig. 25.28. This 'POT' is
used for measurement of linear displacement x;.

Force

Fig. 25.27 Force measurement with load cell


and strain gauges.
The load cell is a short column or a strut with
resistance wire strain gauges bonded to it. The
measurand, in this case, is a force and is applied to the
column thereby producing strain. The force is first Fig. 25.28 Linear potentiometer (POT),
a passive transducer.
detected by the column and is converted into strain
which is a mechanical displacement. The higher the Suppose L is the total length of potentiometer
force, the higher is the strain and thus the input signal whose total resistance K •. The input displacement is x^
(force) is converted into an analogous output (strain). :. Output voltage
This strain changes the resistance of the strain gauges.
Thus we have an output which is a change in the value S°=L
of resistance i.e., electrical in form. Hence, in this case,
it takes two processes to convert an input into an In the absence of external power, the transducer
analogous output. The first process involves con­ cannot work and it hence is called a passive transducer.
version of force into mechanical displacement which 2. Active Transducers. Active transducers are
is done by the column, while the second process those which do not require an auxiliary power source
involves conversion of mechanical displacement into to produce their output. They are also known as self
change of resistance which is done by strain gauges. generating type since they develop their own voltage or
current output. The energy required for production of
Thus we see that the force is detected by the
output signal is obtained from the physical quantity
column in the first stage and hence it is called a
being measured.
Primary Transducer. The output signal from the
Velocity, temperature, light intensity and force
primary transducer is converted subsequently into a
can be transduced with the help of active transducers.
usable output by the strain gauges and therefore they
These transducers include tachogenerators, thermo­
are known as Secondary Transducers.
couples, photovoltaic cells and piezoelectric crystals.
In most of measurement systems, there is a Consider the case of a piezoelectric crystal used for
suitable working combination wherein a Mechanical measurement of acceleration as shown in Fig. 25.29
device acts as a primary detector transducer and the The crystal is sandwiched between two metallic
electrical device acts as the secondary transducer electrodes, and the entire sandwich is fastened to a
with mechanical displacement serving as the base which may be the floor of a rocket. A fixed mass
intermediate signal. is placed on the top of the sandwich.
Primary Sensing Elements and Transducers 759

Glass scales can be read optically by means of a


acceleration
light source, an optical system and photocells. Metal
Fig. 25.29 Piezo-electric crystral measuring scales are scanned by brushes making electrical
acceleration - an active transducer. contact with individual tracks.
The property of the piezo-electric crystals is that The resolution depends upon the digits
when a force is applied to them, they produce an comprising the binary number and is 1/2/7 of full scale
output voltage. The mass exerts a certain force on where n is the number of digits.
account of acceleration on the crystal due to which a
voltage is generated. The acceleration is applied to the 25.8.5 Transducers and Inverse Transducers
base, due to which the mass produces a force. The There is a strong association of control with
mass being fixed, the force is proportional to measurement. The basic requirement for control of
acceleration. The voltage output is proportional force physical quantities such as position, speed,
and hence is proportional to acceleration (the mass temperature, pressure and flow rate in an industrial
being fixed). planet is the ability to measure these quantities. The
It should be noted from above that this transducer control action is only possible if the physical quantity
called "accelerometer" which converts acceleration into can be measured. For example, in a position control
electrical voltage does not need any auxiliary power system called "servomechanism", it is desired to control
source to convert a physical phenomenon (accele­ the position of a shaft. This requires an accurate
ration in this case) to an electrical output (voltage in method for measurement of the shaft position in order
this case) and therefore is an active transducer. that its position be accurately controlled. Further, if it
is desired that the shaft be accelerated in a controlled
25.8.4 Analog and Digital Transducers manner, then the position measuring device must be
The transducers can be classified on the basis of able to measure shaft position for rapid changes i.e.,
the output which may be a continuous function of the device must have a fast dynamic response.
time or the output may be in discrete steps. Figure 25.31 shows the block diagram of closed
1. Analog Transducers. These transducers convert loop control system. The controlled (output) quantity
the input quantity into an analog output which is a is usually a non electrical quantity. The control action
continuous function of time. Thus a strain gauge, an is through an input quantity that corresponds to the
L.V.D.T., a thermocouple or a thermistor may be
---------------( !--------------
called as "Analog Transducers" as they give an output
Feed- । 1 w
which is a continuous function of time. forward I
2. Digital Transducers. These transducers convert network Actuator
the input quantity into an electrical output which is in (amplifier) [ ।
the form of pulses.
As the binary system uses only two symbols 0 and
Measuring i
1 it can be easily represented by opaque and instrument M-
transparent areas on a glass scale or non-conducting 1 or transducer 1
i ।
and conducting areas on a metal scale. A scale
constructed to show the linear position on a movable --------- Electrical units or signals
object and having five digits is shown in Fig. 25.30.
---------- Mechanical units or signals
The complete binary number denoting position is
obtained by scanning the pattern across the scale at a Fig. 25.31 Feedback control system for control
stationary index mark. of non-electrical quantities.
760 Electrical and Electronic Measurements and Instrumentation

desired output (non-electrical quantity). The input


quantity called reference input is usually an electrical
quantity. The controlled quantity is measured and
converted into an analogous quantity by transducers
which form the feedback loop. The loop input
quantity (electrical in nature) is compared with the
electrical quantity proportional to the output in a
comparator. In case, the two are not equal an error
signal is produced. This error signal is amplified and
applied to an actuator in the forward path, which
--------- Electrical units or signals
corrects the output quantity till the output quantity
----------- Mechanical units or signals
reaches the desired level.
1. Transducers. A transducer can be broadly Fig. 25.32 Feedback control system for control of
electrical quantities using inverse transducer.
defined as a device which converts a non-electrical
quantity into an electrical quantity.
It is evident that the operation of the feedback is
2. Inverse Transducers. An inverse transducer is
similar for both control and measuring systems as
defined as a device which converts an electrical
regards accuracy and stability. The essential
quantity into a non-electrical quantity. It is a precision
differences are in power level of the systems and the
actuator which has an electrical input and a low power
performance requirements of transducing elements.
non-electrical output. A piezoelectric crystal acts as an
The control systems handle relatively high power
inverse transducer because when a voltage is applied
levels through the actuator to directly determine the
across its surfaces, it changes its dimensions causing a
mechanical displacement. output quantity. Therefore, the actuator may have
poor regulation without affecting the control action.
A current carrying coil moving in a magnetic field
The transducer and noise at the amplifier input largely
is also an inverse transducer because current carried
determine the accuracy of control. On the other hand,
by it is converted into a force which causes trans­
lational or rotational displacement. Many data the feedback measuring system involves use of fairly
indicating and recording devices are inverse trans­ low power devices at the output. These devices
ducers. An analog ammeter or voltmeter converts include the indicator and inverse transducer.
current into mechanical displacement. However, such However, the inverse transducer essentially
devices which include instruments like indicating determines the characteristics of the system. The noise
instruments, pen recorders, oscilloscopes that convert connected with the transducer and amplifier input
the electrical signals to a mechanical movement are stage is very important.
placed at the output stage (data presentation stage)
are called output transducers. 25.9 CHARACTERISTICS AND CHOICE OF
The most useful application of inverse TRANSDUCERS
transducers is in feedback measuring systems. The When choosing a transducer for any application
development of transducers and inverse transducers, the input, transfer and output characteristics have to be
and the advantages gained through use of feedback taken into account.
has increased their applications manifold and there is
reason to believe that they will continue to grow in 25.9.1 Input Characteristics
importance. 1. Type of Input and Operating Range. The
A block diagram of a feedback control systems foremost consideration for the choice of a transducer is
using inverse transducers is shown in Fig. 25.32 for the input quantity it is going to measure and its
comparison with the simple feedback control system operating range. The type of input, which can be any
shown in Fig. 25.31. In the measuring system, the physical quantity, is generally determined in advance.
output quantity (usually electrical in nature) is A physical quantity may be measured through use of a
converted to a non-electrical form suitable for number of transducers. However, the choice of a
comparison with quantity to be measured particular transducer that is selected for the purpose,
(non-electrical form). The resulting error signal is depends upon the useful range of input quantity over
usually transduced into electrical form and amplified which the transducer can be used. The useful
to give output indication. operating range of the transducer may be a decisive
Primary Sensing Elements and Transducers 761

factor in selection of a transducer for a particular The scale factor is defined as the inverse of
application. The upper limit is decided by the sensitivity and is therefore,
transducer capabilities while the lower limit of range
is normally determined by the transducer error or by scale factor = — ...(25.35)
S dqi
the unavoidable noise originating in the transducer. In
fact the transducer should maintain a good resolution Some manufacturers specify the scale factor and
throughout its operating range. still call it sensitivity.
2. Loading Effects. Ideally a transducer should 25.9.2B Error
have no loading effect on the input quantity being
The errors in transducers occur because they do
measured. In theory, it is impossible, although in
not follow, in many situations the input-output
practice steps may be taken to reduce the loading
relationship given by qo = ftqj. Any departure from
effects to negligible proportions. The magnitude of the the above relationship results in errors. For example,
loading effects can be expressed in terms of force, the output on account of input, z/(- has to be q0 but in
power or energy extracted from the quantity under practice an output q0 is obtained, then the error of the
measurement for working of the transducers. instrument is,
Therefore, the transducer, that is selected for a e = ^0-r/0 -.(25.36)
particular application should ideally extract no force,
The error, a can be expressed in terms of either
power or energy from the quantity under
input or the output quantity.
measurement in order that the latter is measured
The error analysis has already been given in
accurately.
Chapter 3. The error can be split into three
25.9.2 Transfer Characteristics components, which are,
The transfer characteristics of transducers require A scale error
attention of three separate elements, viz., A dynamic error, and
transfer function A error on account of noise and drift.
A error, and Scale Error
A response of transducer to environmental The scale error comprises of four different types
influences. of errors :
(i) Zero Error. In this case the output deviates
25.9.2 A Transfer Function
from the correct value by a constant factor over the
The transfer function of a transducer defines a
entire range of the transducer. This is shown in
relationship between the input quantity and the
Fig. 25.33.
output. The transfer function is,
% = /(?,) -(25.32)
where q0 and £/• are respectively output and input of
the transducer.
The sensitivity of a transducer is defined as the
differential quotient,
S = -^ ...(25.33)
dcli
In general, the sensitivity of transducers is not
constant but is dependent upon the quantity z/-. Fig. 25.33 Transducer zero error.
However, in some cases the relationship between
(zz) Sensitivity Error. Sensitivity error occurs
the output q0 and input z/(7 is linear. In that case the
where the observed output deviates from the correct
sensitivity remains constant over the entire range of
value by a constant value. Suppose the correct output
the transducer and is defined as, is q the output would be Kq^ over the entire range of
the transducer, where K is a constant. This is shown in
dr/. Fig- 25.34.
762 Electrical and Electronic Measurements and Instrumentation

Dynamic Error
Dynamic errors occur only when the input
quantity is varying with time. This is on account of the
fact the systems contain energy storage elements and
due to this the output cannot follow the input exactly
but with a time lag.
Let us consider an R-C series circuit to which a
step input of magnitude E is applied, the voltage
across the capacitor after a time, t of application of step
input is,
Fig. 25.34 Transducer sensitivity error.
ec = E[l-exp(-f/ t)]
(in) Non-conformity. This pertains to a case in
where r = time constant = RC
which the experimentally obtained transfer function
deviates from the theoretical transfer function for The response is shown in Fig. 25.37.
almost every input. This is shown in Fig. 25.35.

Fig. 25.35 Transducer Non-conformity.


Fig. 25.37 Response of RC circuit to a step input.
In the special case of a theoretical linear
relationship between input and output quantities, this
The dynamic or measurement error is,
error is called non-linearity or non-linear distortion.
(iv) Hysteresis. All transducers are subject to the e„;(0= E-ec=exp(-f/r)
effects of hysteresis. The output of a transducer not The dynamic or measurement error can be made
only depends upon the input quantity but also upon small by having a small time constant. It should be
in input quantities previously applied to it. Therefore, understood that the dynamic error is a function of
a different output is obtained when the same value of time. As the time after application of the input
input quantity is applied depending upon whether it increases, the dynamic error reduces. The steady state
is increasing or decreasing. For decreasing values, a
error,
greater output is obtained than with increasing values
eSS=e,n^=e~t/T' =0
for the same value of the input quantity. This is shown
in Fig. 25.36. t —> 00 t —> oo

in this particular case.

Errors due to Noise and Drift


Noise and drift signals originating from the
transducers vary with time and are superimposed on
the output signal. The difference between noise and
drift is that noise consists of a signal of random
amplitude and random frequency whilst drift is a slow
change with time. The magnitude of the noise and
drift is normally independent of the magnitude of the
Fig. 25.36 Transducer hysteresis. input signal.
Primary Sensing Elements and Transducers 763

Errors due to Change of Frequency The high frequency, cutoff (fh) is the frequency at
(Frequency Response) which the transducers gain has fallen to 0.707 of the
Frequency response and the high frequency cut off value it had at its stable low frequency. If the vertical
are the two specifications that describe the response of axis is scaled in db, fh occurs when the gain has
a transducer to a variable frequency sine wave input decreased by -3 db. Figure 25.38 shows that the low
applied to it. For a reasonably, linear transducer, a sine frequency gain is 20 db but drops of 17 db (20 - 3 = 17 db)
wave input yields a sine wave output. As the at a frequency of 4 kHz. Therefore, the high frequency
frequency of the sine wave input is increased, the cutoff takes place at 4 kHz in this case.
transducer is required to respond more and more
The high frequency cutoff can be related to the
quickly. Ultimately, beyond a particular frequency the
rise time by the following relationship :
transducer can no larger respond as rapidly as its
sinusoidal input is changing. So the output of the tr =0.35/fh ...(25.38)
transducer becomes smaller and also the phase shift Therefore, even if the transient response
between the input and output increases. Thus as the specifications are not given, the speed of a transducer
frequency increases the output of the transducer falls. can be determined from its high frequency cutoff.
This roll off of amplitude of output with an increase in
25.9.3 Transducer Response
The response of the transducer to environmental
influences is of a great importance. This is often given
insufficient attention when choosing the best
transducer for a particular measurement. This gives
rise to results that are not as accurate as expected, or,
worse, results that are accepted as more accurate than
they actually are. The performance of the tiansducer is
fully defined by its transfer function and errors,
provided that the transducer is in constant
environments and not subject to any disturbances like
stray electromagnetic and electrostatic fields,
mechanical shocks and vibrations temperature
changes, pressure and humidity changes, changes in
supply voltage and improper mechanical mountings.
If transducers are subjected to the above environ­
mental disturbances, which they are, precautions are
taken, so that changes in transfer function and
resulting errors therefrom do not occur.
Therefore, the transducer selected must be
guarded against the interfering and modifying
inputs.

Fig. 25.38 Frequency response plot. 25.9.4 Output Characteristics


The three conditions in the output characteristics
input frequency is the frequency response. The which should be considered are,
frequency response of a transducer is shown in
A Type of electrical output,
Fig. 25.38. The vertical axis is the gain of the
A Output impedance, and
transducer. This may be simply the output to input
ratio. However, the gain is specified in terms of A Useful ouput range.
decibels.
Type of Electrical Output
Gain in db = 20 log (output/input) = 20 log (^Ay,)
The types of output which may be available from
...(25.37) the transducers may be a voltage, current, impedance
The horizontal axis represents the frequency, and or a time function of these amplitudes. These output
is plotted on a 'logarithmic scale in order to cover a quantities may or may not be acceptable to the latter
wide range of frequencies. stages of the instrumentation system. They may have
764 Electrical and Electronic Measurements and Instrumentation

to be manipulated i.e., their magnitudes changed or


they may have to be changed in their format by signal
conditioning equipment so as to make them drive the
subsequent stages of instrumentation system.

Output Impedance
The output impedance, Zo, of a transducer
determines to the extent the subsequent stages of
instrumentation is loaded. Ideally, the value of output
impedance should be zero if no loading effects are
there on the subsequent stage. However, the output
impedance, Zo, cannot made equal to zero and
therefore, its value should be kept as low as possible to
minimise the loading effects.
The output impedance determines the amount of
power that can be transferred to the succeeding stages
of the instrumentation system for a given output
signal level. If the output impedance is low compared
to the forward impedance of the system, the
transducer has the characteristics of a constant voltage
source (provided the output of the transducer is a
voltage), while in case the forward impedance is high
as compared with the output impedance of
transducer, it behaves as constant current source.
This can be explained as under :
An ideal constant voltage source, furnishes at its
output terminals, a voltage that is independent of the and voltage across load
load connected to the source or of the current drawn
from the source. The voltage may or may not vary in ...(25.40)
‘L r0 + rl 1+r<,/rl
time. In fact, it must vary in time, if it represents a
signal which is true of transducers since their output is In case the internal resistance (output resistance)
dependent upon the input signals which vary with R(/ of the source is much smaller than load resistance
time. (for instance : Ro = 1 kQ and RL = 1 MQ) then ratio
R()/RL is very small as compared with unity and hence,
If a constant voltage source having a voltage of E
voltage across the load is,
is [Fig. 25.39(a)] is applied across a resistance R, the
current is E/R as shown in Fig. 25.39(c). A constant el=e ...(25.41)
voltage source in a strict sense does not exist ; the and the current is,
output voltage will always vary with the current 1= E/Rl ...(25.42)
drawn from the source.
Thus the voltage across the load is constant and
This is because a practical voltage source has an
the current is independent of the internal resistance Ro
internal resistance (output resistance), Ro, on account
of the source. Thus the source behaves as a constant
of which the voltage across the load resistance, EL,
voltage source as indicated in region N of Fig. 25.39(e).
decreases with increase of current or decrease of load
An ideal current source furnishes a current that is
resistance RL.
independent of the load resistance. It remains constant
Figure 25.39(d) shows a practical voltage source
even if the load resistance varies. The current
which consists of a voltage source E in series with an
produced by a constant current source may or may not
internal resistance R(/ and a load resistance RL.
vary with time.
The current in this circuit is given by,
If a constant current passes through a resistor R,
£
1 =--- - ---- ...(25.39) as shown in Fig. 25.40(a), a potential difference occurs
Rno + RLr between the terminals which is given by E = IR. In a
Primary Sensing Elements and Transducers 765

strict sense, as in the case of constant voltage source, a teristics of a constant voltage source. On the other
constant current source does not exist. A practical hand, if the output impedance of the transducer is
current source contains a voltage source E which much higher than that of the succeeding stages, it
causes the current. A constant current source has an exhibits characteristics of a constant current source.
internal resistance Ry which is connected in parallel When the output impedance of the transducer is
with the current source, and is very large as compared equal to that of the following stages of instrumen­
with the load resistance R. The current in such a tation system, matching takes place and maximum
system is determined primarily by magnitudes of E
power is transferred from the transducer to the
and Rl.
succeeding stages. However, it must be understood
The current through load resistance is, that in case maximum power transfer takes place,
^0 I when the output resistance of transducer, is equal to
...(25.43)
the resistance of the succeeding stages, the efficiency is
only 50%. Also, when dealing with a.c. systems, the
If the internal resistance Ro is very large as
impedance matching takes place when the impedance
compared to the load resistance (for instance Ro = 1 kQ
of the load is RL -/XL in case the output impedance is
and Rl = 1Q) the ratio RL/R0 is very small so compared
with I and therefore, load current, rl + 7-xl.
IL= I ...(25.44) Useful Output Range
Therefore, the source behaves as a constant The output range of a transducer is limited at the
current source as depicted in region M of Fig. 25.40(e). lower end by noise signals which may shroud the
There is a region O, between regions M and N where desired input signal. The upper limit is set by the
the source neither behaves as a constant voltage maximum useful input level. The output range can be
source or a constant current source. increased, in some cases, by the inclusion of amplifier
Therefore, we conclude, if the output impedance in the transducer. However, the inclusion of an
of the transducer is low as compared with that of load amplifier also increases the noise level and therefore in
impedance of the successive stages, it has the charac- such situations the amplifier may not be of any use at
all.

25.10 SUMMARY OF FACTORS INFLUENCING


THE CHOICE OF TRANSDUCERS
In the following parts of this chapter, an attempt
has been made to give the details and applications of
the transducers for measurement of different physical
quantities. It will be realised that there are many ways
for measurement of a physical quantity, and in many
cases, there is no best way. The transducers and the
methods used may depend upon the instrumentation
already available and also on the experience of the
user.
Unfortunately most transducers are not sensitive
to just one quantity. If measurements are to be made
under conditions where there is likelihood of two or
more input quantities influencing the transducer, it is
desirable to select a transducer which is sensitive to
the desirable quantity and insensitive to the unwanted
quantity. If this is not possible, ways and means
should be found to eliminate or compensate for the
effects of the unwanted input quantity. The following
is the summary of the factors influencing the choice of
Fig. 25.40 Depiction of constant current source. a transducer for measurement of a physical quantity.
766 Electrical and Electronic Measurements and Instrumentation

1. Operating Principle. The transducers are many under its temperature range. It should be able to work
a times selected on the basis of operating principle in corrosive environments (if the application so requires),
used by them. The operating principles used may be should be able to withstand pressures and shocks and
resistive, inductive, capacitive, optoelectronic, piezo­ other interactions to which it is subjected to.
electric etc. 10. Insensitivity to Unwanted Signals. The
2. Sensitivity. The transducer must be sensitive transducer should be minimally sensitive to unwanted
enough to produce detectable output. signals and highly sensitive to desired signals.
3. Operating Range. The transducer should 11. Usage and Ruggedness. The ruggedness both

maintain the range requirements and have a good of mechanical and electrical intensities of transducer
resolution over its entire range. The rating of the versus its size and weight must be considered while
transducer should be sufficient so that it does not selecting a suitable transducer.
breakdown while working in its specified operating Electrical aspects. The electrical aspects that
12.

range. need consideration while selecting a transducer


4. Accuracy. High degree of accuracy is assured if
include the length and type of cable required.
the transducer does not require frequent calibration Attention also must be paid to signal to noise ratio
and has a small value for repeatability. It may be in case the transducer is to be used in conjunction with
emphasised that in most industrial applications, amplifiers. Frequency response limitations must also
repeatability is of considerably more importance than be taken into account.
absolute accuracy. 13. Stability and Reliability. The transducer

5. Cross sensitivity. Cross sensitivity is a further should exhibit a high degree of stability to be
factor to be taken into account when measuring operative during its operation and storage life.
mechanical quantities. There are situations where the Reliability should be assured in case of failure of
actual quantity is being measured is in one plane and transducer in order that the functioning of the
the transducer is subjected to variations in another instrumentation system continues uninterrupted.
plane. More than one promising transducer design has 14.Static Characteristics. Apart from low static
had to be abandoned because the sensitivity to error, the transducers should have a low non-linearity,
variations of the measured quantity in a plane low hysteresis, high resolution and a high degree of
perpendicular to the required plane has been such as repeatability.
to give completely erroneous results when the The transducer selected should be free from load
transducer has been used in practice. alignment effects and temperature effects. It should
6. Errors. The transducer should maintain the not need frequent calibration, should not have any
expected input-output relationship as described by its component limitations, and should be preferably
transfer function so as to avoid errors. small in size.
7. Transient and Frequency Response. The
25.11 RESISTIVE TRANSDUCERS
transducer should meet the desired time domain
specifications like peak overshoot, rise time, settling It is generally seen that methods which involve
time and small dynamic error. It should ideally have a the measurement of change in resistance are preferred
flat frequency response curve. In practice, however, to those employing other principles. This is because
there will be cutoff frequencies and higher cut off both alternating as well as direct currents and voltages
frequency should be high in order to have a wide are suitable for resistance measurements.
bandwidth. The resistance of a metal conductor is expressed
8.Loading Effects. The transducer should have a by a simple equation that involves a few physical
high input impedance and a low output impedance to quantities. The relationship is
avoid loading effects.
R=pL/ A,
9. Environmental Compatibility. It should be
assured that the transducer selected to work under where R = resistance ; Q,
specified environmental conditions maintains its L = length of conductor ; m,
input-output relationship and does not break down. A = cross-sectional area of conductor ; m2,
For example, the transducer should remain operable and p = resistivity of conductor material; Q-m
Primary Sensing Elements and Transducers 767

Any method of varying one of the quantities Displacement


involved in the above relationship can be the design
basis of an electrical resistive transducer. There are a
number of ways in which resistance can be changed
by a physical phenomenon. The translational and
rotational potentiometers which work on the basis of
change in the value of resistance with change in length
of the conductor can be used for measurement of
translational or rotary displacements. Strain gauges
work on the principle that the resistance of a
conductor or a semi-conductor changes when strained.
This property can be used for measurement of
displacement, force and pressure. The resistivity of (rt) Linear (translational) POT
materials changes with change of temperature thus
causing a change of resistance. This property may be
used for measurement of temperature. Thus electrical
resistance transducers have a wide field of
application.

25.12 POTENTIOMETERS
Basically a resistance potentiometer, or simply a
POT, (a resistive potentiometer used for the purposes
of voltage division is called a POT) consists of a
resistive element provided with a sliding contact. This
sliding contact is called a wiper. The motion of the
(b) Rotary POT
sliding contact may be translatory or rotational. A
linear pot and a rotary pot are shown in Figs. 25.41(67) Fig. 25.41 Resistive potentiometers (POTs)
and (b) respectively. Figure 25.42 shows the diagrams for translational,
Some POTS use the combination of the two single turn rotational, and multitum helix potentio­
motions, i.e., translational as well as rotational. These meters.
POTS have their resistive element in the form of a
helix and therefore, they are called helipots.
The translational resistive elements are straight
devices and have a stroke of 2 mm to 0.5 m. The
rotational devices are circular in shape and are used
for measurement of angular displacement. They may
have a full scale angular displacement as small as 10°.
A full single turn potentiometer may provide accurate
measurements upto 357°. Multiturn potentiometers
may measure upto 3500° of rotation through use of
helipots.

The helical resistive elements are multiturn


rotational devices which can be used for measurement
of either translational or rotary motion. The resistive
element of the POT may be excited by either d.c. or a.c.
voltage. The POT is a passive transducer since it
requires an external power source for its operation.
The resistive body of potentiometer may be wire
wound. A very thin, 0.01 mm diameter of platinum or
nickel alloy is carefully wound on an insulated former.
The resistance elements are also made up from cermet, pig. 25.42 Diagrams for translational,
hot moulded carbon, carbon film and thin metal. rotational and helipots.
768 Electrical and Electronic Measurements and Instrumentation

Let us confine, our discussion of d.c. excited The circuits shown in Fig. 25.42 are called
potentiometers. Consider a translational potentio­ potentiometer dividers since they produce an output
meter as shown in Fig. 25.42(a). voltage which is a fraction of the input voltage. Thus
Let, the input voltage is "divided". The potential divider is
a device for dividing the potential in a ratio
e. & eQ = input and output voltages
determined by the position of the sliding contact.
respectively ; V,
xt = total length of translational pot; m,
xt = displacement of wiper from its zero
position ; m,
R? = total resistance of the potentiometer ; Q

If the distribution of the resistance with respect to


translational movement is linear, the resistance per
unit length is Rp/xv

The output voltage under ideal conditions is :


resistance at the output terminals^
*o = input voltage
resistance at the input terminals
Equations 25.46 and 25.47 are based upon the
...(25.45) assumption that the distribution of resistance with
respect to linear or angular displacement is uniform
and the resistance of the voltage measuring device (i.e.
The under ideal circumstances, the output
output device) is infinite.
voltage varies linearly with displacement as shown in
Fig. 25.43(a). However, in practice, the output terminals of the
pot are connected to a device whose impedance is
finite. Thus, when an electrical instrument, which
forms a load for the pot and is connected across the
output terminals, the indicated voltage is less than
that given by Eqn. 25.46. The error, which is referred
to as a loading error is caused by the input resistance
of the output device.
Let us consider the case of a translational poten­
tiometer as shown in Fig. 25.44. Let the resistance of a
meter or a recorder monitoring the output be R .
As explained earlier if the resistance across the
Fig. 25.43 Characteristics of potentiometers. output terminals is infinite, we get a linear relation­
ship between the output and the input voltage.
Sensitivity s = °UtF’- c0= (xi/xt)ei = Kei ...(25.48)
input
p p where K = x-/xf
= -0=_L ...(25.46)
xi xt However, under actual conditions the resistance,
Rm, is not infinite. This causes a non-linear
Thus under ideal conditions the sensitivity is
relationship between the output and input voltages.
constant and the output is faithfully reproduced and
has a linear relationship with input. The same is true 25.12.1 Loading Effect
of rotational motion. The resistance of the parallel combination of load
Let 0; = input angular displacement in degrees, resistance and the portion of the resistance of the
and 0f = total travel of the wiper in degrees. potentiometer is :
Output voltage eQ = er (0. / 0f) ...(25.47) RR,,R„,
...(25.49)
This is true of single turn potentiometers only. (xf/x,)Rp + R„, KRp + Rm
Primary Sensing Elements and Transducers 769

The total resistance seen by the source is : Except for the two end points where K=0 i.e.,
RRpR.n x. =0 and K = 1 where x; =x, the error is always
R=Rp(l-K) + negative. Figure 25.45 shows a plot of the variation in
KRP+ Rm error with the slider position for different ratios of the
load (output device or meter) resistance to the
KR^(1-K)4 RpR„,
...(25.50) potentiometer resistance.
KR,, + R,„ The error as indicated in Fig. 25.45 is actually
~ L • ei negative. Examine Fig. 25.45, the maximum error is
Current i = — about 12 per cent of full scale if Rm^p = I- This error
R
drops down to about 1.5 per cent when Rm/Rp = 10. For
values of R„/Rp > 10, the position of maximum error
' KR2p(l-K)+RpRm
occurs in the vicinity of x;- / xf= 0.67.
The output voltage under load conditions is :
RRpRm

^Rp + RJ ;; KRpRm
(KR p+Rm)
" KRp(l-K)+RpRm *

=--------------------------- ... (25.51)


K(l-K)(Rp/Rm) + l

The ratio of output voltage to input voltage under


load conditions is :
eo __________ R________ ...(25.52)
e. K(l-K)(Rp/R„;) + l
Fig. 25.45 Variation of error due to loading
Equation 25.52 shows that there exists a effect of a potentiometer.
non-linear relationship between output voltage and
input displacement xj since K-xff xr In case Rm = oo, Maximum percentage error
eQ I e- = K. ^ax=15x(Rp/Rm) ...(25.55)
It is evident from Eqn. 25.52 that as the ratio of
It should be understood that the error in POTS is
RffRp decreases, the non-linearity goes on increasing.
on account of the non-linearity effect produced by the
This is shown in Fig. 25.43(b). Thus, in order to keep
linearity, the value of Rm/Rp should be as large as output device used for measurement whose input
possible. However, when we have to measure the resistance, R , is finite. If meter resistance were
output voltage with a given meter, the resistance of infinite there would be no linearity effects and the
the potentiometer, Rp, should be as small as possible. output voltage will be a linear function of input
displacement x- as shown in Fig. 25.43 and
Error = output voltage under load consequently, there would no error because of the
- output voltage under no load
absence of non-linearity.
=---- ---- —---------------e. K 25.12.2 Power Rating of Potentiometers
[K(l-K)(Rp/R,„) + l] '
The potentiometers are designed with a definite
K2(K-1) power rating which is related directly to their heat
...(25.53)
K(l-K)+Rm/Rp dissipating capacity. The manufacturer normally
designs a series of potentiometers of single turn with a
Based upon full-scale output, this relationship diameter of 50 mm with a wide range of ohmic values
may be written as : ranging from 100 Q to 10 kQ in steps of 100 Q. These
K2(K-1) potentiometers are essentially of the same size and of
xlOO ...(25.54) the same mechanical configuration. They have the
K(l-R) + (Rm/Rp)
same heat transfer capabilities. Their rating is typically
770 Electrical and Electronic Measurements and Instrumentation

5 W at an ambient temperature of 21°C This limits The linearity can be improved by using a
their input excitation voltage. Since power P = ej I Rp, resistance Rjn (made equal to the meter resistance Rm)
the maximum input excitation voltage that can be as shown in Fig. 25.46 without using a meter of higher
used is : resistance. The input-output characteristics with and
without Rjft in the circuit are shown in Fig. 25.46(b).
(“’.Lax = volt -(25.56)

25.12.3 Linearity and Sensitivity


It has been explained earlier that in order to
achieve a good linearity, the resistance of
potentiometer R , should be as low as possible when
using a meter for reading the output voltage which
has a fixed value of input resistance R .
In order to get a high sensitivity the output
voltage eQ should be high which in turn requires a high
input voltage, ef. Due to limitations of power
dissipation as is clear from Eqn. 25.56, the input
voltage is limited by the resistance of the potentio­ Fig. 25.46 Linearization of POT characteristics,
meter. In order to keep the power dissipation at a low through use of an external resistance Rln = Rm.
level, the input voltage should be small and resistance
of the potentiometer should be high. Thus for a high 25.12.4 Construction of Potentiometers
sensitivity, the input voltage should be large and this The resolution of the potentiometers influences
calls for a high value of resistance R . On the other, if the construction of their resistance elements.
we consider the linearity, the resistance of potentio­ Normally, the resistive element is a single wire of
meter should be as low as possible. The resistance conducing material which gives a continuous stepless
of the potentiometer, Rp, cannot be made low because
variation of resistance as the wiper travels over it.
if we do so the power dissipation goes up with the
Such potentiometers are available but their length (in
result that we have to make the input voltage small to
the case of translational potentiometers) and diameter
keep the power dissipation to the acceptable level.
(in the case of rotational potentiometers) restrict their
This results in lower sensitivity.
use on account of space considerations.
Thus linearity and sensitivity are therefore two
The resolution of the potentiometers is dependent
conflicting requirements. If R? is made small, the
linearity improves, but a low value of R , requires a upon the construction of the resistive element and in
lower input voltage e- in order to keep down the order to get high values of resistance in small space
power dissipation and a low value of e- results in a wire wound potentiometers are used extensively. The
lower value of output voltage eQ resulting in lower resistance wire is wound on a mandrel or a card for
sensitivity. Thus the choice of potentiometer translational displacement as shown in Fig. 25.47(a).
resistance, R , has to be made considering both the For the measurement of rotational motion these
linearity ana sensitivity and a compromise between mandrels or cards are formed into a circle or a helix.
the two conflicting requirements has to be struck.
The maximum available sensitivity of poten­
tiometer varies considerably from type to type and
also with size in a given type. The sensitivity can be
calculated from the data supplied by the manu­
facturers which includes maximum allowable voltage,
current or power and the maximum stroke. The
shorter stroke devices have generally a higher
sensitivity. Extreme values are of the order of
15 V/degree for short stroke sector type rotational
potentiometers and 12 V/mm for short stroke
(6.25 mm) translational potentiometers. These, in fact,
are maximum achievable values and usual values of
sensitivity are 10 to 100 times smaller.
Primary Sensing Elements and Transducers J 71

25.12.5 Helipots
The resolution can be increased by using multi­
turn potentiometers. These are called helipots. The
resistance element is in the form of a helix and the
wiper travels along a "lead screw". The number of
turns is still limited to 20 to 40 turns per mm but an
increase in resolution can be obtained by using a
gearing arrangement between the shaft whose motion
is to be measured and the potentiometer shaft. As an
example, one rotation of the measured shaft can cause
10 rotations of the potentiometer shaft. This increases
the resolution of the measured shaft motion by
(b) Circular (Rotational) potentiometer.
10 times. Multitum potentiometers are available to
Fig. 25.47 Wire wound potentiometers. about 60 turns.
This is shown in Fig. 25.47(b). If wire wound type of Similar magnifying or amplifying techniques can
construction is adopted, the variation of resistance is be used for translational motion as well.
not a linear continuous change but is in small steps as
the sliding contact (wiper) moves from one turn to 25.13 NON-LINEAR POTENTIOMETERS
another. This is shown in Fig. 25.48. FUNCTION GENERATORS
In many instrumentation problems, non-linear
conditions arise. These may include the effect of
magnetic saturation, the non-linearity of the voltage
produced by a thermocouple as a function of
temperature, the air speed expressed in Mach number
as a function of wind-tunnel pressures, trigonometric
functions, etc. One simple method of solving these
non-linear functions is by the use of non-linear poten­
Fig. 25.48 Translational potentiometer and tiometers. A standard self-balancing potentiometer is
its characteristics. used to produce a displacement proportional to the
input voltage e- is shown in Fig. 25.49. This
Since the variation in resistance is in steps the
resolution is limited. For instance, a translational
displacement also operates a non-linear resistance 25
which produces an output voltage which is the
potentiometer has about 500 turns of a resistance wire
desired function of the input voltage e-. This operation
on a card of 25 mm in length and for this device the
is illustrated in the following example. Let the input
resolution is limited to 25/500 = 0.05 mm = 50 pm.
voltage e- vary from 0 to 10 V. The desired function of
The actual practical limit is 20 and 40 turns per
this voltage is as plotted in Fig. 25.49(b).
mm. Thus for translational devices the resolution is
eQ = 0.3 ei + 0.5 e2 ...(25.58)
limited to 25 -50pm.
The maximum output voltage
For rotational devices, the best angular resolution
3 to 6 , e0(max) = 03 X 10 + 03 X 0 °)2 = 53 V-
=------ degrees ... (25.57)
A displacement proportional e- is obtained using
where D = diameter of the potentiometer ; mm an amplifier and a servomotor as shown in
In order to get higher resolution, thin wires which Fig. 25.49(rt). A good approximation of the required
have a high resistance have to be put close to each resistance of the non-linear potentiometer is obtained
other and they can be closely wound on account of by shunting a linear potentiometer at 10 equally
their small diameter. Thus the resolution and total spaced taps with the correct resistances. The desired
resistance are interdependent. voltage above ground is computed from Eqn. 25.58
In case a fine resolution and high resistance are and are listed in Table 25.3. The current is assumed to
required a carbon film or a conductive-plastic be 1 mA. The resistance R is the resistance from each
resistance elements are used. Carbon film resistive tap to ground, and AR is the resistance between
elements have a resolution of 12.5 nm. adjacent taps. These resistances are computed and
772 Electrical and Electronic Measurements and Instrumentation

listed in Table 25.3. The circuit of the non-linear The maximum resistance between taps is 980 fl
potentiometer with resistances between each tap is Then a 10,000 Q, 10-tap potentiometer with 1,000 fl
shown in Fig- 25.49. between taps can be used with the correct shunt
resistance externally connected between the taps for
the non-linear potentiometer. The shunt resistance Rx
between each pair of taps necessary to produce the
desired non-linearity is,

where is the resistance between the pair of adjacent


taps of the potentiometer without the shunting
resistors, and AR is the desired resistance.
These values are given in Table 25.3. The
difference between the desired function and the ap­
proximation obtained with this 10-tap potentiometer
is the difference between the curve in Fig. 25.49 and
the straight lines connecting the points on the curve.
An adjustable tapped potentiometer is-shown in
Fig. 25.50. This has the advantages that it can be
adjusted to fit any desired curve, and that it is capable
of having both positive and negative voltage outputs.

80 180 280 380 480 580 680 780 880 980

•— ei —*| eo Non-linear potentiometer


----------------------------- o E = 53 V o----------------------------

Fig. 25.49 Non-linear-potentiometer-function


generator.

Table 25.3
e. R AR *x
eo
0 0 0 0
1 0.8 80 80 87
2 2.6 260 180 220
3 5.4 540 280 319
4 9.2 920 380 613
5 14.0 1,400 480 923
6 19.8 1,980 580 1,380
7 26.6 2,660 680 2,124 Fig. 25.50 Adjustable tapped potentiometer.

8 34.4 3,440 780 3,540


When a fixed function is desired, a wire-wound
9 43.2 4,320 880 7,330 potentiometer with the desired taper may be made.
i 10 53.0 5,300 980 49,000 An example of this type of potentiometer is illustrated
Primary Sensing Elements and Transducers 773

It will be remembered that servo circuits are


comparatively slow and are inadequate for fast analog
computers, yet these circuits are very accurate, usually
± 0.5 percent or better.
If the slope of the body on which the resistive wire
is wound is kept accurately uniform, there will be a
linear relationship between the output voltage and the
position of the wiper (input displacement). However,
if the shape of the body varies, it will cause a
prescribed non-linear transfer function. This is shown
in Fig. 25.52.
There are many standard functions available, or
custom position-to-voltage relationship can be
produced. This non-linear relationship may be used to
Fig. 25.51 Sine-cosine, tapered potentiometer.
compensate for a non-linearity in the quantity being
in Fig. 25.51, where the taper is shown for a measured or to introduce a special function into the
continuous rotating potentiometer with a sine-cosine control and instrumentation scheme without having
output function. The potentiometer shaft is rotated at to resort to special mathematics electronics or a
the desired angular velocity with a servomotor when a computer.
sine-wave input voltage is desired, or it may be
Operated with a self-balancing potentiometer circuit, 25.14 MATERIALS USED FOR POTENTIOMETERS
as shown in Fig 25.49 when the desired output is, The materials used for POTS may be classified as
= a sin e,- wire wound and non-wire wound.

or when the axis is shifted 90°. 1. Wire Wound Potentiometers


Cq = b cos ei These are platinum, nickel chromium, nickel
copper, or some other precious resistance elements.
Wire wound potentiometers carry relatively large
currents at high temperatures. Their resistance
temperature co-efficient is usually small, is of the
order of 20 x 10~6/°C or less. Their resolution is about
0.025 - 0.05 mm and is limited by the number of turns
that can be accommodated on the card.
It should be noted that the interwinding
capacitance between turns, and between windings
and shaft, housing etc. limits the use of wire wound
potentiometers to low frequencies. The response is
limited to about 5 Hz.

2. Non-Wire Potentiometers
These are also called continuous potentiometers. The
non-wire wound potentiometers provide improved
resolution and life. This is because the resolution is no
longer limited by the number of turns that can be
wrapped onto a body. The wiper moves across a
smooth surface (not bouncing from turn to turn),
wear, bounce and the resulting failures are decreased.
The maximum speed that a wire wound potentio­
meter is turned is about 300 rpm. About that, the noise
created as the wiper bounces from turn to turn
becomes significant. A continuous potentiometer may
f t9. 25.52 Generation of non-linear functions
with potentiometers. be turned at a speed of 2000 rpm. However, they are
774 Electrical and Electronic Measurements and Instrumentation

more sensitive to temperature changes, have a higher The disadvantages are :


wiper contact resistance, which is variable and can (?) The chief disadvantage of using a linear
carry only moderate currents. potentiometer is that they require a large
The materials used for non-wire wound (or force to move their sliding contacts (wipers).
continuous) potentiometers are : (zz) The other problems with sliding contacts are
(i) Cermet. Cermet uses precious metal particles that they can be contaminated, can wear out,
fused into ceramic base. These fused metal particles become misaligned and generate noise. So
act as resistance elements. The advantages of using the life of the transducer is limited.
Cermet are large power ratings at high temperatures,
However, recent developments have
produced a roller contact wiper which, it is
low cost and moderate temperature coefficients of the
claimed, increases the life of the transducer
order 100 x 10-6 to 200 x 10-6/°C Cermet is very useful
by 40 times.
for a.c. applications.
Example 25.9 A linear resistance potentiometer is
(ii) Hot moulded carbon. The resistance element is
50 mm long and is uniformly wound with a wire having a
fabricated by moulding together a mixture of carbon
resistance of 10,000 □. Under normal conditions, the slider
and a thermosetting plastic binder. Hot moulded
is at the centre of the potentiometer. Find the linear
carbon units are useful for a.c. applications.
displacement when the resistance of the potentiometer as
(iii) Carbon film. A thin film of carbon deposited measured by a Wheatstone bridge for two cases is: (i) 3850 Q,
on a non-conductive base forms the resistance (ii) 7560D. Are the two displacements in the same direction ?
element. The advantage of carbon film potentiometers
If it is possible to measure a minimum value of 10 Q
is their low cost. Temperature coefficients are upto
resistance with the above arrangement, find the resolution
1000xl0-6/°C
of the potentiometer in mm.
(zv) Thin metal film. A very thin, vapour Solution. The resistance of the potentiometer at
deposited layer of metal on glass or ceramic base is its normal position
used as a resistance element. The advantages of this
= 1000/2 = 5000 Q
potentiometer are its excellent resistance to changes in
Resistance of potentiometer per unit length
environments and use on a.c. The cost is also
moderate. = 1000/50 = 200 Q/mm
(z) Change of resistance from its normal position
25.15 ADVANTAGES AND DISADVANTAGES OF = 5000-3850 = 1150 Q
RESISTANCE POTENTIOMETERS
.'. Displacement of wiper from its normal position
Resistance potentiometers have the following
= 1150/200 =5.75 mm
major advantages :
(zz) Change of resistance from its normal position
(i) . They are inexpensive.
= 7560-5000 = 2560 0
(zz) They are simple to operate and very useful Displacement of wiper from its normal position
for applications where the requirements are
2560/200 = 12.80 mm
not particularly severe.
Since, one of the displacements represents a
(zzz) They are very useful for measurement of
decrease and other represents an increase in resistance
large amplitudes of displacement. of potentiometer from its value at the normal position,
(z'v) Their electrical efficiency is very high and they the two displacements are in the opposite direction.
provide sufficient output to permit control Resolution = minimum measurable resistance
operations without further amplification. xmm/Q
(v) It should be understood that while the = 10x1/200 = 0.05 mm
frequency response of wire wound
Example 25.10 A variable potential divider has a total
potentiometers is limited, the other types of
resistance of 2 k£l and is fed from a 10 V d.c. supply. The
potentiometers are free from this problem.
output is connected to a load resistance of 5 k£L Determine
(vi) In wire wound potentiometers the resolution the loading errors for the wiper positions corresponding to
is limited while in Cermet and metal film K =Xj/xt=0, 0.25, 0.5, 0.75 and 1.0. Use the results to plot
potentiometers, the resolution is infinite. a rough graph of error versus xfxt.
Primary Sensing Elements and Transducers 775

Solution. We have Rp = 2 kQ and Rlu = 5 kQ potentiometer R3R4 is adjusted so that the bridge is balanced
and eo = O. Assuming that the displacement being measured
From Eqn. 25.53, error is,
will move a maximum distance of 12.5 mm towards A,
r k2(k~i) calculate the value of e0.
e'[K(l-K)+Rm/Rp Solution. Total length of AB = 125 mm ; Midpoint
of AB = 62.5 mm from either A or B. If the wiper moves
Substituting the values, the results are tabulated 12.5 inward towards A from midstroke, its distance
below : from B becomes 62.5 + 12.5 = 75 mm.

K Error, V £2 =(75/125)x 5000 =3000 Q

0 Hence, output voltage


0
0.25 -0.174 f R2 R4 'I
0 1^+^
0.5 - 0.454
0.75 - 0.524 3000 2500
1 0 5000 5000
The graph between error versus K =xfxt is plotted
Example 25.12 The output of a potentiometer is to be
in Fig. 25.53. (The errors are no doubt - ve but they are
read by a recorder of 10 k£l input resistance. The
shown as +ve in Fig. 25.53.)
non-linearity must be held to 1 percent. A family of
potentiometers having a thermal rating of 5 W and
resistances ranging from 100 Q to 10 kQ. in steps of 100 Q
are available. Choose from the family of potentiometers, a
potentiometer that has the greatest possible sensitivity and
which meets the non-linearity requirements. Find the
maximum excitation voltage permissible with this
potentiometer. What is the sensitivity if the potentiometer is
a single turn (560° unit) ?
Solution. From Eqn. 25.55, maximum possible
percentage linearity = 15 Rp/Rm- This is to be limited to
1 percent 15 Rp/Rm =1- Therefore, the maximum value
of resistance of potentiometer is,

R =i = 12^22 = 666.7Q
p 15 15
Fig. 25.53 Graph of results of Example 25.10. Thus we are left with choice of potentiometers
having resistances of 100 Q, 200 Q, 300 Q, 400 Q, 500 Q
Example 25.11 A resistive potential divider R^R2 with
and 600 Q.
a resistance of5000 Q and a shaft stroke of 125 mm is used
in the arrangement shown in Fig. 25.54. Potentiometer The potentiometer with the highest value of
R3R4 has a resistance of 5000 Q and et=5.0 V. The initial resistance has the highest sensitivity. Therefore, a
position to be used as reference point is such that Rj = R2 potentiometer with R? = 600 Q is selected. With a
i.e., the wiper is at midstroke. At the start of the test power dissipation of 5 W, the maximum value of
excitation voltage is,
A
%ax) = 7™; =757600 = 54.8 V

The sensitivity of potentiometer when it is a single


turn unit S = 54.8/360 = 0.152 V/degree.

Example 25.13 A helipot is provided with


400 turns/mm. The gearing arrangement is such that the
motion of the main shaft causes 5 revolutions of
B potentiometer shaft. Calculate the resolution of the
Fig. 25.54 Diagram of Example 25.11.
potentiometer.
776 Electrical and Electronic Measurements and Instrumentation

Solution. The resolution of potentiometer 100°C and therefore de-rating in power dissipation
without the gearing arrangement = 1/400 mm =25 pm. has to done on account of increase in temperature. The
With the gearing arrangement which causes 5 de-rated power dissipation is 650 mW while the
revolutions of the potentiometer shaft with one potentiometer is dissipating 667 mW and hence it will
revolution of main shaft = 25/5 = 5 pm. fail.
Example 25.14 It is necessary to measure the position 25.16 STRAIN GAUGES
of an object. It moves 0.8 m. Its position must be known
within 1 mm. Part of the mechanism which moves the object If a metal conductor is stretched or compressed,
is a shift in a shaft that rotates 250° when the object is its resistance changes on account of the fact that both
moved from one extreme to the other. A control length and diameter of conductor change. Also there
potentiometer has been found which is rated at 300° full is a change in the value of resistivity of the conductor
scale movement. It has one thousand turns of wire. Is the when it is strained and this property is called
potentiometer suitable for the application ? piezoresistive effect. Therefore, resistance strain
Solution. The shaft provides a gauges are also known as piezoresistive gauges. The
strain gauges are used for measurement of strain and
250°
------= 312.5°/m or 0.3125°/mm conversion associated stress in experimental stress analysis.
0.8
Secondly, many other detectors and transducers,
A resolution of 1 mm at the object translates into notably the load cells, torque meters, diaphragm type
1 mm x 0.3125° = 0.3125° pressure gauges, temperature sensors, accelerometers
Required resolution for the potentiometer. and flow meters, employ strain gauges as secondary
The potentiometer actually has a resolution transducers.

= 0.300° 25.16.1 Theory of Strain Gauges


1000
The change in the value of resistance by straining
.’. The resolution of the potentiometer is higher the gauge may be partly explained by the normal
than that required for the application and hence the dimensional behaviour of elastic material. If a strip of
potentiometer is very much suited for the application.
elastic material is subjected to tension, as shown in
Example 25.15 A control potentiometer is rated as : Fig. 25.55 or in other words positively strained, its
resistance = 150 Q, power rating - 1 W. De-rate the longitudinal dimension will increase while there will
potentiometer by 10 mW/°C above 65°C, thermal resistance
= 30°C/W.
Can the potentiometer be used with a 10 V supply at 80°C
ambient temperature ?
Solution. Power dissipated by potentiometer
P = (10)2/150 = 0.667 W = 667 mW Fig. 25.55 Change in dimensions of a strain gauge
element when subjected to a tensile force.
The actual temperature of the potentiometer is the
sum of the ambient temperature and temperature rise
be a reduction in the lateral dimension. So when a
produced due to self heating.
gauge is subjected to a positive strain, its length
0pot =80° + (667)x30xl0“3 = 100°C
increases while its area of cross-section decreases as
The allowable power dissipation must be shown in Fig. 25.55. Since the resistance of a conductor
decreased by 10 mW for each °C above 65°C is proportional to its length and inversely proportional
Dissipation allowed to its area of cross-section, the resistance of the gauge
increases with positive strain. The change in the value
= 1.0-10x 10“3(100 -35)
resistance of strained conductor is more than what can
= 0.65 W= 650 mW be accounted for an increase in resistance due to
dimensional changes. The extra change in the value of
A first glance, it would appear that the
potentiometer is well below the rated 1 W power resistance is attributed to a change in the value of
dissipated. This could have been true if the resistivity of a conductor when strained. This
temperature of the potentiometer were 65°C. property, as described earlier, is known as
However, the temperature of the potentiometer is peizoresistive effect.

You might also like